わからない問題はここに書いてね 36

このエントリーをはてなブックマークに追加
1132人目のメルセンヌ数さん
   / ̄   ̄ ヽ
  / ,,w━━━.、)   / ̄ ̄ ̄ ̄ ̄ ̄ ̄ ̄ ̄ ̄ ̄ ̄ ̄ ̄ ̄ ̄ ̄
  ! .fw/f_」」_|_|_i_)  | ここは分からない問題について質問するさくらちゃんスレですわ
  ヽ|:::(6||f;j' ,fj'||)  | 関連スレッドや業務連絡,記号の書き方例は >>2-13 辺りに。
 ∠|::i:!::|:|、_ワノ:i、 <  ローマ数字や丸付き数字などの機種依存文字はお勧め出来ませんわ
  .|::|< |::|ヽーノ`l:i;ヽ \_________________
  .ノ:ノ' i:::l `只´|:|i)::)
 (::(:i  |:::|ノ ) j:j|:(

    (⌒, -- 、⌒)     / ̄ ̄ ̄ ̄ ̄ ̄ ̄ ̄ ̄ ̄ ̄ ̄ ̄ ̄ ̄ ̄
  _  Y      Y  _ < 自分でどこまで考えたのか、途中でもいいから
 ミ \| ・  . ・| / 彡 | 書いてくれればこっちも答えやすくて助かるわー
    @ゝ.  ^  ノ@    | 質問者も解答者もくれぐれもトラブルは起こさんといてなー
                \________________

【前のスレッド】
◆ わからない問題はここに書いてね 35 ◆
http://science.2ch.net/test/read.cgi/math/1023277199/l50
【関連スレッド】
雑談はここに書け!【4】
http://science.2ch.net/test/read.cgi/math/1021808853/l50
くだらねぇ問題はここへ書け ver3.1415926535897
http://science.2ch.net/test/read.cgi/math/1022008901/l50

【過去のスレッド】
◆ わからない問題はここに書いてね1〜27 ◆
01 http://cheese.2ch.net/math/kako/967/967755172.html
02 http://cheese.2ch.net/math/kako/970/970795775.html
03 http://cheese.2ch.net/math/kako/974/974911042.html
04 http://cheese.2ch.net/math/kako/978/978209589.html
05 http://cheese.2ch.net/math/kako/981/981372834.html
06 http://cheese.2ch.net/math/kako/985/985594205.html
07 http://cheese.2ch.net/math/kako/988/988952592.html
08 http://cheese.2ch.net/math/kako/991/991223596.html
09 http://cheese.2ch.net/math/kako/993/993571403.html
10 http://cheese.2ch.net/math/kako/995/995448453.html
11 http://cheese.2ch.net/math/kako/997/997329928.html
12 http://cheese.2ch.net/math/kako/999/999689496.html
13 http://cheese.2ch.net/math/kako/1001/10013/1001342715.html
14 http://cheese.2ch.net/math/kako/1002/10028/1002893257.html
15 http://cheese.2ch.net/math/kako/1004/10041/1004171159.html
16 http://cheese.2ch.net/math/kako/1005/10057/1005735838.html
17 http://cheese.2ch.net/math/kako/1006/10068/1006859798.html
18 http://cheese.2ch.net/math/kako/1007/10078/1007834117.html
19 http://cheese.2ch.net/math/kako/1009/10091/1009102965.html
20 http://cheese.2ch.net/math/kako/1010/10107/1010708150.html
21 http://cheese.2ch.net/math/kako/1011/10116/1011689052.html
【掲示板での数学記号の書き方例】
■数の表記
●スカラー:a,b,c,...,z, A,B,C,...,Z, α,β,γ,...,ω, Α,Β,Γ,...,Ω, ... (← ギリシャ文字はその読み方で変換可.)
●ベクトル:V=[V[1],V[2],...], |V>, V↑, vector(V) (← 混同しない場合はスカラーと同じ記号でいい.通常は縦ベクト
ルとして扱う.)
●テンソル(上下付き1成分表示):T^[i,j,k...]_[p,q,r,...], T[i,j,k,...;p,q,r,...]
●行列(1成分表示):M[i,j], I[i,j]=δ_[i,j]
●行列(全成分表示):M=[[M[1,1],M[2,1],...],[M[1,2],M[2,2],...],...], I=[[1,0,0,...]',[0,1,0,...],...] (← 行(または列ごと)に表
示する.)

■演算・符号の表記
●足し算:a+b
●引き算:a-b
●掛け算:a*b, ab (← 通常"*"を使い,"x"は使わない.)
●割り算・分数:a/b, a/(b+c), a/(bc) (← 通常"/"を使い,"÷"は使わない.)
●割り算分数2:(a+b)/(c+d),a+(b/c),(a/b)+c(←括弧を用い分子分母を他の項と区別できるように表現する。)
●複号:a±b=a士b, a干b (← "±"は「きごう」で変換可.他に漢字の"士""干"なども利用できる.)
●内積・外積・3重積:a・b, axb, a・(bxc)=(axb)・c=det([a,b,c]), ax(bxc)

■関数・数列の表記
●関数:f(x), f[x]
●数列:a(n), a[n], a_n
●平方根:√(a+b)=(a+b)^(1/2) (← "√"は「るーと」で変換可.)
●累乗根:[n] √(a+b)=(a+b)^(1/n)
●指数・指数関数:a^b, x^(n+1), exp(x+y)=e^(x+y) (← "^"を使う."exp"はeの指数.)
●対数・対数関数:log_{a}(b), log(x/2)=log_{10}(x/2), ln(x/2)=log_{e}(x/2) (← 底を省略する場合,"log"は常用対数,
"ln"は自然対数.)
●三角比・三角関数:sin(a), cos(x+y), tan(x/2)
●行列式・トレース:|A|=det(A), tr(A)
●絶対値:|x|
●ガウス記号:[x] (← 関数の変数表示などと混同しないように注意.)
●共役複素数:z~
●転置行列・随伴行列:M', M† (← "†"は「きごう」で変換可.)
●階乗:n!=n*(n-1)*(n-2)*...*2*1, n!!=n*(n-2)*(n-4)*...
●順列・組合せ:P[n,k]=nPk, C[n.k]=nCk, Π[n,k]=nΠk, H[n,k]=nHk (← "Π"は「ぱい」で変換可.)

■微積分・極限の表記
●微分・偏微分:dy/dx=y', ∂y/∂x=y,x (← "∂"は「きごう」で変換可.)
●ベクトル微分:∇f=grad(f), ∇・A=div(A),∇xA=rot(A), (∇^2)f=Δf (← "∇"は「きごう」,"Δ"は「でるた」で変換
可.)
●積分:∫[0,1]f(x)dx=F(x)|_[x=0,1], ∫[y=0,x]f(x,y)dy, ∬_[D]f(x,y)dxdy, 点[C]f(r)dl (← "∫"は「いんてぐらる」,"∬
"は「きごう」で変換可.)
●数列和・数列積:Σ_[k=1,n]a(k), Π_[k=1,n]a(k) (← "Σ"は「しぐま」,"Π"は「ぱい」で変換可.)
●極限:lim_[x→∞]f(x) (← "∞"は「むげんだい」で変換可.)

■その他
●図形:"△"は「さんかく」,"∠"は「かく」,"⊥"は「すいちょく」,"≡"は「ごうどう」,"∽"は「きごう」で変換可.
●論理・集合:"⇔⇒∀∃∧∨¬∈∋⊆⊇⊂⊃∪∩"は「きごう」で変換可.
●等号・不等号:"≠≒≦≧≪≫"は「きごう」で変換可.

※ ここで挙げた表記法は1例であり,標準的な表記法からそうでないものまで含まれているので,後者の場合使う
時にあらかじめことわっておいたほうがいい.
※ 関数等の変数表示や式の括弧は,括弧()だけでなく[]{}を適当に組み合わせると見やすい場合がある.
※ 上記のほとんどの数学記号や上記以外の数学記号は大体「きごう」で順次変換できる.
【一般的な記号の使用例】
a:係数、数列 b:係数、重心
c:定数、積分定数 d:微分、次数、次元、距離、外微分、外積、公差
e:自然対数の底、単位元、分岐指数、基底、離心率 f:関数、多項式、基底
g:関数、多項式、群の元、種数、計量、重心 h:高さ、関数、多項式、群の元、類数、微小量
i:添え字、虚数単位、埋めこみ、内部積 j:添え字、埋めこみ、j-不変量、四元数体の基底
k:添え字、四元数体の基底、比例係数 l:添え字、直線、素数
m:添え字、次元、Lebesgue測度 n:添え字、次元、自然数
o:原点 p:素数、射影
q:素数、exp(2πiτ) r:半径、公比
s:パラメタ、弧長パラメタ t:パラメタ
u:ベクトル v:ベクトル
w:回転数 x:変数
y:変数 z:変数(特に複素数変数)

A:行列、環、加群、affine空間、面積
B:行列、開球、Borel集合、二項分布
C:複素数体、連続関数全体の集合、組み合わせ、曲線、積分定数、Cantorの3進集合、チェイン複体
D:関数の定義域、微分作用素、判別式、閉球、領域、二面体群、Diniのderivative、全行列環
E:単位行列、楕円曲線、ベクトル束、単数群、辺の数
F:原始関数、体、写像、ホモトピー、面の数
G:群、位相群、Lie群
H:Hilbert空間、Hermite多項式、部分群、homology群、四元数体、上半平面、Sobolev空間、重複組み合わせ
I:区間、単位行列、イデアル
J:Bessel関数、ヤコビアン、イデアル、Jacobson根基
K:体、K群、多項式環、単体複体、Gauss曲率
L:体、下三角行列、Laguerre多項式、L関数、Lipschitz連続関数全体の集合、関数空間L^p、線型和全体
M:体、加群、全行列環、多様体
N:自然数全体の集合、ノルム、正規部分群、多様体
O:原点、開集合、整数環、直交群、軌道、エルミート演算子
P:条件、素イデアル、Legendre多項式、順列、1点、射影空間、確率測度
Q:有理数体、二次形式
R:半径、実数体、環、可換環、単数規準、曲率テンソル、Ricciテンソル
S: 級数の和、球面、部分環、特異チェイン複体、対称群、面積、共分散行列
T:トーラス、トレース、線形変換
U:上三角行列、unitary行列、unitary群、開集合、単数群
V:ベクトル空間、頂点の数、体積
W:Sobolev空間、線形部分空間
X:集合、位相空間、胞複体、CW複体、確率変数、ベクトル場
Y:集合、位相空間、ベクトル場、球面調和関数 Z:有理整数環、中心
【一般的な記号の使用例 2】
α:定数、方程式の解 β:定数、方程式の解
γ:定数、Euler定数、曲線 δ:微小量、Diracのdelta関数、Kroneckerのdelta
ε:任意の正数、実二次体の基本単数、Levi-Civitaの記号
ζ:変数、zeta関数、1の冪根
η:変数 θ:角度
ι:埋めこみ κ:曲率
λ:定数、測度、固有値、Z_p拡大の不変量、モジュラー関数
μ:定数、測度、Z_p拡大の不変量、Mobiusの関数
ν:測度、付値、Z_p拡大の不変量
ξ:変数 ο:Landauの記号
π:円周率、射影、素元、基本群
ρ:rank、相関係数
σ:標準偏差、置換、σ関数、単体、σ代数
τ:置換、群の元、捩率 υ:
φ:空集合、写像、Eulerの関数
χ:Euler標数、特性関数、階段関数  ψ:写像
ω:character、1の3乗根、微分形式

Β:beta関数  Γ:gamma関数、SL(2、R)の離散部分群、Christoffelの記号
Δ:微小変化、対角線集合、対角線写像、weight12のcusp form、単位円板、ラプラシアン、行列式
Λ:作用域、添え字集合、対角行列 Π:積記号
Σ:和記号、素体、(共)分散行列 Ο:Landauの記号
Φ:写像 Ψ:写像
Ω:代数的平方、拡大体、領域
【業務連絡】
■900を超えたら新スレに移行準備.
■旧スレ側 → 終了宣言,新スレへの誘導.
■新スレ側 → 開始宣言と目次,旧スレのリンク,掲示板での数学記号の書き方例,
  業務連絡・その他,旧スレ側の残り問題の移動.
■数学板の要望スレで数学板の注意書き(リンク先)の変更依頼.
■単独の質問スレは,このスレか「くだらんスレ」に誘導して下さい.
■誤って過去スレに新たに書き込まれた質問は,最新スレに誘導して下さい.
【数学板削除依頼スレ】
http://kaba.2ch.net/test/read.cgi/saku/986384122/ (レス削除)
http://kaba.2ch.net/test/read.cgi/saku/987829968/ (スレッド削除)
【ローカルルール等リンク先更新総合スレッド2】
http://kaba.2ch.net/test/read.cgi/accuse/1012720188/l50
★__________________________.
|              │
│ はにゃ〜ん     |
| γ∞γ~  \    |
│人w/ 从从) )   │
│ ヽ | |┬ イ |〃  │
│ `wハ~ . ノ)    │
│  / \`「 .     │
| 数学板さくらスレ  |
|_________________________│

〃二二ヽ
| |77777〉
| | ゚д゚ノ|  サクラチャンノハタケイヨウデスワ
|⊂   つ
8132人目のメルセンヌ数さん :02/06/15 19:50
━━━━━━━━━━━━━━━━━━━━━━━━━━━━━━━━━━━━━

                    移転が完了しましたわ♪
              ◆ わからない問題はここに書いてね 36 ◆
          いよいよ始まります それではみなさま心置きなくどうぞ

━━━━━━━━━━━━━━━━━━━━━━━━━━━━━━━━━━━━━
9DoYouRock?!?!:02/06/15 20:03
おつかれベイべー!
10DoYouRock?!?!:02/06/15 20:05
さぁぁぁどんどん質問いってみよぉかぁぁ!
11DoYouRock?!?!:02/06/15 20:06
まずは泣く子も黙る青いルビー、>>12番から。
準備だけで8レスですか
http://science.2ch.net/test/read.cgi/math/1014030375/l50
の知名度って低いのかな。
>>1の名前は“132人目のともよちゃん”
・タイトルの前後は◆
>>1がローカルルールの変更を申請してくる

15132人目の素数さん:02/06/15 21:40
「うちのママって変なのよ♪」
   ∧_∧   ∧_∧
  ( ・∀・) ( ´∀`)
 ⊂    つ⊂    つ
  .人  Y   人  Y
  し'(_)   し'(_)

 「叔母さんにリンゴを渡すとき♪」
   ∧_∧  ∧_∧
  (・∀・ ) (´∀` )
 ⊂、   つ⊂、   つ
    Y 人    Y 人
   (_)'J   (_)'J

  「『つまらないものですけど』なんて♪」
  ∧_∧  ∧_∧
 ( ・∀・ ) ( ´∀` )
 ( つ⊂ ) ( つ⊂ )
  ヽ ( ノ  ヽ ( ノ
 (_)し'  (_)し'

    「だったらageなきゃいいのにNe♪」
   ∧_∧   ∧_∧
  ∩ ・∀・)∩∩ ´∀`)∩
   〉     _ノ 〉     _ノ
  ノ ノ  ノ  ノ ノ  ノ
  し´(_)   し´(_)
P,n∈Zに対してP/2^nの形の有理数の集合はRにおいて稠密であることを証明せよ

いったいどうすればいいですか・・・
17132人目の素数さん:02/06/15 22:11
解りません。助けてください・・・

ガウスの定理を使って、次の面積分を計算しなさい。
 (1)A↑=axi↑+byj↑+czk↑(a、b,cは定数)の時、
    原点を中心とする半径rの球面上でのA↑の面積分。

 (2)x=0,y=0,z=0,x=3,y=3,z=3で囲まれた立方体の表面をSとする時、
    S上でのA↑=x^3i↑+x^2yj↑+zk↑の面積分。

 (3)六つの平面x=0,y=0,z=0,x=2,y=2,z=2で囲まれた立方体を
    考える時、立方体にない表面のうちxy平面にはない部分をS,
    Sとxy平面がつくる正方形をCとする。
    A↑=(yz^2+x^2)i↑+(xyz+x^2y)j↑+(y^2z-xz^2)k↑として、
    ストークスの定理が実際に成り立つことを確かめなさい。

 ベクトルの表記が解り難かったらすみません・・・。
 
18132人目の素数さん:02/06/15 23:01
age
19都内中学3年生:02/06/15 23:13
〔(√a+b)(√c+d)〕の平方根の外し方てどんな感じでしょうか?
学校の先生に平方根を外して式を変形しなさいと宿題を出されたけど
訳がわかりません。
20132人目の素数さん:02/06/15 23:18
√a+b or √(a+b) ?
>19
〔(√a+b)(√c+d)〕

これは

((√a)+b)((√c)+d)の意味か?
それとも
(√(a+b))(√(c+d))の意味か?

それと一番外側の〔 〕はどういう意味か?

問題を全部書くように。
訳が分かってない奴が略すと
話がよけいに分からなくなる。
>17
そのままの問題が教科書に載ってると思うんですけど…
23都内中学3年生:02/06/15 23:25
すいませんでした。「二重平方根の外し方を考えてきなさい」という宿題で
式は((√a)+b)((√c)+d)という意味です。外側の〔〕は不必要でした。

あらためて ((√a)+b)((√c)+d)の平方根の外し方です。
24132人目の素数さん:02/06/15 23:30
>>23
これのどこがニ重根号なのかと。
二重根号ってのは√の中に√がある形だ。
何が二重平方根なんだ。
大体、厨房の範囲ではないと思われ。
>16
∀r∈Rの近くにP/2^nの形の有理数が無限にいっぱいあることを示せばよい
Rではなくて区間[0,1)で示せれば後z∈Zだけずらした物(P/2^n)+zは
通分すればP/2^nの形の有理数になっているので区間[0,1)だけ見てれば十分

この区間の任意の点rを取ってその近傍を見るわけだけど
[0,1)の2等分点、4等分点、8等分点…と取っていけば
rの近くに(2^n)等分点が沢山できるけど、これってP/2^nの形の有理数に
対応してるわけで、rは任意に取ったから[0,1)で稠密になって
結果、R全体で稠密になります。
2724:02/06/15 23:33
ちなみに
√(a+b+2√(ab))=√a+√b
>23
問題を勘違いしているか、写し間違えている可能性しか考えられません。
実数x,yがx^2+4*y^2=1,y>0をみたすとき,z=((x+1)^2+y^2)/((x+1)*y)の最小値を求めよ.
また,最小となるときのxとyの値を求めよ.

この問題をラグランジュ(Lagrange)の未定乗数法を用いて解くことって出来ますか?
3025:02/06/15 23:35
ケコーン。
二重根号のことなのか?
それでもわからんで。
31 :02/06/15 23:37
f(x)=x/{1+e^(1/x)}
の右極限左極限を式コミで求めてください
そして∀x∈Rに対して連続になるようにこの関数を定義してください

eは自然対数の底、Rは実数
32 :02/06/15 23:38
すいません書き忘れです
x→0の極限でおねがいします
>29
4*y^2=1-x^2>0より
-1<x<1だから
(x+1)>0
z=((x+1)^2+y^2)/((x+1)*y)
=((x+1)/y)+(y/(x+1))
≧2(相加相乗平均)

等号成立はy^2=(x+1)^2

未定乗数法はそのまま適用してみれ。
>31
(分母)>1,(分子)→0
だから右も左も0でいいと思うのだが。素直に左右わけて書いたほうがいいかね?
x=0以外では連続。ということで
f(0)=0
35 :02/06/16 00:01
>>34
なるほど、、、よくわかりましたありがとうございます
36 :02/06/16 00:03

Σn/(3^n)
n=1

の収束性を判定してください!!
37132人目の素数さん:02/06/16 00:12
>>36
n/(3^n) < (1/2)^nですが何か?
>36
何か似たようなのを最近見た気がする。
第n部分和を出してその極限程度でいいのかな。
3937:02/06/16 00:24
>>38

ザンネン

>>36
3/4と出ましたが何か?
40132人目の素数さん:02/06/16 00:26
>17
1) (a+b+c)*球の体積だぜ
41 :02/06/16 00:29
>>37
その過程をお願いします
>>38
これの部分和といいますと、、どうすればいいのでしょう?
42132人目の素数さん:02/06/16 00:36
>>34
なぜx=0で不連続なんでしょう?
>36
前スレの812と同じ。
44カリメロ ◆pHFzeqzk :02/06/16 01:45
縦ベクトルとか横ベクトルって(・∀・)何?
>44
縦ベクトル…数を縦に並べたもの
横ベクトル…数を横に並べたもの
>42
式中に1/xがあり
これはx=0では不能であり
定義されていないので
この式ではf(0)が定義されないため。
f(0)=0等と別途定義する必要がある。
>36
Sn=1/3+2/3^2+・・・・・+n/3^n
とすると
Sn-(1/3)Sn=1/3+(1/3^2+1/3^3+・・・・+1/3^n)-n/3^(n+1)
(2/3)Sn=(1/3)(1-1/3^n)/(1-1/3)-n/3^(n+1)
    =(1/2)(1-1/3^n)-n/3^(n+1)
n→∞のとき 1/3^n→0, n/3^(n+1)→0
だから
(2/3)Sn→1/2
Sn→3/4
48132人目の素数さん:02/06/16 11:52
lim[x-> 0](1/x^2-1/(sinx)^2}=
をお願いします。
49132人目の素数さん:02/06/16 12:13
>48
lim[x-> 0]{(1/x^2)-(1/sinx)^2}ならば

通分して、(sinx)^2=(1-cos2x)/2入れると
(1/x^2)-(1/sinx)^2=(1-cos2x-2x^2)/(x^2(1-cos(2x)))

あとはロピタルで逝ける
50132人目の素数さん:02/06/16 12:36
>49
ありがとうございますぅ。
51132人目の素数さん:02/06/16 12:49
>>46
>>47
どうもご親切にどうもありがとうございます
52132人目の素数さん:02/06/16 13:15
複素数平面上で、z_1=(√3+i)/2、z_2=(1+√3)(1+i)/2、z_3=(1+√3i)/2によって表される点をそれぞれP_1、P_2、P_3
とする。また、α=(2+2√3i)/2として、複素数αz_1、αz_2、αz_3によって表される点をそれぞれQ_1,Q_2,Q_3とする。
(1)△P_1P_2P_3の面積
(2)△Q_1Q_2Q_3の面積
(3)有向線分OQ_2↑と実軸の正方向との間の角

お願いします
>52
(1)例えばP1が原点にくるように平行移動して考える。
(2)α倍すると回転と拡大だから面積は |α|^2 倍
54132人目の素数さん:02/06/16 13:38
>>48-50
ロピOKなら、通分した
{(sinx)^2-x^2}/{x^2(sinx)^2}
の分母分子をそのまま4回ずつ微分してもよい。

{(sinx)^2-x^2}''''=-8cos(2x)→-8
{x^2(sinx)^2}''''=8{3cos(2x)-x^2cos(2x)-4xsin(2x)}→24

>>49のようにすると計算の手間が軽くなる。
与式=[(1-cos2x-2x^2)/(2x^4)]*(x/sinx)^2
とすれば分母の微分がもう少し楽になる。

ロピ無しだと・・・
sinxの級数展開を打ち切って挟んでみる。

0<x<<1,a=-1/3!,b=1/5!で
0<x(1+ax^2)<sinx<x(1+ax^2+bx^4) (←別途証明が必要か)
中略
(1/x^2)[1-{1/(1+ax^2)^2}]<与式=(1/x^2)-{1/(sinx)^2}<(1/x^2)[1-{1/(1+ax^2+bx^4)^2}]
x→0で両端→2a

もっとうまくできないものか。
55全スレ916:02/06/16 13:46
1−ノルムが列和の最大値になることを証明しろという問題が解けません。
教えてください。おねがいいたします。

PSこっちに移ったとは知りませんでした。

56たにし:02/06/16 13:56
>>55
オメーわけわかんねーよ
57Prime[132]:02/06/16 14:05
56==全スレ918
行って来い
58132人目の素数さん:02/06/16 14:09
問.関数y=e^x-e^(-x)の逆関数を求めよ。→東京理科00

とりあえず、e^xをXとおいて、y=X-(1/X)として、
これをX=〜という形に直そうと思うのですが、
ここの変形ができません。
59ヽ(`Д´)ノ ウァァァン :02/06/16 14:10
0,1,2,3,4,5,6
から偶数を造るときの組み合わせって
どうやって解くんですか? ( ゚Д゚)ポカーン
60132人目の素数さん:02/06/16 14:17
>>59
かけ算?足し算?
選べる数字は何個まで?2個?
問題全文書けばあ?
ある物質をうまいこと分解してうまいことくっつけると2倍の大きさにできる。
それを繰り返すと無限大に大きくなる。数学でこの考えを使うとき頭がおかしくなります。
整列可能定理逝ってよし。
>>58
>y=X-(1/X)として、

分母はらってXの二次方程式を解く
>>58
X>0ってのを忘れるなよー
64ヽ(`Д´)ノ ウァァァン :02/06/16 14:30
0,1,2,3,4,5,6の、7個の数字のから3桁の
数字を選んで3桁の数字をつくるとき偶数はいくつできるか。
>>60
65132人目の素数さん:02/06/16 14:43
>>64
おちつけよ。
その文章、意味がわからんぞ。
66暗算:02/06/16 14:45
3桁の数字が0,1,2,3,4,5,6の中にないが何か?
>64
練習:0,1,2,3,4,5,6の7個の数字から3つ選んで3桁の数は何個作れるか
練習2:1,2,3,4,5,6の6個の数字から3つ選んで3桁の偶数は何個作れるか

これができるなら、後は1の位が0の時とそれ以外の場合で場合分け
6858:02/06/16 14:54
ふぅーなんとか解けました。
こんなのテスト中おもいつくのかなぁ?
6958:02/06/16 15:00
>>62-63
とりあえずどうもありがとうございました。
7052:02/06/16 15:04
>>53
(1)(2)は無事とけました。
(3)はどうすればいいのでしょうか?
7158:02/06/16 15:14
もう一題おねがいします。
放物線y=√(x+1)とy=x^2-1の交点を求めたいのですが、

x^2-1=√(x+1)として考えて逝くと、方程式がとけないばかりか、
実際には2交点しか存在し得ないのに4つでてきてしまいます。
どうすればよいのでしょうか?
>52
z2と実軸のなす角=1+i と実軸のなす角=45°
αと実軸とのなす角=1+(√3)iと実軸とのなす角=60°
掛け算すると角度は足し算、きちんと書くと
z2=(1+√3)*√2/2(cos45°+isin45°)
α=2(cos60°+isin60°)
αz2=(  )(cos105°+isin105°)
7352:02/06/16 15:26
>>72さん
ありがとうございましたm(__)m
>>71
a,bが実数なら
a=√b ⇔ a^2=b,a≧0
7558:02/06/16 15:35
>>74
何とか求まりました。 理解もできました。
ありがとうございました。
76危(゚Д゚)機⇒ベクトル:02/06/16 16:25
こんにちは。お久しぶりです。

■xy平面上に2点、A(-1.-2)B(1.-2)がある。
線分OAを(1-α):αの比に内分する点をP、
線分OBをα:(1-α)の比に内分する点をQとする。
さらに、
線分PQをβ:(1-β)の比に内分する点をRとする。
実数α、βが、0≦α≦1、0≦β≦1をうごく時、点Rの存在する範囲は?
↑α=0.β=0。α=1.β=0。α=0.β=1。α=1.β=1。
そして、α=(1/2).β=(1/2)。α=(1/2).β=0
の合計6点について考えてみて、△OAB上かなと思ったのですが、
実際の範囲はもっと狭くて、↑の範囲が下に凸の放物線でせばめられていました。
↑のように数値を適当に代入していくやり方しか思いつかないのですが、
そのやり方だと、後2箇所くらいしらべて、やっと必要条件という感じです。
どのように考えていけば、よいのでしょうか?

よろしくおねがいします。
77危(゚Д゚)機⇒ベクトル:02/06/16 16:32
もう一題おねがいします。

■次の式で表される3直線A、B、Cのうちのすくなくとも2直線を含む平面
はいくつあるか?ただし、t、u、vは媒介変数とする。
直線
A(x.y.z)=(-1-t.1+2t.-1+3t)
B(x.y.z)=(-2+u.4-u.7+u)
C(x.y.z)=(1+v.-1-2v.2-3v)

直線のベクトル方程式について平面を考える場合
xyzのどれか2つの連立方程式を解いて、でた値を残りの1つに代入して、
出た値が等しくなったら、その2直線による平面が1つ存在するというように考えたのですが、
それだと答えは平面は1つ。となってしまい。
本当の解は平面は2つでした。
方針やヒント、考え方など教えていただきたいです。
よろしくおねがいします。
78高校1年生:02/06/16 17:20
高校1年生のものです。どこに書いたら教えていただけるのか分からなかったので
ここに書かせてもらいます。もし、場違いな内容だったら申し訳ありません。
確率の問題で、どうしても答えが合わないので、どこが間違いか教えてください。
問題)
さいころを4つ振ったとき、5以上の目が2つ以上でる確率をもとめよ。

私は、5以上の目が2つでるとき、3つでるとき、4つでるときで場合わけをして考えました。
1)2つのとき、残りの2つは4以下であり
(5以上のさいころ選び方)=6通り
また、このさいころはそれぞれ5または6の2通りがあるので、この確率は
6×2×4^2÷6^4

2)3つのとき、残りの1つは4以下であり、1)と同様に考えて
4×2×4÷6^4

3)4つのとき
2^4÷6^4

としてすべてを加えたのですが答えとあいません。答えは、11/27です。
また、余事象を使って正解は得られたのですが、上の考え方のどこが間違っているのか
分からないので、教えてください。
79>78:02/06/16 17:32
数え上げ方がおかしい
例えば
2)3つ5以上 1つが4以下
  5 以上 3つ組は(5,5,5),(5,5,6)...(6,6,6) と8 (2^3)通り
 可能な組み合わせは
4 X 2^3 X 4^2  
となる

80132人目の素数さん:02/06/16 17:34
>>55, >>57
行ってきたけどわかんなかった。
列和って何?
超既出ですみません。
ダイヤとティッシュの問題なのですが、条件付確率を使って解くことはできるのでしょうか?
どうもうまく記号にできないのですが。
どなたかご存知の方教えてください。

参考URL ttp://plaza.harmonix.ne.jp/~k-miwa/magic/something/diamond.html
82数学超ドキュソ:02/06/16 17:35
ΔABCにおいて、sinA:sinB:sinC=5:6:7である。
(1)AB:BC:CA=(ア):(イ):(ウ)より、
cosA=(エ)/(オ)、sinA=(カ)√(キ)/(ク)である。
(2)ΔABCの面積が24√6のとき、BC=(ケコ)であり、
ΔABCの外接円の半径は(サシ)√(ス)/(セソ)である。
また、このときのΔABCの内接円の半径は(タ)√(チ)/(ツ)である。

この問題の(2)からがわかりません・・・。
誰か親切な方ヒントを下さいm(__)m
>82
(1)がヒント
S=(1/2)AB・AC・sinA
外接円 2R=a/sinA  aはBCのこと
内接円 S=(1/2)(a+b+c)r
>77
3次元空間で2直線を含む平面が存在するのは、
2直線が平行、または交わるとき
まず平行なときは簡単(方向ベクトルからAとC)
後は交わるかどうか、3通り調べればよい。平行をのぞいて2通りか。
8581:02/06/16 17:59
追加でまたまた超超既出ですがすみません。
「コイン2枚を投げたとき、そのうち1枚は表でした。
そのとき、他の1枚が裏である確率を求めなさい。」
この問題の答えが2/3というのは普通にわかるのですが、
条件付確率を使って解くとなるとどういう式をたてるのでしょうか?
>76
今のところベクトルから座標に持っていくことしか思いつかない。
OR↑=(1−β)(1−α)OA↑+βαOB↑
(x,y)=(1−β)(1−α)(-1,-2)+βα(1,-2)
x,yをα、βで表す。α+β、αβ をx,yで表す。
α、βを解とする2次方程式を作って、実数解が2つとも0と1の間にある条件
87危(゚Д゚)機⇒ベクトル:02/06/16 18:12
>>84さん >>86さん
どうもありがとうございます。
今見直し中です
88132人目の素数さん:02/06/16 18:37
ラウンジで出たのですがどうしても解けません。
数学は得意な方だと思っているのでちょっと考えてみたのですがまったく手付かずで、
ネタかとも思うんですが問題がしっかりしてるし。
答えが気になります。

a,bはa^2+b^2がa*b+1で割り切れるような自然数とする。
このとき(a^2+b^2)/(a*b+1)が完全平方数である事を示せ。

(背理法で行けそうな気がするんですが)
89132人目の素数さん:02/06/16 18:50
>88
1988年数学オリンピックシドニー大会の第6問。
90ドキュソ:02/06/16 19:13
円に内接する四角形ABCDがあり、AB=2,BC=3,CA=4である。
(1)cos∠ABC=-1/4,sin∠ABC=√15/4である。
(2)CD=2のとき、AD=6である。
(3)線分CDの長さの最大値は(クケ)√(コサ)/(シス)である。
(4)四角形ABCDの面積は、CD=(セ)√(ソ)/(タ)のとき、最大値(チツ)√(テト)/(ナニ)である。

という問題がわかりません・・・。
(2)までは何とかわかるので()では無く答えを書いときました・・・。
(3)からのヒントを何方か下さると嬉しいです(>_<)
91亞秋冷:02/06/16 19:43
フーリエ係数の問題です。以下、問題の引用です。

周期2πの関数f(x),g(x)のフーリエ係数をそれぞれ
An(f),Bn(f),An(g),Bn(g)とする。
ここで、c,dを定数とすると、cf(x)+d(g)のフーリエ係数は
cAn(f)+dAn(g),cBn(f)+dBn(g)となる。これを証明せよ。

です。お願いいたします。
92危(゚Д゚)機⇒ベクトル:02/06/16 20:35
>>76-77でベクトルの質問をさせていただきましたが、
>>84>>86で解説をいただきましたが、わからないところがまだあります。
よろしければ、教えていただきたいです。
まず>>76のほうでは、、ベクトル方程式で、2直線が平行というのは、もしくは交わる
というのはどう考えたらよいのでしょうか?

>>77で、解説のとうりにやってみましたが、『α+β、αβ をx,yで表す。』
というのができません。

よろしくおねがいします。
93132人目の素数さん:02/06/16 22:07
おねがいします。。
94132人目の素数さん:02/06/16 22:25
>93
アンタ誰?
95132人目の素数さん:02/06/16 22:48
>>94
88
>92
ベクトルの平行条件はわかってる?
a↑とb↑が平行⇔適当なスカラkによってa↑=kb↑ (零ベクトルでなければだけど)

さて直線A(x.y.z)=(-1-t.1+2t.-1+3t) の方向ベクトル(傾きみたいなもの)は
tの係数をとって(−1,2,3)
同じようにB,Cについても考えれば平行なのはAとC

平行でないものは交点があるかどうかだが、例えばAとBなら
連立方程式を解けばよい。それはやったんだろ?

領域の問題は
x=(1−α)(1−β)(−1)+αβ
y=(1−α)(1−β)(−2)−2αβ
展開してα+β,αβで整理すればできると思うが?
97132人目の素数さん:02/06/16 23:10
質問です。
京大の問題が解けるのに、
香川大とか岡山大とかの問題がとけなかったりします。
どうしてですか?
京大の問題が解ければ香川大とか岡山大とかの問題が解けるだろうという根拠は?
99132人目の素数さん:02/06/16 23:17
だって京大とか阪大とかの方がはるかにむずかしいんじゃ・・ないんですか?
100高校π年生:02/06/16 23:27
0<a<2においてa√(-3a^2+6)^3
の最大値を求めたいのですが、まともに求めると次数が大きくなりうまくできません。
何かいい方法はないでしょうか。数学3Cまでの教養でお願いします。

>100
二乗して
x=a^2とおくと

-3^3 x(x-2)^3

0<x<4において、 x(x-2)^3の最小値を求めればよい。
るーとの中身が負になるんだが
>100
問題はあってる?定義域は0<=a<=√2になると思うんだよね。
このままやれば
√{a^2(−3a^2+6)^3}=√{−3a^(8/3)+6a^(2/3)}^3
√も3乗も単調増加だから{ }の中だけの増減を調べればわかる。
104高校π年生:02/06/16 23:46
すいません、0<A<2√3でした
√3が抜けてました。ほんとごめんなさい。
>99
点数に差がつけばいいのだから問題ごとの難易度は関係ない。
京大でやさしい問題を出すと差がつかなくなる?・・・かどうかは知らない。
>104
範囲を広くしてどうする…
>104
おいおい、2よりも大きくなるのか。それなら式の方は大丈夫か?
>>100はネタの香りがプンプン。
>99
マジレスしてみる
京大といっても,文系の問題は理系ほど難しくない
後,ほとんどの大学の入試問題は,1問くらいはやたら簡単なのが,1問くらいはやたら難しいのが入ってる.
いかに簡単な問題を確実に解くか,いかに難しい問題をこれは難しいと見抜いて捨てれるか,も重要なファクターだ.

だから問題集に載ってる(〜大)ってのは気にしない方がいい
110高校π年生:02/06/17 00:06
すいません。また間違えました。
問題の式はa√(36-3a^2)^3でした。0<a<2√3です。
√{36a^(2/3)−3a^(8/3)}^3
{ }の中の増減を調べよ。指数が分数になっても計算できるだろう。
112高校π年生:02/06/17 00:20
>111
バッチリです。ありがとうございました。
113132人目の素数さん:02/06/17 00:26
>>110
0<a<2√3
I=a√(36-3a^2)^3
3I^2=3a^2(36-3a^2)^3=A(36-A)^3 ←3a^2=Aとした

0<a<2√3より
0<A<36,0<(36-A)<36

相加相乗平均より
3A+(36-A)+(36-A)+(36-A)≧4*[3A(36-A)^3]^(1/4)
108≧4*(9I^2)^(1/4)
27^4≧9I^2
3^5≧I

うそくさい?
114132人目の素数さん:02/06/17 00:33
計算ミスったかも
I≦3^4かな?
115132人目の素数さん:02/06/17 00:35
やっぱ3^5でよさげ

等号条件は
3A=(36-A)
A=9
a=√3
116高校π年生:02/06/17 00:45
>>113->>115
全く思い浮かびませんでした。
さんくすです。
117132人目の素数さん:02/06/17 00:58
x^2+y^2+z^2=4 のとき、
-x^2+y^2+2*z^2の最大・最小を求める事は可能でしょうか?

元々は、
A=[[0 1 0],[1 0 0],[0 0 2]]
p=[[x],[y],[z]]       で、

p'*p=4のとき、p'*A*pの「最大・最小を求めろ」という二次形式の問題なのですが…。
>117
>-x^2+y^2+2*z^2
この式はどっから出てきた?特にマイナスとかさぁ
>118
対角化したんじゃねえの?
120132人目の素数さん:02/06/17 05:23
>>117
-x^2 - y^2 - z^2 ≦ -x^2 + y^2 + 2*z^2 ≦ 2*x^2 + 2*y^2 + 2*z^2
121危(゚Д゚)機⇒ベクトル :02/06/17 06:33
>>96さん
どうもありがとうございました。
頑張りますね。
122132人目の素数さん:02/06/17 08:58
実数x,yについてx^2=y^2であることはx^3=y^3であるための(   )である。
(1)必要十分条件
(2)必要条件
(3)十分条件
(4)必要条件でも十分条件でもない

解答を一部写します
「x^2=y^2 
x^2 - y^2=0
(x-y)(x+y)=0
x-y=0, x+y=0

x^3=y^3
x^3 - y^3=0
(x-y)(x^2 + xy +y^2)=0・・・・」

ここでx-y=0だけどx^2 + xy +y^2=0の式からはxの2次方程式と見て
判別式をとると、虚数解しかでてこないのではないのでしょうか?
だから、答えは(3)だと思ったのですが、解答は(2)でした。
なぜこうなるのでしょうか?よろしくお願いします。
123132人目の素数さん:02/06/17 08:59
整数m,nについてmとnが共に5の倍数であることは、m+n, mnが共に
5の倍数であるための(     )である。
(1)必要十分条件
(2)必要条件
(3)十分条件
(4)必要条件でも十分条件でもない

m+n, mnが共に5の倍数というのをm+n=5p, mn=5qとおいて、
t^2 - 5p + 5q=0の2次方程式を立ててといたら、答えが間違ってました。
なぜこの発想はだめなのですか?
124教えてください:02/06/17 09:16
次の逆行列を求めよ。
/        \
| 0  −En |
\ En  0  /
Enのとき方が分かりません。複素数を使うところまで習いました。お願いします。
/        \
|  0  +En  |
\ -En  0  / じゃ駄目?

>>123
2次方程式が整数の解を持つという条件を組み入れていないからだ
と思う。
>>123
命題:t^2 - 5pt + 5q=0が自然数の解を持つならば、解は5の倍数である。
証明:
t^2=5(pt-q)でtは整数だから、t^2は5の倍数
5は素数だからtも5の倍数//
128127:02/06/17 09:33
自然数−>整数
に直して。スマソ
129132人目の素数さん:02/06/17 09:40
>>127
解の公式使ったんだけど使っちゃいけないの?
整数でも使えますよね?あと、t^2=5(pt-q)は
右辺にもtが入ってますけどなんで良いのですか?
>>122
実数x,yについて、x^3=y^3であることは x=yと同値。
それは、あなたも気付いているようにx^2 + xy +y^2=0は虚数でしか
なりたたないから。
従って、x^3=y^3 ならば x^2=y^2 が成立。

逆はダメ。だから必要条件。
>>129
解の公式を使って悪いとは言っていないが、私にはそれを使うと、解が
整数になるという条件を組み込んで議論しにくいように思えたから。
要するに、m+n=5p,mn=5qの時(p,qは整数)
t^2-5pt+5q=0という方程式が整数の解を持ってしかも、それが5の倍数に
なれば、必要十分であることが言えるわけだけど、少なくとも整数の解を
持っていれば、それらは5の倍数でなければならないと言ったまで。
で、実は問題に「整数m,nについて」という言葉があるので
答えは必要十分で良いと思う。

整数m,nとしてm+n,mnがともに5の倍数なら、それらは2次方程式
t^2-5pt+5q=0を満たす。(但しm+n=5p mn=5q)
t^2=5pt-5qという方程式もみたすので、>>127で述べたように結局
tは5の倍数。つまり、m,nはともに5の倍数ということになる。
>>123 >>129
t^2=5(pt-q) は右辺を見ると、5で割り切れる。
だから左辺も5で割り切れる、といっている。
右辺にもtが入っていようがなかろうがそれは正しい。

そこで改めて左辺を見ると t^2 だからこれが5の倍数。
従ってt、すなはちこの方程式の整数解は5の倍数。

でもこんなものを持ち出すより、mnが 5の倍数であるなら
m、nのどっちかは5の倍数であるって考えたほうが自然。

133130=132:02/06/17 10:24
>>131
かぶった。
134132人目の素数さん:02/06/17 10:34
>>130
そうですね。すみません、矢印の方向を勘違いしておりました。
むりやりxの2次方程式と見て判別式をとるところはOKなのでしょうか?

>>131
解の公式を使うとt=5±√(25p^2 - 20q) (p,qは整数)となりますよね?
ここからt=5の倍数を導くのは無理ですか?特殊な変形をしないとだめな
んですかね?

>>132
ありがとうございます。右辺のtは関係ないんですね。
そうですね、「m、nのどっちかは5の倍数である」
からやるとだいぶ簡単にとけますね。
1/100の確率で不良品が存在する母集団より無作為に抽出を行った時、
抽出した物の中に不良品が99%の確率で
あるためには何個抽出すればいいのでしょうか?
136132人目の素数さん:02/06/17 10:51
「等差数列a[n]=3 + (n-1)・4/3について数列{a[n]}の項で100より
小さい整数になる項のすべての和を求めよ。」という問題です。
n-1=3mとおきかえたはいいのですが後がわかりません。
137132人目の素数さん:02/06/17 11:23
「x + ln(x) = 3をニュートン法を用いて解きなさい。ただし初期値は1とし
小数点以下3桁の精度で求めよ」という問題なのですがいまいち
わからないのです。
プログラムは使ってはいけないので教えてもらえれば幸いです。
よろしくお願いします。
138132人目の素数さん:02/06/17 11:27
すみません。この問題を誰か教えてください。


Oを原点とする複素数平面上に2点A(α)、B(β)がある。
△OABが角度AOBの大きさが45°である直角二等辺三角形で
あるときα/β=a+biを満たすa,bの値をもとめよ。

とりあえず場合わけをして角度OABが90度のとき
OBAが90度のときってわけるまではなんとなくわかるのですが・・・。
139132人目の素数さん:02/06/17 11:32
>136
とりあえず、nに数字を入れて、項が100以下になるnを探そう。(3+(n-1)4/3<100これを計算する)
すると、nの値がでてくるから
あとは合計がでる等差数列の公式に入れれば答えがでてくるよ。
公式わすれちゃったけど
140117:02/06/17 12:02
>>118
>>119さんのおっしゃる通り、対角化して出てきた行列です。

>>120
ありがとうございました。
>>135 1/100 の当り確率のくじを n回ひいたとき、1回以上あたる
確率を求める。これは、n回全部外れる確率を求め、1から引けばいい。
1-(99/100)^n だね。これが 0.99 以上となる nを求める (あるいは
(99/100)^n < 0.01 となるn を求める)。関数電卓があるなら、対数
を計算しなさい。
142130:02/06/17 12:50
>>134
正確に言うと
x^2 + xy +y^2 = 0 を2次方程式と思ったときの判別式は -3y^2.
これが実数解をもつのはy=0 のときのみで、その時の解はx=0(2重解)。
これはx-y=0 の時に含まれる。

従って x^3=y^3 と同値な条件は
x-y=0 または x^2 + xy +y^2 = 0
すなはち x-y=0 または x=y=0
よってx-y=0と同値。

x^2 + xy +y^2 = (x + (y/2))^2 + (3/4)y^2 =0 と考えると
この実数解はx=y=0のみであることがすぐにわかります。
143132人目の素数さん:02/06/17 12:56
サイコロを5回振って、n回目に出た目を
i(n)とすると(i=1,2・・・,6)
i(1)≦i(2)≦i(3)≧i(4)≧i(5)  となる確率を求めよ。
早急にお願いします。
144132人目の素数さん:02/06/17 12:57
こんな問題出されました、スレ違いですが(?)オナガイスマス

100点満点のテストで3人が取った点数は各々何点でしょう?
・C男は、F太とM美が間違えた問題を全て正解。
・F太は、C男とM美が間違えた問題を全て正解。
・M美は、C男とF太が間違えた問題を全て正解。
・最高点と最低点の差は4点。
・C男はM美より3点高かった。
以上

ヒントだけでもいいっす!よろしく〜〜〜
ニュートン法は、f(x) = 0という方程式の解を求めるとき、解
の予想(初期値)をしてそれを X1 と置き、あとは、
Xn+1 = Xn - f(x)/f'(x) で改良していく方法。>>137
問題なら f(x) = x + ln(x) - 3, f'(x) = 1 + 1/x
で実行すればよい。正しい解は x = 2.207940031569322998581。
それに対して、上記を X=1.0で始めれば、3回目で 2.2079392
に達する。4回やれば 10桁以上合う。
>>144
題意が取りにくいんだが、
・全員の点数を足したら200点
・全員の点数は整数
ってことでC=69,F=65,M=66
でいいのか?
>>143 早急にというから、コンピュータで求めてやったぞ。
確率は 812/7776 だ。
148144:02/06/17 13:23
>146さん
ありがとうございました。
私は題意がとりにくいばかりか全く答えの導き方がわかりません。
もしよかったらどうしてそうなるのか、教えてくださいませんか?
149144:02/06/17 13:35
この問題を解かないと罰ゲームがまっています。この答えで免れるでしょう。
ほっとしています。でもどうしてこんなにすぐに解けてしまうのか??
少しだけでも教えてくれたら嬉しいんだけどな。
150135:02/06/17 13:59
>141さん
あーそうか!!

助かりました、本当にありがとうございました。
151132人目の素数さん:02/06/17 14:31
http://www.sansu-olympic.gr.jp/class1/hint_q5.html

の11分がわかりません
152134:02/06/17 15:38
>>142
お返事どうもありがとうございます!!
大変わかりやすかったです。ところどころ勘違いしていたところも修正できました。
判別式からとグラフから得られるんですね。グラフのほうは視覚的でわかりやすいです。
助かりました。感謝!
153134:02/06/17 15:44
>>131
解の公式を使うとt=5±√(25p^2 - 20q) (p,qは整数)となりますよね?
ここからt=5の倍数を導くのは無理ですか?やはり特殊な変形をしないと
だめなんですかね?整数問題は女神のなんちゃらって聞いたことあります
けど。
154132人目の素数さん:02/06/17 15:52
誰か138の問題を教えてください・・・・。
155132人目の素数さん :02/06/17 15:58
>>154
一つの書くが45度の直角2等辺三角形だから、比が1:1:√2になるよね。
だから、君が言うようにどこが90度なのか場合分けして、
Aを(Bを)45度回転と1/√2に収縮した複素数がBに一致(Aに一致)
することを立式化すればいい。
156132人目の素数さん:02/06/17 16:38
zの周りの小さな領域 N(z;e) = {z|z-x<e}....

こんな問題があったときの、記号;の解釈を教えてください。
157132人目の素数さん:02/06/17 17:18
>148-149

100個の点からなる集合Xを考え、問題の配点によってこの集合を分割する。

例えば、問題1が20点、問題2が30点、問題3が50点、という配点だったとしたら、
まず20個の点を取ってきて、それらの点からなる集合を集合1とし、
次に、集合1に含まれない点から30個の点を取ってきて集合2とし、
残りの点すべて(50個)からなる集合を集合3とする。

そうするとちょうど問題1→集合1、問題2→集合2、問題3→集合3、
といような対応ができ、各集合に含まれる点の個数が対応する問題の点数に相当する。
※もしも問題がもっと細かく分かれている場合には、集合ももっと細分化して考える。

このとき、
C男が正解した問題に対応する集合すべての和集合を集合C、
F太が正解した問題に対応する集合すべての和集合を集合F、
M美が正解した問題に対応する集合すべての和集合を集合M、
と置くと、条件より、
(1-i)Mの補集合とFの補集合の和集合が、Cに等しい。
(1-ii)Cの補集合とMの補集合の和集合が、Fに等しい。
(1-iii)Cの補集合とFの補集合の和集合が、Mに等しい。
(2)C,F,Mのそれぞれに含まれる点の個数について、最多のものと最少のものの差は4個。
(3)Cの点の個数はMの点の個数より3個多い。
ということがわかる。

図を書いてみればわかるが、(1-i)(1-ii)(1-iii)より、
Cに含まれる点の個数、Fに含まれる点の個数、Mに含まれる点の個数、の合計は、
全体集合の点の個数の2倍(すなわち200個)に達する。
ここで、これら3つの集合のうち点の個数最少のものの点の数をXとすると、(2)より、
点の個数最多の点の個数はX+4、残る1つの集合の点の個数はX+n(0≦n≦4)になり、
X+(X+4)+(X+n)=3X+(4+n)=200
という式が成り立つ。(3)よりn=1またはn=3であるが、
n=3ならば3X=193となり、不適(右辺が3で割り切れない)。
よってn=1であり、このとき3X=195よりX=65が導かれる。

したがって3つの集合それぞれに含まれる点の個数は、65個、66個、69個であるが、
ここで再び(3)を考えると、Cが69個、Fが65個、Mが66個であることがわかる。
この各個数が3人の得点に等しいから、C男は69点、F太は65点、M美は66点となる。
158132人目の素数さん:02/06/17 17:26
【問】点Aを原点Oを中心としてθ(0°≦θ≦360°)だけ回転した点を
   A´とする。A(p,q)のとき、(ベクトルOA´)をp,q,θを使って表せ。
 
【答】(ベクトルOA´)=( p・cosθ−q・sinθ , p・sinθ+q・cosθ )
 
何故こうなるのかわかりません。これはどう見ても加法定理を使ってると
思うんですが、それだと|OA(ベクトル)|が1でなくてはならないと思うのですが…
教えてください。
159132人目の素数さん:02/06/17 17:43
「等差数列a[n]=3 + (n-1)・4/3について数列{a[n]}の項で100より小さい
整数になる項のすべての和を求めよ。」という問題です。
n-1=3mとおきかえたはいいのですが後がわかりません。
nの範囲は1≦n≦73になりました。
160132人目の素数さん:02/06/17 17:44
>>138 >>154
さしあたり OA > OB の場合だけやってみる。OB を長さ √2倍して、
角度を 45度まわすには、γ = √2(cos 45°+ i sin 45°) = 1 + i
という複素数をかける。-45°なら 1 - iだ。α = (1±i)β だから、
答は自明だろう。
161160:02/06/17 17:46
ごめん、>>155 にすでに教育的回答があった…。
162132人目の素数さん:02/06/17 17:53
32/3・(1 - 1/2^n)が整数になるn(整数)を求めるのがわかりません。
どうすればよいのでしょうか?
>>159 別に n-1 = 3m などという置き換えは必要ない。
1≦n≦73 の nの総和 馬 から直接答を導け。
>>162 答えが n=2 ないし n=4 以外にはないことを、どう示すかが
カギだ。
>>158 複素数の乗算で証明するのがいちばん簡単と思うが、
今の工房はその知識があるんだろうか?
166132人目の素数さん:02/06/17 18:03
【問】正三角形ABCの内部の点PがAP=10,BP=8,CP=6を満たしているとき
   この正三角形の一辺の長さを求めなさい。

わかりません。皆様の知恵をお貸し下さい。お願いします。
167132人目の素数さん:02/06/17 18:08
>>165
複素数の知識があまり無いので、他の方法があれば
それでお願いします。
168132人目の素数さん:02/06/17 18:16
>>158

丸作ってやりなよ。たぶんいける。
170132人目の素数さん:02/06/17 18:21
>>163
そんなことできるのですか?考えてみたんですが、ややこしくて
頭が動きません。nがとびとびになりますよね・・・。
す、すみません、解答教えてください。
171132人目の素数さん:02/06/17 18:23
>>158
|OA|^2=(p^2+q^2)でしょ?
>>166 一辺は (2/181)(209 + 54√3)√((2581+336√3)/193)
= 13.5329... という、とんでもない値になるみたいなんだが。
173132人目の素数さん:02/06/17 18:24
>>169
ありがとうございます。
174132人目の素数さん:02/06/17 18:29
>>172 私は2√(25 + 12√3)になりました・・・(汗)
175172:02/06/17 18:32
>>174 お、数値的には合います。私の簡略化不足。同じ答みたい。
176132人目の素数さん:02/06/17 18:36
>>168
何度もやったんですが…わかりませんでした。
>>171
あの問題が、証明問題だったらその条件でクリアーできるんですが
そうじゃなくて、1からあの答にたどり着きたいんです。
177:02/06/17 18:36
ドリーですよー。
(__)
(oo)
/-------\/ http://www.KissLoveTOCO.com
/ | ||
* ||----||
^^
178163:02/06/17 18:40
>>159 >>170
求める級数は a[n] = 3 + (4/3)(n-1) = (4/3)n + 5/3 だろ?
この形から出発するんだ。(4/3)n を 1<=n<=73 まで総和をとった
ものは、単に 1<=n<=73 までの総和を 4/3倍したものだろ?
5/3 を73個足したものは、73×(5/3)だろ?
179132人目の素数さん:02/06/17 18:49
>>178
はい!いってることはわかりました。
それでそこからどうすればよいのでしょうか?
>>176 世話のやけるやっちゃなあ。
たとえば cos(x+y) = cos(x)cos(y)-sin(x)sin(y) というような
式は、既知としていいのか?
もし良いのなら、(p,q) = (r cos(φ), r sin(φ)) としておけば、
これを θ 回した座標は (r cos(φ+θ), r sin(φ+θ))。
ここで、x座標に関して展開してみると、
r cos(φ+θ) = r(cos(φ)cos(θ)-sin(φ)sin(θ)) = p cos(θ) - q sin(θ).
181132人目の素数さん:02/06/17 18:52
加法定理を使って計算すれば
(ベクトルOA´)=( 1/( p² + q² ))× (p・cosθ−q・sinθ , p・sinθ+q・cosθ))
 
になってしまいます。
182163:02/06/17 18:54
>>179 それ以上は知らないよ。まあ、1から73の総和をとってみれ。
183132人目の素数さん:02/06/17 18:56
>>180
やっと納得する答が得られました。本当にありがとうございました。
184132人目の素数さん:02/06/17 18:57
>>182
そ、そんなぁ〜a[n]は整数じゃなきゃいけないのに・・・
元にもどっちゃったじゃないですか〜
うわぁーーん
185163:02/06/17 19:04
>>184 あ、ごめんごめん。「100より小さい数」じゃなくて、「100より小さい
整数」か。それで 3m = n-1 にこだわってたんだな。
倍n=1,73}(3+(4/3)(n-1)) = 倍m=0,24}(3+4m) でやってごらん。
>184
等差数列を3つ置きにとればやっぱり等差数列だよね。
そういう問題じゃ無いのかい?
187163:02/06/17 19:06

誤 倍n=1,73}(3+(4/3)(n-1))
正 倍n=1,73, ただしa[n]は整数}(3+(4/3)(n-1))
188132人目の素数さん:02/06/17 19:14
>>181
何か勘違いが抜けないようだね。
|OA|=|OA´|=√(p^2+q^2)にならなきゃおかしい。
点A(p,q)なのになぜ|OA|=1になる?
189132人目の素数さん:02/06/17 19:20
ん?発想は逆なのか。
答えから|OA´|を計算したら1になってしまい
ならば|OA|も1になるはずなのに変だ、ということか。
190132人目の素数さん:02/06/17 19:20
>>188
本当に馬鹿を反省します。全くの勘違いでした。。
ミスも修正しました。
191132人目の素数さん:02/06/17 19:25
>>190
ん、がんがれ。
192お願いします:02/06/17 19:27
一辺の長さが10cmの正方形ABCDにおいて、
この正方形に内接する半径5cmの円をR、
点Aを中心とする半径10cmの円をQとした場合に、
円Qと円Rで囲まれた三日月の部分の面積を教えてください。
パイを使って表すそうです。
東大卒2人が挑戦しましたが、解けませんでした。
どうしても解き方が知りたいです。
高校入試に出たそうなので、中学校の知識で解けるそうです。−イ
よろしくお願いします。
上記、イの部分は伝聞なので、
ひょっとしたら、出題ミスなどの理由で、
実は解けない問題なのかもしれません。
よろしくお願いします。
193132人目の素数さん:02/06/17 19:35
>>192
がいしゅつ。答えに逆三角関数が含まれ、外せないことが証明済み。
194132人目の素数さん:02/06/17 19:35
>>192

>高校入試に出たそうなので、中学校の知識で解けるそうです。−イ
無理です。
195132人目の素数さん:02/06/17 19:40
196132人目の素数さん:02/06/17 19:46
3匹釣られたか。
ある意味今井級
198132人目の素数さん:02/06/17 19:49
>>185
倍n=1,73}(3+(4/3)(n-1)) = 倍m=0,24}(3+4m)
答えは1275になりました。
どうして倍m=0,24}(3+4m)になるんですか?
a[n]=(3+(4/3)(n-1) (1≦n≦73)の整数項は
a[m]=3+4m(0≦m≦24)でいいのでしょうか?
a[3m+1]=3+4m(0≦m≦24)ですか?
置き換えた数列はどうなるのでしょうか?

>>186
わたしもそう思って、n=3m+1で置き換えて新しく等比数列を作ろうと
おもったんですけど、うまくいかないんです。手が動かないというか・・
199132人目の素数さん:02/06/17 19:51
「nが1より大きい整数のとき √1+√2+√3+・・・+√n が無理数に
なる事を示せ」
という問題の糸口が掴めません
誰かヒント下さい
200132人目の素数さん:02/06/17 19:53
>199
糸口
任意の自然数kに対して、
k以上2k以下の素数pが存在する
201132人目の素数さん:02/06/17 19:54
>>198
正確な問題文は?>>159
a[n]はa[1]から始まるのようなことが書いてあれば
足し始める最初の項に迷うことはないはずだが。
202132人目の素数さん:02/06/17 19:55
32/3・(1 - 1/2^n)が整数になるn(整数)を求めるのがわかりません。
どうすればよいのでしょうか?

前にお返事いただいたのですが、ちょっとわかりませんでした。
もうちょっと教えてください。
203132人目の素数さん:02/06/17 20:01
>>201
n=1,2,3,4,5,6,・・・・・・・です
足し始める最初の項はa[1]ですね。
n=3m+1で置き換えた解答はどうなるのでしょうか。
nで考えるとどうしても頭がついていきません・・・うぅ。
204163:02/06/17 20:02
>>198 答は 1275 で正解だ。a[m]=3+4m(0≦m≦24) でいいのだ。
よく考えてごらん。
>>202 さしあたり、1-1/2^n = (2^n - 1)/2^n の分子が 3で
割り切れる必要がある。2^n は nが偶数のとき、3で割ると 1余り、
奇数のとき、3で割ると 2余る。これくらいのヒントでどう?
206132人目の素数さん:02/06/17 20:11
>>202
nが4以下の偶数のとき(負数でもよい)
207198:02/06/17 20:14
>>204
a[n]=3+(4/3)(n-1) (1≦n≦73)をn=3m+1で置き換えると、
a[n]の[n]のところも置き換わるから、
a[3m+1]=3+4m(0≦m≦24)になるのではないでしょうか?
でもa[3m+1]だと、数列の式っぽくないような気がする・・・


>198
元の数列が初項3,公差4/3の等差数列なら
3つおきにとれば初項3,公差4の等差数列。これならできるかな。
>207
aがいやならbmって置けばいいじゃん。
210137:02/06/17 20:30
>>145
遅くなりましたがありがとうございます。
早速やってみたいと思います。
211132人目の素数さん :02/06/17 20:37
>nが偶数のとき、3で割ると 1余り、
奇数のとき、3で割ると 2余る。これくらいのヒントでどう?

なんでそういえるのですか?もう少しヒントきぼんぬ
>202
2^nが32の約数になる。n=1,2,3,4,5を調べる。
nは0以下の整数でもいいの?じゃあ>205を考えて、>206で
終わりじゃないか。
>211
2^(2k)=4^k=(3+1)^k≡1
2^(2k+1)=(4^k)*2={(3+1)^k}*2≡2(mod3)
214132人目の素数さん:02/06/17 20:47
解き方がさっぱりわかりません。
解き方の流れの説明、誰かお願いします。


1) A(−1、−1) B(5、1)と第1象限にある点Cとで
 できる△ABCが正三角形となるときのCの座標?
2) △ABCの辺BC、CA、ABの中点がそれぞれD(1、3)
 E(2、−1) F(0、1)のときのA、B、Cの座標?
215ドキュソ:02/06/17 20:48
円Oに内接する四角形ABCDがあり、AB=4,AD=5,cos∠BAD=-1/5である。
また、対角線ACとBDは点Eで垂直に交わるとする。
このとき、BD=(ア)である。また、ΔABDの面積は(イ)√(ウ)であり、
AE=(エ)√(オ)/(カ)である。
さらに、円Oの半径は(キク)√(ケ)/(コサ)であり、BC=(シス)√(セ)/(ソタ)、
四角形ABCDの面積は(チツテ)√(ト)/(ナニ)である。


という問題がわかりません・・・。
AEはどうやって求めるのでしょうか?それがわからないので
それから後に進めません(>_<)
親切な方、教えてもらえると嬉しいですm(__)m
>214
1)C(x,y)と置いてAC=BC=AB
2)A(x1,y1),B(x2,y2),C(x3,y3)のように置いて中点の公式
217132人目の素数さん:02/06/17 20:55
>>212
なるほど!やっとわかりました。ありがとうございます。

>>213
記号の意味はわかるんですが、ちょっと難しくてわからなかったです。
218数学しろうと:02/06/17 21:03
たぶんFAQだとはおもいますが、W杯決勝リーグので1,2,3位を予想する
トトカルチョしたときの確率って何分の一になるのでしょうか??
16チームの確率は16*15*14・・・・(16!って書くんでしたっけ??)
だと思うんですが、トーナメントだと隣り合ったチームは1位、2位となることは
ありえないと思うので。。。。
219218:02/06/17 21:04
間違えた。
16チーム総当りだと16*15*14=3360通り?
220207:02/06/17 21:10
>>208
答えは出せるのですが、数学の解答っぽくない・・。

>>209
あるほど!わかりました。a[m]ってかくとnのところにmを代入するだけに
なるから、nがmにかわっただけで、意味がないですよね?
新しくb[m]にしなきゃいけないんですね。
221214:02/06/17 21:10
1) C(x、y)とおく。AB^2=BC^2=CA^2なので、
 AB^2=(5+1)^2+(1+1)^2=40
 BC^2=(x−5)^2+(y−1)^2=40
 CA^2=(x+1)^2+(y+1)^2=40

ここでつまってしまいました。BC^2=CA^2で連立し、計算しても
 3x+y=6という式でとまってしまいます。
どうすればいいでしょうか?


2) できました。ありがとうございました。
222132人目の素数さん:02/06/17 21:13
>>218
4^3*4!=1536
自信なし
223192:02/06/17 21:25
>>193
>>194
>>195
の皆さま、
情報を教えていただき、ありがとうございました。
私は、掲示板書き込みが初めてでした。
既出であることを確認せずに投稿してしまってすみませんでした。
どうもありがとうございました。
224132人目の素数さん:02/06/17 21:25
x^2+xy-2x-2y+1って因数分解できるでしょうか?
もし、できるのだったらやり方を教えて頂きたいのですが。
225132人目の素数さん:02/06/17 21:30
>>218
┏┻┓┏┻┓┏┻┓┏┻┓┏┻┓┏┻┓┏┻┓┏┻┓
.←  A  → ←  B  → ←  C  → ←  D  →

まずA〜Dの4つのブロックのうち、3位以内に残らないと思う1つを選ぶ・・・4通り
各ブロック進出チームの1位2位3位を決める・・・3*2*1=6通り
選んだ各ブロックから勝ち残るチームを決める・・・4*4*4=64通り

合計4*6*64=1536通り
226214:02/06/17 21:34
>>224 できません。
ところで、そのような形の因数分解の方法をご存知ですか?

ex) x^2+3xy+2y^2+x−2
 =x^2+(3xy+x)+(2y^2−2)    ←x^2、x、xのない項同士でまとめる
 =x^2+(3y+1)x+2(y+1)(y−1)
 ={x+2(y+1)}{x+(y−1)}
 =(x+2y+2)(x+y−1)
227224:02/06/17 21:40
>>226
ありがとうございました。
気になっていたので、すっきりしました。
あと、例題まで書いて頂いてありがとうございます。
228数学しろうと:02/06/17 21:47
>>222 >>225
すばやい回答ありがとうございます。
図解までしていただき、よくわかりました。
ありがとうございます。
229132人目の素数さん:02/06/17 21:51
(1)a[n+1]=p・a[n] + q
(2)a[n+1]=p・a[n] + f(n)
(1)は両辺にq/(1-p)を引いて等比数列に持ち込むタイプですよね?
(2)はa[n+1] + α(n+1) + β= a[n] + αn + βとおいて、
等比数列に持ち込むタイプですよね?そこで、
(1)のαが1のときは上のように解けないですよね?
また、(2)のαが1のときは同じく上のように解けないですよね?
これはなぜでしょうか?
230132人目の素数さん:02/06/17 21:59
>>229
αってpのこと?
公比が1のときは等比数列の和の公式a(1-r^n)/(1-r)が使えないから。
>221
そこまでできていたら
y=−3x+6 にして2次式に代入すれば良し
232132人目の素数さん:02/06/17 22:06
教えてほしいことがあるんですけど、

二つのベクトルx,x'は、x-x'が原点(0,0,0)とx0を結ぶ直線L上の点であれば…

ていう部分の文の意味がわからないんですけど、
ベクトルが点ってどういう意味ですか?
>232
位置ベクトルなんて言葉は知ってる?
ベクトルと点は、原点を始点として、1対1の対応がつけられるのです。
234132人目の素数さん:02/06/17 22:10
与えられたベクトルの始点を原点に平行移動すれば
ベクトルを表していたものを点(座標)として見ることができる。
235132人目の素数さん:02/06/17 22:11
かぶりスマソ。
236132人目の素数さん:02/06/17 22:12
むしろ点(座標)をベクトルとして見れるというべきか。
237221:02/06/17 22:19
>>231 なるほどできました。ありがとうございます。
238132人目の素数さん:02/06/17 22:26
「(2)はa[n+1] + α(n+1) + β= a[n] + αn + βとおいて、
等比数列に持ち込むタイプですよね?」
は出鱈目
239137:02/06/17 22:35
ニュートン法を教えてもらったものです。
計算したのですがプログラムだと確かに2.2079・・・となるのですが
自分の手計算だとなぜか2.58・・・に収束してしまってます。
X1+1 = 1 - ( 1 + ln 1 - 3 ) / ( 1 + 1/1 )で答えが2
X2+1 = 2 - ( 2 + ln 2 - 3 ) / ( 1 + 1/2 )で答えが2.46になってしまいます。
どっかで間違ってると思うので指摘していただければ幸いです。
240132人目の素数さん:02/06/17 22:55
>225 >228
遅レスでスマンが、Aブロックが1位、Bブロックが2位ってのは
起こらないよ。(準決勝で当たるから)

正しくは 16×4^3=1024 だべ。
241145:02/06/17 22:56
>>239 アンタはどうやら手計算のとき、log(x) に常用対数
(底を 10 にした対数)を使っている。自然対数 (ln(x)) を
使うんだ。
242137:02/06/17 23:01
>>241
あ、ホントだ。
一生懸命logで計算してました(汗
なんとか答えに導けました、ありがとうございます。
243132人目の素数さん:02/06/17 23:17
問) A(2(x1)、2(y1)) B(2(x2)、2(y2)) C(2(x3)、2(y3)) D(2(x4)、2(y4))を
  頂点とする四角形ABCDの辺AB、BC、CD、DAの中点をそれぞれP、Q、R、Sと
  するとき、(AC)^2+(BD)^2=2{(PR)^2+(QS)^2}

これをやっていたのですが、展開、移項、整理、これをやっているうちに
計算量が膨大になってしまい、もっと楽に計算できる方法を考えたのですが
まったく手付かずです。どのようにすれば計算が楽になるか、教えてください。
244229:02/06/17 23:23
>>230
>αってpのこと?
公比が1のときは等比数列の和の公式a(1-r^n)/(1-r)が使えないから。

すみません、pの間違いでした。。。和の公式ってどこででてくるのでしょうか?
総和を求める問題ではなくて、一般項を求める問題なのですが。

>(2)はa[n+1] + α(n+1) + β= a[n] + αn + βとおいて、
等比数列に持ち込むタイプですよね?そこで、

すみません、間違えました、a[n+1] + α(n+1) + β = p{a[n] + αn + β}ですね。
この式のpが1のときは上のように解けないですよね?これはなぜでしょうか?
245225:02/06/18 00:01
>>240
ナイス指摘だ。スマソ
246148・149:02/06/18 00:16
>>157

感激しました。丁寧に書いて下さってありがとうございます。

惚れてしまいそうです。また、よろしくお願いします。

247数学しろうと(自宅):02/06/18 00:19
>>225 >>240
帰りの電車でぽーっと考えてたら、4チームのトーナメントでも隣どうしが
1、2位にはならないことに気づきました。

でも、>>225さんにはブロックで考えるというヒントをいただきました。
>>240 さんもありがとうございました。
>>243
AC↑ = (2α, 2β), BD↑ = (2γ, 2δ) とおいてごらん。
PR↑ = (α+γ,β+δ), QS↑ = (α-γ,β-δ) みたいな式に
なるから (符号逆かな?まあいいや)。
249132人目の素数さん:02/06/18 07:44
>>243
四角形PQRSが平行四辺形であることに注目すると
証明すべき等式は中線定理から直ちに得られる。
・・・っていうのは多分248と同じ事かもね。
250132人目の素数さん:02/06/18 08:22
f1(t)およびf2(t)は同一周期Tを有する周期関数であるとする。
一方、C1n、C2nはそれぞれf1(t),f2(t)の複素フーリエ級数係数である。
この時、次式を示せ。

1/T∫[-T/2,T/2] f1(t)f2(t)dt = Σ[n=-∞,+∞]C1nC2-n

何故こうなるのかわかりません。
よろしくお願いしますm(_ _)m
251250:02/06/18 08:30
↑と同じ式です
 1  T/2             +∞
━━∫ f1(t)・f2(t) dt = ΣC¹nC²-n
 T  -T/2           n=-∞
252132人目の素数さん:02/06/18 09:04
>>250
(見づらいから勝手に記号を変えるけど)
f(t)=Σa(n)exp(2nπi/T)
g(t)=Σb(n)exp(2nπi/T)
のとき、収束とか難しいこと考えずに乱暴に計算すれば
f(t)g(t)
={Σ_n Σ_m}a(n)exp(2nπi/T)b(m)exp(2mπi/T)
={Σ_n Σ_m}a(n)b(m)exp(2(n+m)πi/T)
={Σ_k Σ_n}a(n)b(k-n)exp(2kπi/T)
=Σ_k {Σ_n}{a(n)b(k-n)}exp(2kπi/T)
となるからf(t)g(t)の複素フーリエ級数係数はΣ_n{a(n)b(k-n)}。よって
(1/T)∫_[-T/2,T/2] f(t)g(t)exp(2kπi/T)dt = Σ_n {a(n)b(k-n)}
で、ここでk=0としたのが件の等式ではないかと・・・。

f,gがC^1級(C^2級?)だとa(n),b(n)が1/nのオーダーなので
上の乱暴な計算が許されるような気がする。
ぜんぶ嘘かもしれないんで信用しないように(w
253250:02/06/18 09:10
>>252
おぉ!どうもありがとうございます。
考えてみます。
254132人目の素数さん:02/06/18 09:57
関数解析の本を読んでいて、以下のところで引っかかってしまいました。
分かる方、教えてくださいませ。

急減少C^∞級関数全体の空間S(Schwartz空間?)と、
C^∞_0 :={u∈C^∞ ; suppがコンパクト}
は同じもののような気がするのですが、違うのでしょうか?

あと、コンパクト、は色々なところに出てきていて、いまいちその
概念が掴みきれていないのですが、ここでは、有限可算個の閉集合で
supportが囲みこめる、ということですか?


255132人目の素数さん:02/06/18 10:20
>>254
例えば空間(定義域)が1次元ユークリッド空間(数直線)ならば、コンパクト部分集合
とは有界閉集合のこと。つまり、コンパクト台とは関数が0で*ない*区間がある
閉区間[a,b]に含まれること。

これで違いが分かった?

コンパクトの本当の定義はそれで良いけどその定義を普通の空間に適用したとき
どういう意味を持っているか知らないのは少し問題。どんな本で位相を勉強したの?
256255:02/06/18 10:24
>>254
訂正:よく見たらコンパクトの定義が違うよ。もう一度教科書の定義を見直すべし。
257132人目の素数さん:02/06/18 11:46
よく混ぜたトランプをね、二人で一組ずつ持ってるですよ。
そいで、二人同時にそれをめくっていくんですよ。
すると、なんでだか、同じカードがでてくるですよ。
なんで?

確率を計算しなさい。
(途中の式も書くこと。考え方が合っていれば点数を上げます。)
>>250 の問題、単なる計算問題としてしまうにはもったいないの
で、物理的解釈を加えておく。
1/T∫[-T/2,T/2] f(t)g(t)dt = Σ[n=-∞,+∞]Fn G-n ... (A)
の左辺はf(t)とg(t)が似た変化をする関数ほど大きな値になる
ため、相関関数という。実関数なら G-n = Gn* (* は複素共役)
だから、通常右辺は Σ[n=-∞,+∞]Fn Gn* と書かれる。連続
関数のフーリエ変換では、∫f(t)g(t)dt = ∫F(ω)G(ω)*dω と
いうことである。

興味深いのは g(t) = f(t) ないし g(t) = f(t-τ) の場合で、
これを特に自己相関という。f(t)は信号の電圧波形と仮定すれば
f(t)^2 は電力に相当する量だが、その総和と右辺F(ω)^2 の
総和すなわち周波数領域のエネルギーは相等しいことがわかる。
パーセバルの等式という。

f(t)=g(t) として f(t) = 0 (-T/2<=t<0); 1 (0<=t<T/2)
を (A)で評価してやれば、Σ1/n^2 = π^2/6 というかなり
高度な関係式が導けるはずだ。
>>257 出題の段階で定式化できていない。0点。回答の必要なし。
260250:02/06/18 11:58
>>258
わざわざどうもありがとうございます。
後でじっくり読ませてもらいます。
261257:02/06/18 12:16
>>259
そんなこと仰って、逃げるのですか? 低脳!
この程度の問題なら考え得る全ての事象を挙げて、
その各々について解いてみて欲しいのです。
そのくらいのことできねいのか?
お願いします! これの答えがわからないと困るんですよ、マジで。

オマエラ、なんとかしろ!
262259:02/06/18 12:24
>>261 じゃあ特別に低脳をふりしぼって考えてやらんでもないが、
もっと低脳な出題文をなんとかしろ。「確率を計算しろ」じゃ、
何を計算していいかわからん。

1枚目で一致する確率、2枚目で一致する確率…、を求めろという
ことか?一致するまで、平均何枚めくることになるか、期待値を
求めろということか?それとも、何か別のことか?
>>250
f1(t)=Σ[k]<f1,e(k)>e(k)(t) C1(k)=<f1.e(k)>
f2(t)=Σ[k]<f2,e(k)>e(k)(t) C2(k)=<f2,e(k)>
ここで<f,g>=1/T∫[-T/2,T/2]f(s)g~(s)ds
e(k)(t)=exp(2πkt/T)
(~は複素共役)
これはフーリエ級数そのものを言い換えたもの
e(k)~=e(-k)
<e(k),e(j)>=δ(k,j) δ(k,j):(k=j)の時1,それ以外は0
および
<f.g>=<g,f>~
<αf,g>=α<f.g>
<f+g,h>=<f,h>+<g,h>
であることに注意し
<f1,f2>を考えると
<f1,f2>=Σ[k]<f1,e(k)><f2,e(k)>~<e(k),e(k)>=Σ[k]C1(k){C2(k)}~
(収束性は、f1,f2のフーリエ級数を有限項で打ち切ったものを利用したら
出来ると思います。)
最後の問題はC2(k)の複素共役はC2(-k)かということですがf2が実関数
なら自明ですが、複素数の時は....成り立つでしょうか?
>>257は、久々のナイスキャラの予感。。。
265257:02/06/18 12:46
>>262
有り難うございます!
そんな難しく考えるなよ。
最後までカードをめくっていっても、一致しない場合と、
何枚目かで一致する場合とがありますよね?
その、各々の確率を計算するんだよ! そんぐらい分かれ!
説明が足りなくてスミマセン……
ほれ、さっさと計算を始めろ! ホントにできんのかよ?
よろしくお願いします。
266250:02/06/18 12:48
>>263 きれいな証明、どうもありがとう。相関関数の定義は、
g(t) が複素数の場合もこめて<f,g>=1/T∫[-T/2,T/2]f(s)g~(s)ds
でやるのが正しいと思います。
267266:02/06/18 13:04
いや感謝されるつもりはなく、逆に聞きたかっただけなんですが、f2が実関数じゃ無い時も
成り立つのかな?と
268132人目の素数さん:02/06/18 13:09
本にy=mx+n上の異なる2点A,Bのx座標をそれぞれα,β(α<β)とする。
ABの長さは√(1+m^2) ・(β-α)と書かれてあったのですが、
これはよく使うんですか?
2点間の距離の公式を使えば解決すると思うのですが。
>>268
好きにすれば
270250:02/06/18 13:39
>>267 あ、そういうことなら fが実関数でないと、C-n = Cn*
はなりたちません。一般にf(x) = p(x) + iq(x) で、p(x)
から出る複素フーリエ係数が Pn、q(x) が Qn (Cn = Pn+Qn)
とすれば、C-n = Pn* - Qn* のはず。
271132人目の素数さん:02/06/18 13:40
実関数=実変数実数値関数 ?
272132人目の素数さん:02/06/18 13:48
>>250
複素数の形式でフーリエ級数を扱う時は、実数値関数か、複素数値関数
であるか明記する必要があると思われます。
273254:02/06/18 14:14
>255
Schartz空間Sの関数はsupportがコンパクトとは限らないということですね?
例えばexp(-|x|)はSに含まれるけどC^∞_0には含まれないと。

ついでに、closure(C^∞_0)=L^2にはなりませんか?

あと、位相はまともに一冊の本で勉強したことありません。
距離の拡張みたいなモン、というイメージしかないです…。
274132人目の素数さん:02/06/18 14:56
なりますが、証明にはここの欄にして、30行は必要になります。
275132人目の素数さん:02/06/18 15:01
誰か>>244をお願いします!!
>>257 >>265 トランプの組(52枚とする) を2組、ずらりと並べれば、
特定の場所で2枚のカードが一致する確率は 1/52, 一致しない確率
は 51/52(いずれの位置でも同じ)。その一致・不一致を端から見て
いけば、
最初の位置で一致: 1/52
2番目で一致:(51/52)(1/52) = 51/52^2
3番目で一致:(51/52)(51/52)(1/52) = 51^2/52^3
 :
52番目で一致:(51/52)…(51/52)(1/52) = 51^51/52^52
最後まで一致しない確率は (51/52)^52 = 0.3643 だ。
>>244 >>275 また問題をタイプ分けしたがっているヴぁかがいる。
お前は、出入り禁止のはずだぞ。
278257:02/06/18 15:36
>>276 なるほど! わかりました! 有り難うございます!
一応例は言っとくよ。ところで合ってんのか?
順序立てて考えれば、わりかし簡単な問題ですよね。
このくらいパッと即答できねえのか。パッと。
結局、これを賭としてやる場合は、一致する確率は 1-(51/52)^52=63.6%
だから、一致する方に書けた方が有利というわけですね。
薄々わかってたけどな。
感謝します!

でも俺への尊敬が感じられないから67点な。

また、なんかあったらよろしくネ!
279132人目の素数さん:02/06/18 15:42
>>277
そいつ荒らし?それかコピペ厨?
280277:02/06/18 15:51
>>279 前から、「三次関数の変曲点の定理を東大の入試で使った
ら正解になりますか?」とか、「確率を分数で表したとき分母は
順列ですか組み合わせですか?」とか、わけのわからん質問を
して回答者をパニックに落とし入れるDQN。荒らしとは違い、純粋
に頭がおかしいんだとおもう。

数学(入試)の問題はすべて自分の作り上げつつある、一種の
タイプ分類体系に組み込むことができるはず、という、強い
信念をもっていて、それから外れたアドバイスには決して耳を
かさない。もしかすると石川県にお住まいのIさんのご親戚か
もしれない。
281132人目の素数さん:02/06/18 16:01
>>280
そういうのってわけわからん質問か?
結構初学者にとっては大事なとこだと思うぞ。
282132人目の素数さん:02/06/18 16:18
>>88>>95
これできなくってなんかむかついたから昨日本屋いって数オリの本たちよみしてきた。
そもそも完全平方数もへったくれも
 
 (a^2+b^2)/(ab+1) (a,bは正の整数,a≦b) が整数になる...(*)⇒b=a^3
 
らしい。証明は
 
 S={b|(*)が成立しb=a^3が成立しないaがとれる。}
 
とおいてSが空でないとして背理法をつかうそうだ。
 
しかしこの問題オーストラリアの数論学者20人くらいにやらせてだえもとけんかったが
数オリでは10人くらい正解したそうだ。それって次世代の数学者はすごいってことを
意味してるのか、だから数オリの問題なんてくだらないってことを意味してるのか。。。
>>281
初学者じゃないらしいですよ
>>281
じゃあ、あなたが専用回答者になってくれ
285132人目の素数さん:02/06/18 18:32
>>1の名前は“132人目のともよちゃん”
・タイトルの前後は◆
>>1がローカルルールの変更を申請してくる

前スレ埋まったのでage
>>284
ハゲドウ
>>281は、粘着君のひつこさを、自分で体験しる!
>229
>(1)は両辺にq/(1-p)を引いて等比数列に持ち込むタイプですよね?
>(1)のpが1のときは上のように解けないですよね?

解く前にq/(1-p)はp=1で分母が0になるから。

>(2)はa[n+1] + α(n+1) + β = p{a[n] + αn + β}とおいて、
>(2)のpが1のときは同じく上のように解けないですよね?

p=1なら(2)の変形でa[n+1]-a[n]=f(n)という階差数列を取ればいいので
わざわざ等比級数を考える意味は無い。
288132人目の素数さん:02/06/18 20:00
>277
答えたい奴が答えればいいんでは?
特に今回は230がわけのわからんアホレス入れてるから
>277=230が恥を覆い隠そうとしてるように見えなくもない。

???
こいつ、真性? それとも、ネタなの???
>>277 = >>230と考える根拠って一体・・・)
>289
必死だな(w
291132人目の素数さん:02/06/18 20:15
=かどうかはともかく
230はアホってことで
まとめましょうや

次の問題にGO1
292289:02/06/18 20:17
>>290
おい、お前、俺という漢を、誤解してるんじゃないのか
俺はいつでも真剣勝負、必死に生きてるゼ!

♪全力、全力、いつも全力・・・(分かる人だけ、ついてきて下さい)
>292
逝ってらっさい。
294289:02/06/18 20:25
・・・誰もついてきてくれないと、寂しいじゃないか!
295こむさ ◆Y72KA/Ns :02/06/18 20:29
θ≠kπ (k∈Z) のとき、数列{sin(nθ)}は発散することを証明せよ。

教科書の発散の定義によると
∀K ∃P∈N ∀n≧P ⇒ a(n)>K

なんですけど、-1≦sin(nθ)≦1 なのにどうやってとくのでしょうか?
296132人目の素数さん:02/06/18 20:31
質問です。
8で割ったあまりが1、3、5である奇数は3つの平方数の和でかける。
らしいのですがこれの証明どうするんでしょう?
日本語の本で証明ののってる本しりませんか?
>295
その定義は極限が∞の場合
298132人目の素数さん:02/06/18 20:34
>>295
>θ≠kπ (k∈Z) のとき、数列{sin(nθ)}は発散することを証明せよ。
>教科書の発散の定義によると
このへんは人によって言葉の定義に微妙な差があることがあるのであんまり
こまかいとこにこだわらないほうがいいと思います。たぶんこの問題は
θ≠kπ (k∈Z) のとき、数列{sin(nθ)}は収束しないことを証明せよ。
と解釈するもんだと思います。
おなかがすいたときに食べる数字ってなーんだ?
極座標
xy平面において、原点O(0.0)を中心とする半径1の円C{0}と、
最初にその中心が座標(2.0)の点にあり、C{0}上を反時計回りに滑ることなく
回転移動する半径1の動円Cがある。動円Cの中心をO′とする。
また原点Oを端点とする点A(1.0)を通る半直線OAから反時計回りに見た時の、
半直線OAと原点Oを端点とする半直線OO′が作る∠をθ(0≦θ≦2π)とする。
この時、最初、座標(3.0)の点にあった動円C上に固定された点Pと原点Oとの距離
OPをθを用いてあらわせ。また点Pの軌跡の概形をかけ。

やけにながったらしい問題です。
とりあえず、動円を{θ2}回転させた時に、点Pの位置は、
OからO′まで引いた直線と、円との交点(Oからの距離3の地点)
からθ{2}反時計回りにずれた点になるんじゃないかと考え、
とりあえずθ{2}回転させた時のO′の極座標を求めなければいけないと思ったのですが、
この部分がわかりません。
ヒントおねがいできればありがたいです。
>>299
5.5
↑すこし書き間違いました。
『とりあえず、動円を{θ2}回転させた時に、点Pの位置は、
OからO′まで引いた直線と、円との 交点P′とする(Oからの距離3の地点)
からθ{2}**このθ{2}は円O′P′とOPのなす∠**反時計回りにずれた点になるんじゃないかと考え、 』

↑2箇所修正しました。

この問題を教えてください。

(1) x^3(x-1)^2/(x-2)(x-3)≧0 を解け

(2)不等式 x≧√1-y^2 を満たす点(x、y)の存在範囲を示せ

(3)不等式 √2(x-1)(x^2-2)>2-x-x^2 を解け   

問題量が多くてスイマセン。
どなたかご教授ください、おねがいします。   
304132人目の素数さん:02/06/18 21:04
√{2(x-1)(x^2-2)}か?
305132人目の素数さん:02/06/18 21:05
>>297 
よく見てみたらそうでした。どうもです。

>>298
なるほど。では、背理法を用いて、
θ≠kπ (k∈Z) のとき、数列{sin(nθ)}は収束すると仮定すると
∀ε>0 ∃P∈N ∀n≧P ⇒ |a(n)-α|<ε が成立する
ところが、-1≦sin(nθ)≦1 よりこれは矛盾
こんな感じでいいんですか?なんか違うような気がするけど・・・・。
306132人目の素数さん:02/06/18 21:13
>>305
>ところが、-1≦sin(nθ)≦1 よりこれは矛盾
ここ飛躍ありすぎ。
>299
π(パイ)
>305
-1≦sin(nθ)≦1 だけでは矛盾にならない。すべての値をとるとは限らないから。
309こむさ ◆Y72KA/Ns :02/06/18 21:29
うーん頭でグラフ浮かべたら当然ってことはわかるのに
それを書いて証明するっていうのは難しいものです。
背理法でとくっていう方向性はあっているんでしょうか?
おひまなかたいらしたら
>>300+302をよろしくお願いします。
>310
極座標と言っているけれど、x−y座標でもよさそうに思うけど、どうですか。
円の中心が(2cosθ,2sinθ)でいいよね。
この中心を通ってx軸に平行線を引いたときの角度を考えてはどうでしょう。
角2θが関係してくると思います。
312なる:02/06/18 22:47
微分です。 aは正の実数とする。xy平面のy軸上に点P(0,a)をとる。関数y=x^2/(x^2 +1) のグラフをCとする時C上の点Qにおいて「QにおけるCの接線が直線PQと直交する、ただしQは原点以外に存在する」ようなaの範囲を求めよ。  おねがいします。
>>311さん
なんとか軌跡までたとせりつくことができました。
ジオイド(?)ってやつですよね。
314132人目の素数さん:02/06/18 23:29
>>312
見覚えあるな。最近の過去問か。
316素数:02/06/18 23:37
---------(1モナ)
    -----(2モナ)
         --------(6モナ)
  -----------(4モナ)
      --------(3モナ)
   \************\

→時間軸 ↑個数
一度に使える個数は10モナあったとする

\と\の間に投入できる個数の最大値ってどうやったら求まりますか?
この問題でいう答え1を時間軸での虱潰しで最大値を出す、
(要するに9モナを求める方法っす
以外で求める方法はあるんでしょうか?

出庫と在庫の問題みたいな。
317132人目の素数さん:02/06/18 23:38
>>316
デムパ?
318素数:02/06/18 23:39
ズレた・・・

ここでいうと、最大値13モナっす。
使える個数14個で答え1。
319132人目の素数さん:02/06/18 23:48
>>151 ,できないならできないことを証明してください
320303:02/06/19 00:01
>>304
すいません、それでお願いします
321ばか野郎=1 ◆wncubcDk :02/06/19 00:31
算数、数学の問題においては特に断りがなくとも分母が√の場合有理化するのは常識ですか?
範囲は高校までです。

※質問スレ(隔離)で同じ質問をさせていただいたのですが、他の先生方にもご意見を伺ったほうがいいというアドバイスを受け、
 このスレに書き込ませていただきました。
 他スレとここで重複した質問になってしまったこと、不快な方おりましたらご指摘いただければ次回から気を付けたいと思います。
322ばか野郎=1 ◆wncubcDk :02/06/19 00:32
さらに文がよみにくくなってしまった。。。すみませんです。
>>321 「分母を有理化して表示せよ」と書かれていない限り、気にする
ことはないと思うよ。有理化は、電卓がなくて手計算していた時代の
遺物だろう。たとえば 1/√2 = 1/1.4142 は、この形で計算するのは
たいへんだが、これを√2/2 = 1.4142/2 とすれば暗算でも数値化で
きる。電卓なら、どちらでも同じだけどね。
324132人目の素数さん:02/06/19 00:39
一応しておいた方がいいと思うけどね、俺は
325132人目の素数さん:02/06/19 00:40
わりと可    1/√2
不可    1/(1+√2)
326307:02/06/19 00:56
関数f(x)=X^3-3xのグラフ上の二点(1,f(1)),(a,f(a))とを結ぶ直線の傾きが
x=bにおけるf(x)における微分関数に等しいときbをaであらわせ。ただし、
1<b<aとする。
お願いします。
327ばか野郎=1 ◆wncubcDk :02/06/19 00:58
>>323
レスありがとうございます。自分でもちょっと調べてみたんですけれど、
有理化せよと書いてなくとも模範解答では有理化されてたので疑問に
思いました。でも深く考えないでいいのかな、と思ったり。

>>324
もししなかったら不正解になります?
{x=sin(1/x)=0}は閉集合ですか?開集合ですか?
どちらでもないんですかね・・?
329243:02/06/19 01:00
>>248-249

遅くなりましたが、ありがとうございました。
平行四辺形で見て、中点に着目....なるほど。
さらに・・

整数は開集合ですか・・・?
>>327
採点者による
>>324
そんなにしたくなかったらしなけりゃいい。

オウンリスクでな
333132人目の素数さん:02/06/19 02:01
>>326
f(x)=x^3-3x
f'(x)=3x^2-3
3b^2-3=(a^3-3a+2)/(a-1)=(a^2+a-2)

b^2=(a^2+a+1)/3
b=√{(a^2+a+1)/3}
334あおい:02/06/19 02:57
次の数列の階差数列を初項から第8項まで書け。@1、3、7、13、21、・・A2、5、10、17、26、・・・。誰か教えてくださいーー!!!
335はっけよい:02/06/19 03:07
sin18°の2乗+cos18°の二乗はいくつになりますか?
336あおい:02/06/19 03:09
この問題も、おしえてください。次の等比数列の初項から、第10項までの和を求めよ。@初項3、公比2の等比数列。A初項4、公比3の等比数列。B2、10、50、250、・・・
337132人目の素数さん:02/06/19 03:09
>>334
(1)2,4,6,8,10,12,14,16

(2)3,5,7,9,11,13,15,17
338132人目の素数さん:02/06/19 03:10
>>335
sin18°の2乗+cos18°の二乗 =1
339132人目の素数さん:02/06/19 03:22
>>336
(1)
a(n)=3*2^(n-1)
S(n)=3(1-2^n)/(1-2)=3(2^n-1)
S(10)=3(1024-1)=3069

(2)
a(n)=4*3^(n-1)
S(n)=4*(1-3^n)/(1-3)=2(3^n-1)
S(10)=2(59049-1)=118096

(3)
a(n)=2*5^(n-1)
S(n)=2*(1-5^n)/(1-5)=(1/2)(5^n-1)
S(10)=(1/2)(9765625-1)=4882812
340はっけよい:02/06/19 03:22
>>338
ありがとう
341132人目の素数さん:02/06/19 04:51
関数の同次性ってのは分かるんですけれども、
集合の同次性というのはどういうことなんですか。
本を読んでたら、正の同次錘?(positive homogeneous cone)というのが
出てきて、ちくと困っております。
すげー、ドキュソな質問ですみませんが、教えていただけないでしょうか。
>341
どういう集合で?
343341:02/06/19 05:13
即レスありがとうございます。
どういう集合かといいますと、説明しづらいですが。
公理として「R^n上の部分集合Aは正の同次錘である。」
と、出てきて、Aっていうのはリスクの集合です。
>334,>336ってさ、教科書に書いてあるとおりやればできる問題と違う?
このくらいの宿題は自分でやらなきゃ力にならんぞ。
345132人目の素数さん:02/06/19 05:18
>>344
数学本当に苦手な人はできなかったりする。
得意な人は理解できないだろうけど。
>343
集合の言葉というより
R^nの図形としての錘であろ。
>321
分数の形を「比の値」ととるか「割算」ととるかだろう。
1/√2でも1:√2と思えばこのままでOKだし1÷√2と思えば
分母の有理化をするだろう。
だから三角比の分野では分母の有理化をしないことも多い。
他の分野では有利化するのが常識と思ったほうがよい。
ま、それも誰かが言っていたように採点者や、問題によるけどね。
348341:02/06/19 07:23
レスありがとうございます。
んでも、Aが正の同次錘であるということを用いて
色々証明したりしているんで、数学的な定義・性質を知りたいな
と思って質問しました。ちくと自分で調べてみます。
もし、分からないことがあったら、またよろしくお願いします。
349132人目の素数さん:02/06/19 08:27
>>331
そんなことはない
b/√a において a と b が互いに素のときは減点されるいわれは無い

DQNな教師なら、それでも減点するであろう
351助けて〜:02/06/19 09:53
おねがいします。。次の問題を皆の前で発表しなきゃいけないんですけど。
さっぱりわかりません・・・・。

tを-iとは異なる複素数とし、Z=(1-ti)分の(1+ti)とおく。

|z|=1ならば、tが実数であることを示せ。


ゴメンなさい。分数の書き方が分からないので変な書き方になりましたが・・。
誰か教えて〜。
>>351 2通りの解法を紹介する。
まず、安全確実だけどつまらない方法。t = x + iy として
z = (1+ti)/(1-ti) に代入。|z|=1 から y(x^2+(y+1)^2) = 0
が導ける。それなりに計算はたいへん。

エレガントなのは、|z|^2 = z・z~ = 1 (z~ は zの複素共役)から、
z = 1/z~ で評価する方法。tで書き直せば
(1+ti)/(1-ti) = (1+t~ i)/(1-t~ i) だが、変形すれば t = t~ が
すぐわかる。
>>282
(30^2+112^2)/(30×112+1)=4。

a,b,c=(a^2+b^2)/(ab+1)が整数のとき
cは−5か平方数。
354助けて〜:02/06/19 10:31
>352
ありがとうございます〜〜!!!感謝です★
355352 これでもいいか:02/06/19 10:45
>>351 こんな方法もあった。z = (1+ti)/(1-ti) を逆に tについて
求めれば、t = i(z-1)/(z+1)。|z| = 1 から z = cosθ + i sinθ
として、ここに代入すると、t は実数になる ((z-1)/(z+1) が純虚数
になる) ことが直接導ける。計算は t = x + iy よりは楽だが、そこ
そこ煩雑。
356352 もう一丁:02/06/19 10:55
>>355 の方法で、z = cosθ + i sinθ = exp(iθ) (オイラーの関係式)
を認めてやれば、これを代入する。z = i(1-exp(iθ))/(1+exp(iθ))
だが、分子分母に 1+exp(-iθ)をかけてやれば、zが実数であること
は一発でわかる。計算としては、一番簡単か。
z = (1+ti)/(1-ti) を複素平面 tから zへの写像と考えると、これは
等角写像であり、t平面の直線ないし円は z平面でも直線ないし円と
なる。上記を既知とすれば、この例は、t平面の t=実数という直線が、
z平面の |z|=1 の円に写像していることを証明すればよい。円に
なることは確かだから、あとは tとして任意の実数値を 3通り代入
しそのすべてで |z|=1 となっていれば証明おわり。大学なら、これ
でいいけど、高校レベルでの証明にはならないね。
>>352
平衡3進整数ってなんすか?googleっても出てこないんですけど。
360132人目の素数さん:02/06/19 14:20
361132人目の素数さん:02/06/19 14:26
w=(1+zi)/(1-zi)で、zが実数(z≠-i)でなかったら|w|≠1であることを示せ。

|w|=1の時,z=-iか、zが実数であることを示せばOK

I)w≠-1の場合
zi(w+1)=w-1
z=-i(w-1)/(w+1)=-i(w-1)(w~+1)/|w+1|^2
(w-1)(w~+1)=ww~-w~+w-1=|w|^2+w-w~-1=w-w~
A=(w-w~)は純虚数
(A~=(w-w~)~=w~-w=-(w-w~).A~=-A, ReA=-ReA,ReA=0だから)
∴z=i(w-w~)は実数

II)w=-1の場合
1-zi≠0でなければならないが
この時w=-1とすると
-1+zi=1+zi
-1=1となり不合理
従ってこのような場合は無い。
362132人目の素数さん:02/06/19 14:26
mod3で0,1,2ではなく-1,0,1
>351
   |z|=1
⇔ | 1-it |^2=| 1+it |^2
⇔ Re(it)=0
⇔ Im(t)=0
364359:02/06/19 14:53
よくわからん。
5=1×3+2だから 10進の5=3進の12 だけど。。。
平衡3進で10進の5はどう表すの?
365359:02/06/19 15:06
2=1×3+(-1)×1か。。。
3=1×3+0×1
4=1×3+1×1
5=1×9+(-1)×3+(-1)×1
つー事は  10進の5=平衡3進の1-1-1?  変なの
366132人目の素数さん:02/06/19 15:58
>>151
なにこれ。できないって解答のページに書いてあるじゃん。
http://www.sansu-olympic.gr.jp/library/1999/final_ans_all.html
できないことの証明は。。。めんどそう。だれかやって。
367132人目の素数さん:02/06/19 17:31
cos(120°×n)+isin(120°×n)
+cos{(-120°×n)}+isin{(-120°×n)}
をどうすれば
2cos(120°×n)
になるようにすればいいのかわかりません。
どなたか教えてください。
368132人目の素数さん:02/06/19 17:34
>>367
おまえなめとんのか
>>367
cos(-x)=cos(x), sin(-x)=-sin(x)
370132人目の素数さん:02/06/19 17:46
中心極限定理の中心ってなんですか?
371132人目の素数さん:02/06/19 17:47
>>367
っていうか、これ3角関数の基本だから。
372132人目の素数さん:02/06/19 17:58
>>371
ってゆーか、これ3角関数のネタだから。
373132人目の素数さん:02/06/19 18:32
X〜N(48, 4), Y〜N(50, 5)

(1) P( X+Y > 100)
(2) P( X > Y)
374132人目の素数さん:02/06/19 19:04
>368
すみません!すみません!
375ドキュソ:02/06/19 19:40
(z-1)(z~-1)=4が点1を中心とする半径2の円だと
いうコトを証明したいのですがアフォなため出来ません(汗
明日当たるのでかなり困ってます・・・
親切な方、どうか教えてくださると嬉しいですm(__)m
376132人目の素数さん:02/06/19 19:43
>>375
おまえなめとんのか
明日あたっとれ
>>375
それは難問ですね。ヒントは背理法を使います。
>377
からかってあげるのもかわいそうかな。
>375
共役複素数を掛けると絶対値の2乗になります。
379132人目の素数さん:02/06/19 19:59
>>378
ってゆーか、これ複素数のネタだろ。
380132人目の素数さん:02/06/19 20:01
>>375
ラジアン使うと簡単に解けるよ。
x=Asinωt
381  :02/06/19 20:03
「4以上の偶数はすべて、2つの素数の和で表すことが出来る」を
誰か証明お願いします。
382ドキュソ:02/06/19 20:05
すいません、なめてるワケでもネタでもないです・・・。
文系で数学ドキュソな為何もわからないんです(涙)
>375
(z−1)(z~−1)=(z−1)(z−1)~=|z−1|^2=4
より
|z−1|=2
|z−1|はzと1との距離
>381
なるものはなる
a{n}=cos(2nπ/3)+Σ{k=1〜n}(1/2)^(kー1)の時、
lim[n→∞](1/n)Σ{k=1〜n}a{k}=(゚Д゚)ハァ?

↑の問題の(゚Д゚)ハァ?を求めていただきたいです。
基本はおさえてるつもりですが、まったく歯が立ちません。
387(゚Д゚)ハァ?:02/06/19 21:16
(゚Д゚)ハァ?=lim[n→∞]Σ{k=1〜n}(1/2)^(kー1)

あ (゚Д゚) と (゚Д゚) は (゚Д゚) 自 (゚Д゚) 分 (゚Д゚) で (゚Д゚) や (゚Д゚) れ
388さき:02/06/19 21:19
こんにちは。中学校2年生です。
今日数学で連立方程式の文章題をやりました。
先生がやりかたを黒板に書いてくれたのですが意味がわかりません。
なぜ、そのようにとくのかが分かりません。
お願いです。だれか教えてください。
先生に聞いたのですが、先生も分からないといっていました。

問題 ある市では、ダンボールと新聞紙を回収しています。
ダンボールは1キロ7円 新聞紙は9円お金がもらえます。
ダンボールと新聞紙をあわせて1050キロ回収して
8950円になりました。それぞれ何キロ回収したでしょうか?

答え 
全部ダンボールだとすると
7×1050=7350
(8950−7350)÷(9−7)=800
新聞紙 800キロ
1050−800=250
ダンボール 250キロ

よろしくお願いします。

>>386
ふざけてごめんなさい。
ネタじゃなく本当にわからないんです。
詳しくおしえていただけませんか?
390132人目の素数さん:02/06/19 21:28
0から9までの10個の数字で表現できる最大の数は?

べき乗は使用可、階乗は使用不可

0から3までなら、2^(10^3) かな?
10^3=?
3^10=?
392NANASI:02/06/19 21:32
すみません、わからない公式があるのですが、書きこんでいいですか?
393(゚Д゚)ハァ?:02/06/19 21:32
>>389
とりあえず、
lim[n→∞]a(n)=α の時、 lim[n→∞](1/n)Σ{k=1〜n}a{k}=α
を示せ。

こ (゚Д゚) れ (゚Д゚) く (゚Д゚) ら (゚Д゚) い (゚Д゚) 自 (゚Д゚) 分 (゚Д゚) で (゚Д゚) や (゚Д゚) れ
>386
a{n}=cos(2nπ/3)+Σ{k=1〜n}(1/2)^(k-1)
cos(2nπ/3)の部分は-1/2,-1/2,1を繰り返すからその狽ヘ-1<=煤@<=0
Σ{k=1〜n}(1/2)^(k-1)の部分は等比数列の和の公式で求める。
{1-(1/2)^n}/{1-(1/2)}=2−(1/2)^(n-1)
これのまた狽セから2n-(等比数列の和)
1/nをかけて極限をとれば2だけが残る
7x+9y=8950(1)
x+y=1050(2)

-(1) -7x-9y=-8950
7(2) 7x+7y= 7350

-2y=-1600
y=800 x=250

これはネタにマジレスと煽られますか?
396395:02/06/19 21:35
>>388
>388
寝たか?まじか?
連立方程式と言いながら、なぜ鶴亀算で求める?
>>394さん
ありがとうございます。
399さき:02/06/19 21:43
397さんへ
答えかたが3つあり、ひとつは連立方程式でした。もう一つは一次方程式でした。
これは、計算のみで答えを出すという方法です。
小学校の算数の方法と先生は言っていました。
400ななし:02/06/19 21:45
60×20のさんぶんのいちじょう、っていくつですか?
>>400
素因数分解しる
402ななし:02/06/19 21:54
すみません、ここに書ききらないので
[email protected]
できいてもいいですか?
>>402
だめ
404402:02/06/19 21:56
うう、、でもそうか、MSNでも
同じですよね。。(悩)
>399
まあネタでもいいから少しマジに答えてみるか。
いわゆる鶴亀算ですね。
集まったものが全部どちらか一方だけだったとすると実際とは差ができてしまいます。
この差はどこから出たか?
それは1Kgあたり9円と7円のちがいです。だからその差2円で割ってやれば
何Kgが違っていたかがわかるというわけです。
406132人目の素数さん:02/06/19 21:58
>>402
マジで聞きたいなら分けて書け。
ネタならおもしろくない
407132人目の素数さん:02/06/19 21:59
量が多いんだろ?
宿題丸投げなんだろ?
408402:02/06/19 22:01
はい、じゃわけてかきます。
7.8=おおきいかっこ、3乗

おおきいかっこのなか=ぶんぼ100
409402:02/06/19 22:01
宿題じゃないですぅ。。一問だけです。
ネタ決定
>400
(60*20)の1/3乗なのか、60*(20の1/3乗)なのか?
いずれにしてもそれ程簡単にはならないが、何が知りたい?
変形なのか近似値なのか。
412132人目の素数さん:02/06/19 22:03
>>408
そんなんじゃ相手にされない。
>>2-10あたりを見て数式の書き方覚えれ。
413さき:02/06/19 22:04
405さん、ありがとうございます。
少し分かりました。
けれどもなぜ、全部ダンボールだとしたのに、新聞紙の重さが答えで
出るのですか?もうすこしお付き合いをしてください。
414402:02/06/19 22:04
それで、分子がかっこくくりが3つあって

(60×20のさんぶんのいちじょう)+(20×にぶんの(x+1.5))さんぶんのいちじょう+(20×1.5のさんぶんのいちじょう)
415132人目の素数さん:02/06/19 22:05
>>408
ペイントソフトで式書いて
bmp→gifに変換して
アプロダにでも上げれ
416402:02/06/19 22:06
ネタじゃないです。。結構必死です。。泣
417132人目の素数さん:02/06/19 22:06
>>400
立方根+筆算で検索かける。
418402:02/06/19 22:07
おい、数学馬鹿ども早く教えて下さい。
419402:02/06/19 22:07
60*(20の1/3乗)です!
420402:02/06/19 22:07
新スレたてて聞こうかなあ。
421402 ◆GO501t7U :02/06/19 22:08
トリップつけました。(煽りがきましたので)
422132人目の素数さん:02/06/19 22:08
-4x^2y^3の次数を言え。という問題がありまして
答えが「次数は5」となっていたので、ああ、2+3=5なんだなあ思っていたら

x^3y + y^2z + zy は何次式か。という問題で3+2+1=6だろうと答を見たら4次式となっていました。

こういう問題ではどうやって答えを出すのでしょうか?
さんぶんのいちじょう
x^(1/3)と書け

ネタじゃないならな。
これでもふざけたら放置する。
424132人目の素数さん:02/06/19 22:08
>>420
それだと荒らされて終り
425402 ◆GO501t7U :02/06/19 22:08
418と420は私じゃないです。。
>413
全部ダンボールとしたのが現実とは違っているので、それがどれだけ違っているか
という誤差(その分が新聞の重さになる)がわかったんです。
427402:02/06/19 22:10
別の所で答え聞いたから、もういいよ。(^.^)/~~~バイバイ
428るーと552049:02/06/19 22:10
>>422
最高次の項だけを見るんだよん。
429132人目の素数さん:02/06/19 22:12
421=425がニセモノだったりして
ネタ決定
431402 ◆GO501t7U :02/06/19 22:14
すみません、書き方があるようなので
よく練習してからまたきます。
432402 ◆GO501t7U :02/06/19 22:14
真面目なのに(泣
433132人目の素数さん:02/06/19 22:15
>>428
この場合 x^3y + y^2z + zy の x^3yを見れば良いのですね
理解できました、ありがとうございました。
434402 ◆GO501t7U :02/06/19 22:22
7.8= (分母が100)^3

     ↑カッコの中の分母 

(60*20^(1/3))+(20* 2ぶんの(x+1.5))^(1/3)+(20*1.5^(1/3))
435402 ◆GO501t7U :02/06/19 22:24
下の一行が分子です(いっぱいいっぱいッス。。)
436132人目の素数さん:02/06/19 22:29
402が努力を認めて誰か教えてあげてください。
437402 ◆GO501t7U :02/06/19 22:29
書き方が悪いのかなぁ。。やっぱり
438さき:02/06/19 22:29
426さんありがとう。
私は中学2年生のあまり頭がよいほうではない生徒なので
どうしてもよく分かりません。簡単に説明してください。
439402 ◆GO501t7U :02/06/19 22:29
ありがとうございます>436
440132人目の素数さん:02/06/19 22:33
>>437
xは実数
8=(1+x)^3
x=?

このぐらいからやってみて
441402:02/06/19 22:35
x=511ですね。
これは分かります
442132人目の素数さん:02/06/19 22:37
>>441
トリップ解析は?
443402 ◆GO501t7U :02/06/19 22:38
いえ、この問題の過程と答えが知りたいだけなのです。
私が解けなくて困ってるわけではないので。。
ネタであることを露呈しますた。。。
445132人目の素数さん:02/06/19 22:39
>>443
ハァ?
自分で解けないから過程と答えが導けないんだろ?
446132人目の素数さん:02/06/19 22:40
なーんだ。俺、釣られちゃったよ。なかなか手が込んでるね。負けたよ。
447402 ◆GO501t7U :02/06/19 22:40
>>434のような場合、
さんぶんのいちじょう、っていうのはさんじょうに相殺されるのでしょうか。
448るーと552049:02/06/19 22:41
解けたんなら過程と答えも一緒にあるだろ。
449るーと552049:02/06/19 22:42
>>445
ケコーン
450132人目の素数さん:02/06/19 22:43
(a+b+c)^3だと思って展開する。

a=x^(1/3)なので
a^3の項があればxに直せる。
>>402
早く寝ろ。明日、遠足だろ。
452402 ◆GO501t7U :02/06/19 22:44
私はもちろん解けません(きっぱり)

解けないのは旦那です。
453450:02/06/19 22:44
悪手だった。まず両辺を(1/3)乗するべきか。
454132人目の素数さん:02/06/19 22:45
質問です。
1から 2^n - 1 までの数字が適当にならんでいて、
二分検索木を作った場合、木が完全に平衡になるときの
数字の並べ方の組み合わせは何通りでしょうか?

1〜3場合
2,1,3もしくは2,3,1の順に並んでいるときに先頭から順に入れていくと
平衡した二分検索木がつくれます。
一般に1〜2^n - 1までの場合、先頭から順に入れていくときに平衡した
二分検索木がつくれるような並びは何通りありますか?
455402 ◆GO501t7U :02/06/19 22:46
展開してみます。ありがとう>450
□αは複素数で、|α|<1とする。複素数zが、|(α+z)/(1+α{bar}z)|<1
を満たすための必要十分条件は|z|<1であることを証明せよ。

□2つの複素数α=√3/2+(1/2)i 、β=(1/√2)+(1/√2)iが与えられている。
この時、-2i=α^n*β^m(1+√3i)となるような自然数の組(n、m)のうちで、
n+mが最小となるものを求めよ。

方針やヒント、考え方など教えていただきたいです。
よろしくおねがいします。
457132人目の素数さん:02/06/19 22:47
>>452
答えと過程を丸々書いてくれるお人よしは少ないよ。
旦那本人をここに呼ぶか、↓で聞き直せば?
こけこっこが全部やってくれるよ。

数学の質問スレその2
http://school.2ch.net/test/read.cgi/kouri/1020087580/
>456
両辺2乗して因数分解

極座標表示
459402 ◆GO501t7U :02/06/19 22:50
両辺を(1/3)乗するべきか

>ありがとう!なんかそれですごくわかったです!
>438
全部ダンボールだとすると
7×1050=7350
(8950−7350) この金額の差はなぜだろう?
全部がダンボールというわけではなかったからですね。
じゃあダンボールを減らして、新聞紙を増やして行きましょう。
1Kgダンボールを減らして代わりに新聞を増やすと2円増えますね。
差額分だけ金額が増えるためには何kg?

余計な話ですが、普通はダンボールのほうが高くないかな。これは今解いてる
問題とは関係なし。出題者の側の問題です。
461402 ◆GO501t7U :02/06/19 22:55
>>457
行ってきます!
>>456
i) |α|<1 かつ |(α+z)/(1+α{bar}z)|<1 ⇒ |z|<1
ii) |α|<1 かつ |z|<1 ⇒ |(α+z)/(1+α{bar}z)|<1
この2つを示せばよい

α=cosA+isinA
β= cosB+isinBに直す

α^n=cos(nA)+isin(nA)
β^n=cos(nB)+isin(nB)
α^n*β^n=cos(nA+nB)+isin(nA+nB)
>>462
丁寧に解説いただき、ありがとうございました。頑張ります。
nとmまちがえた
465132人目の素数さん:02/06/19 22:59
>>454
意味がわからんのだが。
466電気系の1年生:02/06/19 23:20
大学で課題が出たんですけど、よくわからないので手伝ってください。
問1
A=(y+z-2)i+(yz+4)j-xzkとし、Sをx=0,Y=0,Z=0,x=2,y=2,z=2の6つの平面で
囲まれる一辺の長さが2の立方体の表面で、xy平面にない部分とする。このAとSに
ついてストークスの定理が成り立つことを示せ。(ヒント)Sを実際に図示してみて、
またストークスの定理の定義を忠実に考えてみること。

問2
閉曲面Sを境界とする領域Ωの体積をV,nをSの単位法線ベクトルとする。
A=5xi+2yj+3zkとするとき、
∫A・ndS=10V
であることを示せ。(ヒント)発散定理を用いてみよ
467132人目の素数さん:02/06/19 23:22
                   ■弘前強盗放火殺人事件■
   ミミミミ||||||||||||||||||||||イイ〃   犯 人 逮 捕 に ご 協 力 を
   ミ     """""    彡       お 願 い し ま す
   ミ〉 :::: :;;;;;; :::::: :::;;;;;;:: 〈ミ   特徴 年齢40〜50歳ぐらい、
   Y             Y   身長160〜170cmぐらい、中肉、
    | ~"""''''、 ,''''"""~ ::|.   白髪まじり、前髪が立っている
  r'^|シィてユゝ i i ィてエゝミ:|'^.i  目がぱっちりしている、額が広い
  ゝ{|    .ノ | | ヽ、  :|} ノ  津軽弁のなまり、あごがとがって
  .し|:::  ´ ノ :ヽ    :|ノ   いる、頬に縦じわがある
    | 、 ./〈r、...r.〉\ , .|
     | ヽ ,__..,;,,;,,.__、/ /    この男に関する情報をお寄せください
    ,ト、 ´ ̄~~ ̄` /;;|.    青森県弘前警察署特捜本部
     . `\、,.__,./´      0 1 2 0 − 3 0 7 X X X
>>466
ヒントの通り
>>466 この種の問題はね、左辺の計算は地獄、右辺は天国。で、一度
地獄の計算をして、ガウスの定理ないしストークスの定理のありがた味を
身を持って体験させようというものだ。質問スレに出したって、だれ
も地獄の計算はやらないよ。
Aを濃度が非可算無限の集合とする。
また集合Sの閉包は(S)^cと表す。
Y=∪[α∈A]X_αとした時
(Y)^c=∪[α∈A](X_α)^cは成り立つか?
成り立つ場合は証明を、そうでない場合は反例、もしくはその証明をしたいんです。
よろしければ教えてください、お願いします。
>>470
X_α=[0,α]⊂R (0<α<1)
Y=?
成る程そういう反例があるか…
んじゃ、ちょっと条件をつけます。
Yをベクトルスペースとし、可分でないとします。
んで、BをYに含まれる濃度が可算無限の部分空間とし、
X_αをBによる商と仮定した場合についてはどうでしょうか?
473132人目の素数さん:02/06/20 03:38
>>472
言ってることがよくわからん。
>X_αをBによる商
ってどういう意味???
f,g∈A_αとすると、f-g∈Bになるという事です。
そういう条件を満たす集合を作って、それぞれに添数をつけ
その添数の集合がAということになります。
書き方が悪かったですかね。
それ以外はOKですか?
475132人目の素数さん:02/06/20 03:51
閉包とるとき、X_α⊂Yで考えるんだね。
X_α⊂Y/Bじゃないんだね。
そのとおりです。
やっぱり書き方が悪かったみたいですね、申し訳ありません。
もう一つ聞く。

Y,Bの細かい条件無視して、
Yをx-y平面、Bをx軸とする。
このとき、X_α={(x,0) | 0<x<1}なんて言うのも許されるの?
>f,g∈A_αとすると、f-g∈Bになるという事
これだけ読むとそういう風にとれるけど。
ん〜、出来れば関数空間で考えて欲しいんですけど…
それに477さんの例ではどこまでが細かい条件を無視していて
また、一般的にその例がどういった事を意味しているのか良く掴めません。
Yが可分でない事やY=∪[α∈A]X_α、Bが可算個の元からなる部分空間である事は外したくないんですけど。
この例だけ見るとY=∪[α∈A]X_αがまず成り立っていないような気がしますが…
仮にX_α={(x,α) | x∈R}(α∈R)としてもBが非可算になってしまうので、ちょっと困ります。

しかし、こういう事を聞かれるところを見ると、やはり成り立たないんでしょうね…。
479132人目の素数さん:02/06/20 05:34
>>470
>(Y)^c=∪[α∈A](X_α)^cは成り立つか?
xが右辺に属するならば、あるαがあって、xはX_α^cに属する。
⇔あるαがあってxのどんな近傍もX_αと共通部分を持つ。
⇒xのどんな近傍も、Yとの共通部分が空でない
従って⊇は言える。


反例
f(x)はR^n上の関数で、各点収束の位相を入れる。
Supp F={x|f(x)≠0}とする。
X(r)={fのSupportがコンパクトで、かつ半径rの球に含まれる}
X(r)^cに属する関数fのSupportもコンパクトであることはすぐにわかる。
従って右辺に属する関数のSupportはコンパクトにならなければならないが、
左辺に属する関数は、そうとは言えない。

>>479
回答ありがとうございます。
ただ、理解力がなくて申し訳ないですが、この場合Bに相当する集合は何になるのか分かりません。
それと、
X(r)={fのSupportがコンパクトで、かつ半径rの球に含まれる}

X(r)={fのSupportの閉包がコンパクトで、かつ半径rの球に含まれる}
という事で良いでしょうか?
481132人目の素数さん:02/06/20 06:44
>>480
R^nのコンパクト集合は、有界閉集合だから、閉包を取っても変わらない
ので、その点は気にしなくていいと思う。
Y^cとはYの元が幾らでも近くにある元の全体のことだった。

n=1で説明しる。XとてR上の関数の全体とする。

X(n)={f(x)|f(x)=0 if |x|>=n}とする。
この時
f_n(x)=1-|x|/n (|x|<=n),それ以外は0
とおく。
lim f_n(x)=1であることがわかる。
f_n(x)∈X(n)だから
定数関数1のどんな近傍にも、あるnが存在して、X(n)の元f_n(x)が
あることを意味するので、定数関数1はY^cの元。
しかし、X^c(n)にぞくする関数fはf(x)=0(|x|>n)を満たす。(→問題)
従ってもし定数関数1が右辺に属しているとすると、あるnがあって
X^c(n)に属する必要があるが、条件を満たすnなど無い。
482132人目の素数さん:02/06/20 10:26
N人の人間がいるとして、誕生日が一致する確率を教えてください…
>>482 アホスレ「誕生日のパラドクス」
http://science.2ch.net/test/read.cgi/math/1021473768
を頭から50記事ほど読め。
484132人目の素数さん:02/06/20 10:58
>>483
組み合わせとか馬鹿みたいに考えてたら
1から引けばいいことに感激しますタ
485132人目の素数さん:02/06/20 11:12
問) 点A(2x1、2y1) B(2x2、2y2) C(2x3、2y3) D(2x4、2y4)を頂点とする四角形ABCDの
  辺AB、BC、CD、DAの中点をそれぞれP、Q、R、Sとするとき、次の等式が成り立つことを証明せよ。
    AC^2+BD^2=2(PR^2+QS^2)
--------------------------------------------------
  P(x1+x2、y1+y2) Q(x2+x3、y2+y3) R(x3+x4、y3+y4) S(x4+x1、y4+y1)
--------------------------------------------------

左辺={(2x3−2x1)^2}+{(2y3−2y1)^2}+{(2x4−2x2)^2}+{(2y4−2y2)^2}
  =途中計算(長いので省略)
  =4[{(x3−x1)^2}+{(x4−x2)^2}]+4[{(y3−y1)^2}+{(y4−y2)^2}]

右辺=2[{(x3+x4−x1−x2)^2}+{(y3+y4−y1−y2)^2}+{(x4+x1−x2−x3)^2}+{(y4+y1−y2−y3)^2}]
  =途中計算(長いので省略)
  =4[{(x3−x1)+(x4−x2)}^2]+4[{(y3−y1)+(y4−y2)}^2]

となってしまい、両辺が合わなくなってしまいました。
どう間違っているのですか、教えてください。
>>485 この問題は >>243 >>248 >>249 でガイシュツ。
487485:02/06/20 11:57
おっと、過去に同じ質問をしたことを忘れていました。すいません。
488132人目の素数さん:02/06/20 13:11
与えられたデータについて最小二乗法を用いて近似直線を
求めよ。傾き、切片ともに小数点以下2桁まで求めればよい。

x 1 2 3
y 0.45 0.85 1.10

という問題です。
ぜひお願いしますです。
>>488 こんな計算だれがするものか。公式は web にいっぱいころがってる。
自分でやれ。たとえば、
http://aoki2.si.gunma-u.ac.jp/lecture/Regression/sreg/sreg.html
490132人目の素数さん:02/06/20 13:20
>>485
>途中計算(長いので省略)

暗算で出るよ。ぜんぜん長くない。

>>487
・・・アンタねえ
491490:02/06/20 13:24
>>485
言い忘れた。「右辺」が間違い。
>>481
>R^nのコンパクト集合は、有界閉集合だから、閉包を取っても変わらない
>ので、その点は気にしなくていいと思う。
あ、確かに。
訳分からん事を書いてしまいました、すみません。

あと、一応479の反例は一般の場合(n≧1)で考えても分かるんです。
しかし、条件を限定してYに対して可算部分空間Bをとって
f〜g⇔f−g∈B(f,g∈X_α)として同値関係を定義し、
各同値類X_αをY/〜の元はなく、Yの集合として考えたいのです。
この具体例の場合はX(n)⊂X(n+1)だから、同値関係は定義されてません…よね?
X(n)={f(x)|f(x)=0 if |x|>=n,|x|<n-1}と定義すると、Limf_n=1が成り立ちませんし。
注文が多くて申し訳ないです。
>>489
すいませんです。
でもいいページありがとうございます。
494132人目の素数さん:02/06/20 15:41
>>296です。
>8で割ったあまりが1、3、5である奇数は3つの平方数の和でかける。
>らしいのですがこれの証明どうするんでしょう?
>日本語の本で証明ののってる本しりませんか?
これJ.P.セールの数論講義って本にのってるかもしれないという情報が
あるんですがこれ持ってるひといませんか?のってるなら注文して買おうと
おもうんですがのってなかったらやなもんで。どうなんでしょう?
495132人目の素数さん:02/06/20 15:45
>>492
過去記事よんでみましたが、どんな問題をかんがえていてそのような
問題にをかんがえるに至ったかもう少し細かい状況をカキコしたほうが
いいんじゃないですか?
496ベッセル関数名無しさん:02/06/20 16:28
ベッセル関数の積分の漸近形を探しています。ご存知の方は教えてください。
【詳細】
整数次の第1種ベッセル関数 J_n(x) の区間 0からx の積分をA_n(x)とする。
xが大きなところの関数形を求めよ。ただし、0から無限の定積分はWeber積分
と呼ばれていてガンマー関数で書けることは知られている。またJ_n(x)自身の
漸近形はよく本に載っている。知りたいのは積分した A_n(x) の漸近形。
>>496
数学辞典にのってる停留位相の方法ってつかえないの?
498132人目の素数さん:02/06/20 18:04
aa
499132人目の素数さん:02/06/20 19:00
>>282 >>88 >>95
 あまり、自信が無いですが、こういうのはどうでしょう。。
まず、a^2+b^2=N(ab+1)(すべての文字は正整数 a>b)となったとします。
 ここで他の整数解を探してみます。
   c^2+b^2=N(cb+1)となるcがあればいいです。
   (c^2-a^2)=N*b*(c-a)
(c+a)=N*b―☆
よってN*b-a=cとなるようなcをもってきて(c,b)の組を作れば条件を満たします。
 ここで、ある平方数でないNに対して、上のa>cのようなより小さな整数解を常に
作ることが可能であることを示すことができれば無限降下法を用い、矛盾が起こります
 もし、より小さな整数解を作ることのできないa,bがあり、Nが平方数でないと仮定します。
500132人目の素数さん:02/06/20 19:07
(1)N*b-a=cが正でないときN*b≦aです。
 ☆にこれを代入し展開すると、b^2≦N
 これを再び☆に代入し整理するとb^3≦a
ここで等号が成立すると、Nは平方数なので、成立しません。
 ∴a=b^3+kとかけます。a
 (kは正整数)
 与式=(a^2+b^2)/(ab+1)=(a*b^3+b^2)/(ab+1)(←b^2)+a(a-b^3)/(ab+1)
ところがaと(ab+1)は互いに素なので(a-b^3)/(ab+1)=k/(ab+1)
が整数で無ければなりません。ところが明らかに分母より分子が小さいので整数と
ならず矛盾します。
 (1)N*b-a=cが正のときN*b>aです。
条件よりa≦c N*b=a+c≦2a
a^2+b^2=N(ab+1)≧2a^2+N
-a^2+b^2≧N ところが左辺が負で矛盾。
 よって無限降下法により矛盾。
501132人目の素数さん:02/06/20 19:09
追記:もし小さい整数解を作ることのできないA,Bがあれば矛盾
よって、常に作れる。
よって矛盾ということです
502ベッセル関数名無しさん:02/06/20 19:19
>>497
>数学辞典にのってる停留位相の方法ってつかえないの?

もう少し詳しく教えて。「数学辞典」とは例えばどこの会社のものですか?
手元の辞典にはINDEXにありませんでした。
それと停留位相法が「鞍点法」または「最急降下線法」と同じ方法でしょうか?
そうすれは、>>497 さんの方法は普通のベッセル関数の漸近形を求めるのと
同じ方法になりますね。
>>502
いや。詳しくはしらんのだけど。岩波の数学辞典の漸近級数のページみてたら
f(x)=∫[a-d,a+d]exp(-ih(s))g(s)ds
の形の関数は
f(x)=exp(ixh(a))+c)(1/√x)(・・・)
の形の漸近展開をもつってのがのってたもんで。(ただしh'(a)=0,h''(a)≠0)
それで公式集のBessel関数の定義みてみたら
Jn(x)=(定数)∫[0,Π]exp(ixcos(s))cos(ns)ds
ってなってたからちょっと変形したら
Jn(x)=(定数)(1/2)∫[-Π,Π]exp(ixcos(s))cos(ns)ds
になってxについて不定積分したらさっきの定理のつかえる形に
なるんじゃないかな〜ってふっとおもった。でも手元に漸近展開の資料も
なんもないのでほんとにできるかどうか自信ないけど。
・・・のとこ具体的にどうなるのかまでは全然かいてないし。g,hの微分係数で
書けるとしかかいてない。
2行目
f(x)=∫[a-d,a+d]exp(-ixh(s))g(s)ds
に訂正
505132人目の素数さん:02/06/20 19:57
∫sinx/xdx
はどうなりますか?
506132人目の素数さん:02/06/20 20:40
●8進数において 702−356はいくつになるか。
●5進法で表すと 43203となる数字を
 7進法で表すといくつになるか。

全く分かりません。よろしくお願いします。
簡単な解説を添えていただけたら嬉しいです。
507132人目の素数さん:02/06/20 20:49
まず、各々十進数には直せますかな?
>>506 ヒント
7*8^2+0*8^1+2*8^0
3*8^2+5*8^1+6*8^0
4*5^4+3*5^3+2*5^2+0*5^1+3*5^0
509506:02/06/20 21:02
>>507
すみません、直せません。2進数→10進数はできますが…
510132人目の素数さん:02/06/20 21:08
□αは複素数で、|α|<1とする。複素数zが、|(α+z)/(1+α{bar}z)|<1
を満たすための必要十分条件は|z|<1であることを証明せよ。
という質問をきのうさせていただきましたが、
>>両辺2乗して因数分解
とのヒントをいただきましたが、
そうすると、|α|^2+2αz+|z|^2<1+2α(bar)z+(α(bar)^2)z^2
↑をどう因数分解できるのでしょうか?
>>509
>>508読め
512132人目の素数さん:02/06/20 21:12
lim(n→無限)sin(πn!)
は0になるのですかならないのですか?
513132人目の素数さん:02/06/20 21:20
>510
展開を間違えてるよ。
514132人目の素数さん:02/06/20 21:32
>>513さん
何度やっても↑になるんですが、
|(α+z)/(1+α{bar}z)|<1 これを|(α+z)|<|1+α{bar}z)|
とできますよね?
515132人目の素数さん:02/06/20 21:39
複素では一般に z^2≠|z|^2
|α+z|^2を展開してみてくらさい。
517132人目の素数さん:02/06/20 21:40
>>512
0です
518132人目の素数さん:02/06/20 21:44
>>516さん
|α|^2+2αz+|z|^2ですか?
519132人目の素数さん:02/06/20 21:45
統計の質問をさせて頂いていいでしょうか?

実はX、Y、ΔYの実験データに傾きがあるかどうか検定したいのですが、

一定値、一次関数のχ2乗フィッティングとFーテストとしたのですが、

もとのエラーがχ2乗分布に従う時しか正しくないと指摘されてしまいました。

線形相関係数を使う様にと言われましたが、線形相関係数 rと t分布検定と

言う事でしょうが、線形相関係数は、XとYだけで計算されるので

おかしいなぁーと思っています。

どうするのが正しいのでしょうか?
>518
違う。定義に従って絶対値の計算をしてみよう。
521132人目の素数さん:02/06/20 21:55
>>517
教科書に
limsin(πen!)=0
を証明しろという問題があったんですが、
でも,πも有理数の無限級数や積で表されるから,
limsin(π*πn!)=0 ってのも成り立つかなと
じゃあ
lim(n→無限)sin(πn!)
はどうなるのかなと思ったんですが
522132人目の素数さん:02/06/20 21:59
すいません、ここに質問するのもお恥ずかしいのですが、

    123.40
3870× ーーーー
     3.77
をどなたか解いていただけないでしょうか。明日までに仕事で必要なんですが、
昔から算数全然できなくて。−−−は、割るということです。
よろしくお願いします。
>521
nが整数の時、sin(πn!)はいつも0でしょ。
極限とって別物がでてくる筈はない
524132人目の素数さん:02/06/20 22:00
>>520 自分で適当な文字をおくのでしょうか?
α=x+yi
z=a+biとして、
|α+z|^2=(√(x+a)^2+(y+b)^2)^2でしょうか?
>519
話しがよく分からん
正しくないと指摘したのは誰で
それはキミの計算結果が違ってたっていう意味?
>524
絶対値|z|≧0の定義は
|z|^2 = z z~
>522
電卓を使え
Windowsなら、標準装備してるはず
528132人目の素数さん:02/06/20 22:05
>>523
じゃあ
limsin(n!)も0になると?
529132人目の素数さん:02/06/20 22:06
>527
電卓あっても、どう計算していいか分からないんです。
でも、ここで聞くのは場違いだったかも、すいませんでした。
>>528
あんた馬鹿?
>528
sin(n!)はnが整数の時いつも0になると?

キミの脳内ではsin(1)=sin(2)=sin(6)=…=0?
>>521
ちょっと、ちょっと。みんなしれっと無視してるけど
>limsin(πen!)=0
>を証明しろという問題があったんですが、
これほんと?どうやんの?
533132人目の素数さん:02/06/20 22:08
>>529
電卓の使い方がわからないとはなんと奇特な人だ…
126774.20954907161803713527851459
534132人目の素数さん:02/06/20 22:10
>>526
|α+z|^2↓
(α+z)*(α+z)~↓
αα~+αz~+α~z+zz~でしょうか?
535132人目の素数さん:02/06/20 22:10
>>531
だから、>>523が正しいとしたらだって,
しかもいつもではなく,n→無限のとき
>>535
ますます馬鹿露呈中です
537132人目の素数さん:02/06/20 22:12
>534
あってるよん
538132人目の素数さん:02/06/20 22:13
>>532
e=1/0! + 1/1! + 1/2! + 1/3! + …
なので、n>2 のとき、 e×n! の小数部分は
1/(n+1)) + 1/((n+1)(n+2)) + …

n→∞ のときに、 (e×n! の小数部分)→0 を示せばいい。
539132人目の素数さん:02/06/20 22:13
報知、決定!
>535
その極限は
nが実数全てを通って無限に発散していくときという意味ではなく
nが1,2,3,…と自然数値を取りながら発散していくときという意味だよ
極限をとるときはどういう点列で極限をとるかで、全く違う答えが
でてくるのはよくあることだよ

って高校でやらなかった?
>535
ひょっとしてまだ高校逝ってなかったのならスマン
542132人目の素数さん:02/06/20 22:15
>>538
なる!thx
543132人目の素数さん:02/06/20 22:17
とりあえず,
>>523はちがうってことでいいな?
ネタ放置
545132人目の素数さん:02/06/20 22:20
ということは
|(α+z)|<|1+α{bar}z)|

αα~+α~z+αz~+zz~<1+αz~+α~z+αα~zz~となるんですか?
546519:02/06/20 22:21
>525
正しくないと指摘したのは論文のレフリーで
結果については同意するが、統計処理が不適切と言う
コメントでした。

X、Y、ΔYのあるデータに傾きがあるかどうかの検定で、
一定値と1次関数のχ2乗フィッティングをして、パラメターを
増やして意味があるかどうかFー分布のテストをしたのですが、
χ2乗分布の場合しか正しくないと指摘され、
相関係数を使うべしとの御言葉ですが、
(線形)相関係数の計算式はXとYしかないので不思議に思って
いるのです。普段しない統計処理なので判らなくて質問しています。
>545
そのとおり
548506:02/06/20 22:26
遅れましたが>>506です。
>>508さんのヒントを参考に解いてみたのですが、答えはそれぞれ
●450−238=212
●2928
で よろしいでしょうか?
549高1くん ◆nOegwuqo :02/06/20 22:28
ちょっと前に質問したものですが、この板ってなんで解答してくれないわけ。受験板のほうが解答してくれるし。
しかも質問者を馬鹿扱いで煽るだけ。数学だけ見ても医学部よりできないんだよね、理学部とかってれべる低いし
レベルの低い煽りは無視
551132人目の素数さん:02/06/20 22:30
くだらない質問には答える気力がわかないんだよ
552132人目の素数さん:02/06/20 22:30
>533
あれ答えだったんですね。ありがとうございました。
553複素数(゚Д゚):02/06/20 22:31
>>547
バカな質問につきあわせてしまってごめんなさい。
でもおかげさまで、大事なところを思い出せました。
近いうちにまた複素数の質問するかもしれませんが、よろしくです。
>549
そもそも、この板は中高生の宿題に答える板では無いのです。
受験板で答えてもらえるのであればそちらで聞けばいいんでは
ないでしょうか?
555高1くん ◇nOegwuqo:02/06/20 22:32
偽者が暴れているようじゃ
失敬、失敬
556高1くん ◆nOegwuqo :02/06/20 22:32
>>550
れべる低い煽りは、この板のレスだ
しつもんしゃにとってはくだらなくないんだよ

>>551
くだらない、っていうけど、大学で数学習う方がくだらないじゃん
557132人目の素数さん:02/06/20 22:34
次の方、どうぞ
558132人目の素数さん:02/06/20 22:36
哀れだねぇ
煽りも中途半端で駄目ぼ
>535=>556=今井
ってことでよろし

(´-`)。oO(この程度の煽りを相手してくれる数学板は優しいなぁ。。。)
562高1くん ◆nOegwuqo :02/06/20 22:43
絶対値が1の複素数Z1、Z2、Z3があり
A=(z1+z2)(z2+z3)(z3+z1)/(z1z2z3)

Aの最大値と最小値は?
という問題ですが、教えてください。(煽りとかじゃないです)
563高1くん ◆nOegwuqo :02/06/20 22:45
>>560
今井ってだれ、医学部?
564132人目の素数さん:02/06/20 22:46
次の方、どうぞ
565548:02/06/20 22:49
>>506なのですが、>>548の解答で正解でしょうか?
もうキャップ付きでは誰も相手にしないよ
さようなら
>>565
>>548の212というのは何進数で書いているんですか?
568548:02/06/20 23:05
>>567さん
448 と 238 は8進数に直しまして
それを普通に引いて212としたのですが
それも8進数に直す必要があるという事でしょうか?
569 :02/06/20 23:10
⊇、⊆っていう記号、学校で教えてもらった「部分集合」の記号と
微妙に違うんですけど、、、これって下がイコールなんじゃないですか?

あと、∩、∪は「かつ、または」の意味だと思うんですが
⊃、⊂ってのは、どういう意味なんですか?
570132人目の素数さん:02/06/20 23:12
>>568
そもそもどうなんだろ。これ
>●8進数において 702−356はいくつになるか。

A:702−356を8進数の引き算とみなしたときいくらか8進数でこたえよ
B:702−356を10進数の引き算とみなしたときいくらか8進数でこたえよ
C:702−356を8進数の引き算とみなしたときいくらか10進数でこたえよ
どれなんだろ。問題文の作者の作文力を問い詰めたい。小1時間ほど。
>548,568
1つ目は、8進数でといっているから答えも8進数の方がいいと思う。8進数のまま
計算するのが楽。(上の位から借りてくるとき、10じゃなくて8にする)
2つ目はまだ7進数にする仕事が残ってます。
>>568
●進数で答えよ。と指定がないなら、何進数で答えてもいいと思うが、
>>548の「212」は10進表示である』ことを明示しなければならない。

まあ、8進数同士の計算問題だから、答えも8進数に直しておいた方が
自然で、採点者の負担が減って好感触となるはずだ。
>569
元々下の部分は=だった。(過去形、今でも名残はあるが)
数学をやる奴はずぼらというか、面倒くさがりというかどんどん手抜きして
いつのまにか正式の記号になった。=を書かない流儀もある。
574132人目の素数さん:02/06/20 23:18
>>569
>⊇、⊆っていう記号、学校で教えてもらった「部分集合」の記号と
>微妙に違うんですけど、、、これって下がイコールなんじゃないですか?
これイコール付ける派と付けない派の2派がそのむかしあったらしい。
という話を先生がしてた。今は付けないほうが主流だとおもうんだけどどうなんだろ。
つけてもつけなくても意味は通じるしおなじ意味にとられるよ。
A⊂BはAはBに含まれるの意。しいて“AはBに含まれ且つA≠Bである”
をいいたいならA⊂Bの下に≠をつけてかく。
>>569
≧も ≥ なんて書くこともあるよん。

(環境によって見えないかもしれないが)
576線形代数の問題です:02/06/20 23:21

n次の多項式行列A(x)の階数がnであるための必要十分条件は、
det(A(x))≠0であることを示せ


n次の多項式行列、A(x)とtA(x)は同値であることを示せ


n次正方行列AとtAは相似であることを示せ

よろしければヒントでもうれしいので教えてください。
577132人目の素数さん:02/06/20 23:22
正直村と嘘つき村がある。
正直村の人は必ず本当の事を言い、
嘘つき村の人は必ず嘘をつく。
1回だけ質問して正直村か嘘つき村かを
調べるにはどうすれば良いか?

どうすればいいのですか?
578132人目の素数さん:02/06/20 23:23
     1  1  1
4÷5=―+―+―  これ解けますか?(2分以内)
     ○  ○  ○
580132人目の素数さん:02/06/20 23:24
もう1人に尋ねたらどう答えてくれそうでしょうか?

あなたはどっちから来たんですか?

etc
581132人目の素数さん:02/06/20 23:27
>>578
ab=a−b
b(a+1)=a
b=a/a+1
a=−1、b=なし
582548:02/06/20 23:29
>>570さん >>571さん >>572さん

とても参考になりました。
明日8進数の問題について、小1時間ほど問い詰めてきます。
本当にありがとうございました。
>>562
最大値8
最小値0
過程はご自分で。
584569:02/06/20 23:32
ってことは、教科書は昔のことを教えてるんですね(^^;
⊇、⊆ってのは、集合同士の関係で「部分集合」を表す記号で
⊃、⊂ってのも、おなじ意味ってことですか?
585某定時校生:02/06/20 23:33
>>577
物理版からいらっしゃったのですか?
586132人目の素数さん:02/06/20 23:33
>>584
ご明察。
587132人目の素数さん:02/06/20 23:33
>>584
そうだっていっとろうが
>>584
正解
>577
有名問題。普通は村人はyes(はい)no(いいえ)しか答えないと限定されている。
「ここは正直村かと聞かれたらyesと答えるか」のように2つの事柄を
結合させて聞く。うそつきも2重否定をして本当のことを言うことになる。
590569:02/06/20 23:37
納得。ありがとうございました!
1週間後のテストに向けて頑張ってきます。
591七誌:02/06/20 23:38
∫exp(−x2乗)dxを解いてください!
592132人目の素数さん:02/06/20 23:39
三角形ABCにおいて
cosA+cosB+cosC>□
である。□を求めよ。
どのように解けばいいんでしょうか?(ちなみに答えは□=1)
>>577
1+1は2ですか?とかあなた人間ですか?とかいろいろありそうな。
相手がバカとか人間じゃないとこまるけど。
594132人目の素数さん:02/06/20 23:43
3行3列正方行列A=
┌ a b c ┐
| d e f |
└ g h i ┘
のトレースと逆行列A^(-1)、転置行列tAを求めてください。
>>591 (√π/2) erf(x) + c
596七誌:02/06/20 23:52
>>595
erfってなんですか?
597あいこ:02/06/20 23:55
y=x^m×(a-x)^nのグラフの概形はどうやったらかけますか?
与えられた条件はa>0でm,nは1より大きい自然数。
  ヒントもありました。;m.nが偶数、奇数にわけよ。ということです。
>>594
trA=a+e+i
detA=a(ef-hi)-b(di-fg)+c(dh-eg)
tA=
┌ a d g ┐
| b e h |
└ c f i ┘
なにこれ?
599132人目の素数さん:02/06/20 23:59
591の問題だれも解けないの?
有理化の質問に答えてくれた先生ありがとう。
601132人目の素数さん:02/06/21 00:00
>>597
ひたすら微分してつかむ
602132人目の素数さん:02/06/21 00:02
∫[0,1]x{∫[x^2,1]e^(-y^2)dy}dx
この累次積分の順序を変えることにより積分の値を求めよ
おねがいします
603あいこ:02/06/21 00:02
>>601
それ以外の方法はないですか?
604132人目の素数さん:02/06/21 00:04
>>603
どっちみち微分しなければ見えてこない。
この手の問題はひたすら自分でやって変化を追っていくのが大事です。

奇数と偶数に分けるのはポイントです。
605あいこ:02/06/21 00:06
>>604
奇数と偶数に分けることはどういった
意味があるのでしょうか?
606132人目の素数さん:02/06/21 00:10
>>605
-∞<x<0やa<x<+∞での様子を書くときに場合分けが効くのだろう。
0<x<aでのようすでは、m,n>=2ならだいたい形は似ているだろう。
607132人目の素数さん:02/06/21 00:11
どうでもいい話だが、最近はあいこもどきの女が多いな。
俺のいるとこでもみかけるよ。
>>602
(与)=∫[0,1](∫[0,√y](xe^(-y^2))dx)dy
=∫[0,1](y/2)e^(-y^2)dy
以下できるだろ。
609132人目の素数さん:02/06/21 00:19
すいません・・・
だれか>>576の問題を考えていただけませんか?
610132人目の素数さん:02/06/21 00:36
>>609
考えてあげたいのはやまやまなんだけど言葉がわからんのよ。
“AとBが相似”ってのはたぷん“A=PBP^(-1)となる正則行列がある”
って意味だとおもうんだけど“A(x)とB(x)が同値”とはなんぞや?
君の学校のせんせか君の教科書独特のことばだと思うので手がだせん。
君は“A(x)とB(x)が同値”という言葉の意味は説明できる?
∫e^(−x^2)dxを解いてください
612132人目の素数さん:02/06/21 00:39
すいません>>592の問題教えてください。
お願いします。
613609:02/06/21 00:45
>>610
“A(x)とB(x)が同値”とは、「行列の同値」の意味で、

A、Bに対して、
PAQ=Bとなる
正則行列P、Qが存在することを言うそうです。

相似と似ているみたいです。
で、そのことを「A〜B」で表します。
 cosa+cosb+cosc
=2cos((a+b)/2)cos((a-b)/2)-2cos^2((a+b)/2)+1
=2cos((a+b)/2){cos((a-b)/2)-cos((a+b)/2)}+1
=4sin(a/2)sin(b/2)sin(c/2)+1
615132人目の素数さん:02/06/21 00:48
>>612
これまじだったの?あの前後になんか妙なあらしがはいってたので
そいつかと思った。これ
cosA+cosB+cosC>□を満たす最大の□を求めよ。
だね。つまりS=cosA+cosB+cosCの変化範囲をしらべればよい。
C=180°-(A+B)で一変数けして
S=cosA+cosB-cos(A+B)
こいつのA>0,B>0,A+B<180°におけるSの範囲をしらべればよい。
凸関数の範囲に関する定理をしってるとちょっとらくできるけど
素直にBを固定して0<A<180°-BにおけるSの範囲をもとめて
その最小値m(B)の範囲をしらべればよい。
616132人目の素数さん:02/06/21 01:04
>>610
ならこんな感じかな?

Aの階数=行列式が0でない小行列の最大サイズ
だから。

AとBが同値⇔AとBの階数が等しい
だからA(x)とtA(x)は同値

これはちょっとむずかしい。ひとまず
AとBが相似⇒tAとtBが相似
をしめしておいてジョルダンの定理から
Aはあるジョルダンの行列J_k(a)の直和B(=斜めにならべたもの)に相似、
このときtAはtBに相似なので結局BとtBが相似であることを示せばよい。
PをP[ji]=1(j=n-i+1),=0(j≠i-n+1)でさだめられる行列
(つまり右上から左下にむけて1をならべた行列)とすると
P^(-1)BP=tB
から題意がしめされる。
・・・
こんな感じかな。
617132人目の素数さん:02/06/21 01:09
>>614>>615 解答ありがとうございました。
>>615
>素直にBを固定して0<A<180°-BにおけるSの範囲をもとめて
>その最小値m(B)の範囲をしらべればよい。
このあたりをもう少し詳しく教えてください。
618609:02/06/21 01:15
>>616
ありがとうございます。
1、2はわかりました。
3も難しいですが理解できます!
でも3はもっとシンプルな解答にもなるはずなんです。
もう少し考えてみます。

あと、正方行列になりたつ性質をそのまま多項式行列に利用してもいいものなのでしょうか?
例えば、AとtAの階数は等しいから、
A(x)とtA(x)の階数は等しい、
とか。
619132人目の素数さん:02/06/21 01:17
>616
[1 0]
[0 0]

[x 0]
[0 0]

共にrank=1 で同値になる?
誰か>>579解けた人いますか?
俺はだめだった・・・
>>620 ん? >>579 の答か?

1/2 + 1/4 + 1/20 = 4/5 あるいは 1/2 + 1/5 + 1/10 = 4/5

じゃだめか?
622132人目の素数さん:02/06/21 01:29
>>617
S=cosA+cosB-cos(A+B)
=2sin(A+B/2)sin(B/2)+cosB (0<A<180°-B)
でA+B/2がB/2<A+B/2<180°-B/2まで動くことを利用すると
(単位円かいてみればわかる。)
S>2sin^2(B/2)+1
=2(1-cosB)/2+1
=1
からBを固定したときのSの変化範囲の下はじが1だとわかる。つまりm(B)=1。
(さっきは最小値といったけど正確には下限値という。最小値とかくと減点)
これからSのへんか範囲の下端が(Bをうごかしてもやっぱり)1だとわかる。
ちなみに>>614さんの変形をつかうともっと楽。たしか京大の過去問の
解答の質問の過去ログ(さくらの一桁あたり)でだれかつかってたとおもう。検索してみ。
623132人目の素数さん:02/06/21 01:31
>>621
thanks
624助けて〜:02/06/21 01:32
3点A(0,1,2)B(1,2,1)C(4,-1,2)を通る平面をαとする。
(1)△ABCの面積を求めよ。
     答え √14
(2)原点Oから平面αに垂線OHを下ろす。このとき、点Hの座標と線分OHの長さを求めよ。

(3)四面体OABCの体積を求めよ。

2,3番が分かりません。誰か解答教えてください。お願いします。
625132人目の素数さん:02/06/21 01:33
>>619
もしかして同値の定義の正則行列は有理関数体でとれないってか。
つまりdiag(x,1,1,・・・1)は正則行列じゃないとみなせってか。
そうかもね。でも韓国語講座もおわったしもう寝たいんだけど。
626132人目の素数さん:02/06/21 01:39
はじめまして。
錘の定義を教えていただけませんか。
627132人目の素数さん:02/06/21 01:41
すいすい
628609:02/06/21 01:43
三角錐
629132人目の素数さん:02/06/21 01:43
>>619
>>619さんの指摘をうけたので係数環が多項式環だとして再挑戦。
もとの係数環kは体だとして(つまり体上の多項式環だとして・・・
これぐらいは仮定させてくれ。)
任意のk[x]係数の行列A(x)はB(x)diag(d1(x),・・・)と同値。
いか3の解答と方針はいっしょ。
これでいいかな。もねます。まちがってたらゴメ。
いちおう

4÷5=の問題
http://science.2ch.net/test/read.cgi/math/1017927566/
>>624
ガイセキ・・・
632これもおねがいしまっす:02/06/21 01:46
複素数ZがZ4+Z2+1=0(Z4乗+Z2乗+1=0)を満たす。次の値を求めよ。
(1)Z6(6:6乗)
(2)|Z|
(3)|Z−@|2+|Z+@|2 (2は2乗ってことです。)
633132人目の素数さん:02/06/21 01:47
ガイセキ・・・???(涙)
634132人目の素数さん:02/06/21 01:52
>>622
ありがとうございました!
京大の過去問だったんですか・・・
635132人目の素数さん:02/06/21 01:56
すいません、この積分がわからないのでよろしくお願いします。
∫[0,x](tanh(x)/x)dx
636132人目の素数さん:02/06/21 01:56
>>624
(2)がわかれば(3)はおまけ問題。
(2)はいろんなやり方ができる。

(a)
平面αの方程式を出す。
平面αの法線ベクトルを出す。それをM↑とする。
原点を通って方向ベクトルがM↑の直線mを出す。
直線mとαの交点がH。

(b)
平面α上の点Hは
AH=sAB+tACとあらわせる。sとtはパラメータ。
OH=OA+AH=OA+sAB+tAC
|OH|^2をsとtの式であらわし
これが最小となるsとtを求める。

(c)
外積でさっくり
637132人目の素数さん:02/06/21 02:01
すいません、ちょっと定義がわからないんですが教えていただけませんか。
集合Xはradially closedってどういう意味ですか?
638132人目の素数さん:02/06/21 02:02
>>632
(1)
>>(Z4乗+Z2乗+1=0)を満たす
両辺に(−1+Z2乗)をかける

(2)略

(3)展開
639611:02/06/21 02:06
嵐じゃないので普通に解いてください。
お願いします。
640132人目の素数さん:02/06/21 02:06
>>632
(1) Z^6-1=(z^2-1)(z^4+z^2+1)=0
  ∴z^6=1

(2)|z^6|=1より|z|^6=1
  ∴|z|=1

(3)z=cos(kπ/3)+isin(kπ/3) (ただしk=1,2,4,5)
  よって|z-i|^2+|z+i|^2=(1+1)*2=4
641132人目の素数さん:02/06/21 02:07
>638
(1)
Z6乗(−1+Z2乗)=(Z4乗+Z2乗+1)(−1+Z2乗)
ってことですか?
>>639
不定積分は無理。
嵐かどうかは本人が決めることではない。
643132人目の素数さん:02/06/21 02:09
はーい質問!
「^」ってなんの意味ですか?
644132人目の素数さん:02/06/21 02:09
>>632
(1) 条件式を使って次数を下げる。
(2) zを極形式で表現。(1)と比較。
(3) 中線定理を用いる。
645132人目の素数さん:02/06/21 02:10
>>641
A=0の両辺にBをかける → AB=0
646640:02/06/21 02:11
(3)zの式を出すまでもなかったな。
  |z|=1より(与式)=4で明らかじゃん・・・
zを直接出さないと気がすまない人が多いね
648626:02/06/21 02:12
すみません、漢字間違えてました。
錘ではなくて錐です。
649うう、わかんない・・・:02/06/21 02:15
>640
なんでZ6乗-1になるの〜?
-1はどこからきてるの?
650642:02/06/21 02:16
恐るべき神!
>>649
いきなり-1が出てくるのは答えか背景を知っているから
z^4=-(z^2+1)
z^6=z^2*z^4=-z^2(z^2+1)=・・・と次数を下げるのが自然か
652640:02/06/21 02:18
>>649
ただ右辺を展開しただけ。
653640:02/06/21 02:20
一般に
x^n-1=(x-1){x^(n-1)+x^(n-2)+・・・+x+1}
が成り立つ。

頻出なので覚えておけばよい。
次数を下げるだけで答えが出る保証もないのか
zについて解いてしまうのが安全確実だった
前言撤回
655うむむ:02/06/21 02:27
うーん、ということは653さんの公式みたいなのを使うから、
Z2乗-1をかけるってことですよね。。
それを展開して整理すると、Z6乗=1がわかるってことですね!
656132人目の素数さん:02/06/21 02:32
条件式が偶数次数の項ばかりなので、
いっそのことz^2を1つの文字として見たら、
z^2+z+1=0のように考えられるから、>>653の公式適用に気付きやすい鴨。
657(^−^):02/06/21 02:36
(1)は分かったとして、次の|Z|の値を求める問題。。
640さんは|Z6乗|=1なので...ってしてるけど、|Z|=±1にはならないの?
誰か教えて〜。
658132人目の素数さん:02/06/21 02:38
cosh λt=s/s^2-λ^2

の証明をしたいのですがそもそもcosh λtって

1/2(exp(λt)+exp(-λt)であってます?
どうしてもうまくいかんのですが。
分母にsが出てこないムードむんむんです。
659658:02/06/21 02:39
あ、658はラプラス変換です。
660132人目の素数さん:02/06/21 02:43
>>658
手始めにexp(ax)でも変換して見ろ
661658:02/06/21 02:46
>>660
つーことはハイパボリックの式はあってるってこと?
L exp(ax)=1/(s-a)ですかね?
662132人目の素数さん:02/06/21 02:48
>>657
複素数z=x+iy(x,yは実数)において、
|z|=√(x^2+y^2)
のことをzの絶対値といいます。
つまり、実数のときと同じ、複素数でも絶対値は必ず≧0です。
663132人目の素数さん:02/06/21 02:50
>>661
それがわかってるのに、>>658ができないのかい?ふむ。
1/2(1/(s-a) + 1/(s+a)) = ??
664658:02/06/21 02:54
>>663
分数の計算ができない大学生が、今ここに誕生してました。
すいません。。。
665132人目の素数さん:02/06/21 03:19
正五角形をコンパスと定規(測らない)だけで書く方法忘れちゃいました。
しってる方いますか?
666132人目の素数さん:02/06/21 03:24
>>638
どう展開すんの?
667132人目の素数さん:02/06/21 03:24
>666
??????

1辺の長さが1の正五角形の対角線の長さは(1+√5)/2
だから、√5/2が作図できればよいがそれは簡単。
670666:02/06/21 03:41
>>668
すみません。質問した者ではないですが、
展開のしかたが分からないのです。
671132人目の素数さん:02/06/21 03:57
俺は思うんだが、
2乗してー1になる数(i)を考えないと2次方程式が解けないんだったら、
同じように4乗してー1になる数を考えないと4次方程式は解けないんじゃ無いのか?

そこんとこどうなの???
672132人目の素数さん:02/06/21 04:00
>>671
(1+i)/(√2)
これで満足?
673132人目の素数さん:02/06/21 04:00
>>671
複素(数)平面で考えれ
ちょっと前に有理化のこと教えてくださった先生方、ありがとうございました。
>>671
そう言う疑問を持つのは、マジな話センスが良い。
ちゃんと考えて、n次方程式でnが2より大きくなっても、複素数まで考えれば十分。
ということをガウスらが証明した。それが代数学の基本定理というもの。
677 ◆GaussrLU :02/06/21 07:39
>>296, >>494
セールの数論講義には二次形式の話が載ってたから,
それを使えば結論が出せると思います.
数論の3つの真珠という本もその話題に少し触れていました.
678132人目の素数さん:02/06/21 07:49
>>677
Nathanson「Additive Number Theory」 Springer
にちゃんとした証明(2次形式使う奴)が載ってるよ。
これは、このへんの分野の標準的な教科書かな。
679132人目の素数さん:02/06/21 08:11
「∫***dx を解いてください」っていう質問多いな
この言い回しを見ただけで,「あ,こいつには説明するだけ無駄だ」と思って報知決定だな
680132人目の素数さん:02/06/21 08:14
「4乗してー1になる数を考えないと4次方程式は解けないんじゃ無いのか?」
に関してはあまりセンスがいいとは思えない
681132人目の素数さん:02/06/21 08:19
ベアストウ法をやさしく説明してください。
ふと思いついた問題なんですが、
1から9までの数字を1回ずつ使ってできる最大の数といったら
なんになるんでしょう?
記号は何でも使ってよいこととします。
ただし、8を横向きにして∞とかいうのはナシでお願いします。
既出でしたらスミマセヌ。
683132人目の素数さん:02/06/21 11:54
『使う』とは何を?
数字を。
685132人目の素数さん:02/06/21 11:58
ダメダコリャ
>>682
(2+3+4+5+6+7)/(9-8-1)
687132人目の素数さん:02/06/21 13:01
3456789/((√√√…√2) - 1)

ルートの数を増やせば、全体はいくらでも大きくなる
688132人目の素数さん:02/06/21 13:34
(N)!
((N)!)!
(((N)!)!)!
  ・
  ・
  ・
いくらでも大きくなる
3↑↑↑↑↑↑↑↑↑↑↑↑↑↑↑↑3
690132人目の素数さん:02/06/21 14:57
複素数zに対して、w=z+1/z-2とおく。
複素数平面上で複素数zをあらわす点が虚軸上を動くとき
複素数wをあらわす点はどんな三角形を描くか。

さっぱりわかりません。だれかおしえてください
691名無人 ◆TCcC3EVE :02/06/21 15:09
まず与式をzについて解いてから、虚数の条件に代入

虚数の条件ってのはバーつけたらマイナスになるってことね。
692132人目の素数さん:02/06/21 15:17
すみません。もう少し詳しく説明お願いします。
z=ai
w=-2+i(a-1/a)

三角形になるか?
694132人目の素数さん:02/06/21 15:37
z+1/z-2を(z+1)/(z-2)のつもりで書くDQN
>>690
w=(z+1)/(z-2) ?
w=z+1/(z-2) ?
696132人目の素数さん:02/06/21 15:39
加減算優先ってどこの国?チョソ?
697132人目の素数さん:02/06/21 15:51
三角形?
一次分数変換+反転で検索
699132人目の素数さん:02/06/21 16:19
>>677-678
>>296です。う〜ん。微妙ですね。
>セールの数論講義には二次形式の話が載ってたから,
>それを使えば結論が出せると思います.
つまり奇数nについて
 n=x^3+y~y+z^3が整数解をもつ⇔n≡1,3,5 (mod 7)
またはこれを含む命題そのものは結局のってるんでしょうか?
注文するのは面どいのでとりあえず本屋と図書館にある整係数2次形式の本を
あらかたあたってみたんですが結局わからずじまいでした。
>>677さん数論講義にのっててさっきの問題の十分条件になりそうな2次形式
の命題ってどんなのですか?
>>678さんお手数ですが証明の概略だけでもちょっとおしえて下さい。
よろしくおねがいします。貧乏人なのでそれからどちらかを注文しようかと思います。
700132人目の素数さん:02/06/21 16:35
釣れた釣れた
701132人目の素数さん:02/06/21 16:45
>>678
ありゃりゃアマゾンで検索したらNathansonの教科書って2冊ありますよ?
1.Additive Number Theory : Inverse Problems and the Geometry of Sumsets
(Graduate Texts in Mathematics, 165)
2.Additive Number Theory : The Classical Bases
(Graduate Texts in Mathematics, Vol 164)
これどっちもよまんといかんのですか?結構たかいんですけど。
整係数2次形式に詳しいのはどっちでしょう?
ちなみに>>699
 奇数nについて
 n=x^2+y^2+z^2が整数解をもつ⇔n≡1,3,5 (mod 7)
のまちがいです。
702132人目の素数さん:02/06/21 16:55
>>701
さらに訂正
 奇数nについて
 n=x^2+y^2+z^2が整数解をもつ⇔n≡1,3,5 (mod 8)
なおす必要もないんだけど。一部ネタとおもってる人がいるみたいなので。
703132人目の素数さん:02/06/21 17:02
>>702
ムキになるなよ ムキになるついでにnは正としとこうぜ

本が高いのなんだのっていうなら
とりあえずセールが買いだな
いい本だから損はない
704132人目の素数さん:02/06/21 17:05
>>703
やっと話が通じそうなひとがいた。
セールの本にのってる2次形式の理論からさっきの命題みちびきだす
方法ってわかります?それともやっぱりひとひねりいれないとなかなか
おもいつかないぐらいのところまでしか解説してないんですか?
705132人目の素数さん:02/06/21 17:15
>>699-704
はあ、やっと数論講義の内容簡単に紹介してるページみっけた。
>セール教授のエコール・ノルマル・シュペリオールでの講義をまとめた.
>第1部で,平方剰余の相互法則,p進体などの準備をもとに整係数2次形式を
>考察する.第2部で,ディリクレ算術級数定理,保型関数を扱う.
アマゾンばっかたよってたらあかんゆ〜ことね。いかにものってそう。
こっちにしよかな・・・。
706132人目の素数さん:02/06/21 17:18
>>704
あの定理はガウスが エウレカ num=△+△+△!
と日記に書いたほどの定理と同じ内容なんだろ
難しいんだよ
Serreの本では たぶん Hasseの原理のあとに
でてくると思う つまりp進体での解の有無に帰着すると
おもう それを勉強するので悪いわけはないが

もっと初等的な証明もあるはずで
いや 初等的に限らなくてもいいけど
ヴェイユの全集に収められている論文に解説があったと思うけどね

といってもそれでは君には役立たないか?
707132人目の素数さん:02/06/21 17:23
>>706
>Serreの本では たぶん Hasseの原理のあとに
>でてくると思う つまりp進体での解の有無に帰着すると
>おもう それを勉強するので悪いわけはないが
これつまりさっきの方程式が整数解をもつ...(A)
⇔すべてのp進体で解を持つ。...(B)
をしめしておいて(B)の方をn≡1,3,5(mod 8)のときにしめすってことですか?
708132人目の素数さん:02/06/21 17:29
>>707
ほぼそういうことです
なぜほぼかというと整数解と有理数解の違い
でちょっと微妙だから
(もちろん実数での解はあるからいい nは正としとこうぜ)
709132人目の素数さん:02/06/21 17:36
>>708
なるほど。やっと何を勉強すればいいかわかってきました。
そんでその方針の証明が“数論講義”にのってるんですね。
やっぱこれ注文することにします。
>nは正としとこうぜ
ういっす。
710132人目の素数さん:02/06/21 18:29
cotA+cotB+cotC=√3を満たす三角形ABCはどんな三角形か?

ちなみにcotAcotB+cotBcotC+cotCcotA=1でこれを利用すると与式は
cot^2(A)+cot^2(B)+cot^2(C)=1となりさらに変形していくと
sin^2(A)sin^2(B)+sin^2(B)sin^2(C)+sin^2(C)sin^2(A)=4sin^2(A)sin^2(B)sin^2(C)
となるんですがここで詰まってしまいました。
どのように解いていけばよいでしょうか。教えてください。
>>710
cotAcotBcotC=kとおく。
cotA+cotB+cotC=√3
cotAcotB+cotBcotC+cotCcotA=1から
cotA,cotB,cotCは x^3-√3x+x-k=0 の三解
この方程式を変形すると
(x-1/√3)^3 + 1/3√3 - k =0
これのすべての解が実数でなければいけないのでk=1/3√3
よって cotA=cotB=cotC=1/√3 で正三角形。
>>711
ありがとうございました!
713132人目の素数さん:02/06/21 20:47
o(m,C)においてのキリング形式が
B(X,Y)=(m-2)Tr(XY)
になることを示せ。

B(X,Y)=Tr(ad(X)・ad(Y))ということしかわかりません。
714132人目の素数さん:02/06/21 21:31
自作自演っぽいな
715132人目の素数さん:02/06/21 22:04
△ABCにおいて、
AB=x、BC=2x、CA=9であるとき、
次の問いに答えよ。
(1)xの値の範囲を求めよ。
(2)cosCの最小値とそのときのxの値を求めよ。

答えは(1)3<x<9
   (2)x=3√3のとき 最小値√3/2
らしいんですが、解き方が全くわかりません。
どなたかお願いします。
716N:02/06/21 22:12
1は三角形成立条件
717132人目の素数さん:02/06/21 22:13
3角形というのは2辺の長さの和は残りの1辺の長さよりも長くなくてはならないんだ。
だからa、b、cという長さの3角形というのが有ったら必ず
a+b>c、b+c>a、c+a>b
が成り立つんだよ。
それからABとBCの間の角度をθとおくとθに関する定理があったはずだ。
718N:02/06/21 22:30
平面上の四点A,B,C,Pにおいて
PA(以後ベクトルでお願いします)+2PB+4PC=(K-1)AB(Kは実数)が成立している
点Pが三角形ABCの内部にあるKの条件を求めよ。
----------------解答--------------------------------------
与式を変形して K*PA+(3-k)PB+4PC=0
左辺の各項の係数が正であれば題意を満たすことができる。
∴K>0かつ3-K>0⇔0<K<3 以上
このように解き、先生にチェックしてもらったのですが
「いいん・・・ちゃう?」
と、、、まぁ、曖昧な返事でした。記述問題としてみたとき、このような
答案で果たしていいのでしょうか?宜しくお願いしますm(_)m
719132人目の素数さん:02/06/21 22:41
>>718
>与式を変形して K*PA+(3-k)PB+4PC=0
>左辺の各項の係数が正であれば題意を満たすことができる。
なんで?
720132人目の素数さん:02/06/21 22:41
俺なら減点しる
省略しすぎ
721715:02/06/21 22:41
すみません、715なんですが。
(1)はわかりましたが(2)がやっぱりわかりません。
恐縮ですが、もう少し具体的に教えて頂けませんか・・・
出来の悪いヤツですみません・・・
722132人目の素数さん:02/06/21 22:43
cosC を x で表してみる > 721
>721
いわゆる余弦定理という奴ね。
724132人目の素数さん:02/06/21 22:46
>>721
余弦定理を使うんよ。
725N:02/06/21 22:49
αPA+βPB+γPC=0とあるとき、一般に面積BPC:CPA:APB=α:β:γ
が成立する。 と予備校の授業で習ったので、
ABCの内部にPが存在する⇔α、β、γ>0
という考えを前提にしたつもりです。さすがに無理があるかなぁ・・・・
今になって少し無理を感じていますが(^^;)
726132人目の素数さん:02/06/21 22:49
>>718

{K*PA+(3-k)PB}/3=(-4/3)PC くらいは変形しろ
727132人目の素数さん:02/06/21 22:51
どうでもいいことだが α,β,γ<0 でもいいだろ > 725
>718
そのくらいで十分だと思う。
理解しているということも十分分かる。
729競馬:02/06/21 22:53
すまねぇ質問です。
1枠2頭、2枠2頭、3枠2頭、4枠2頭、5枠2頭、6枠2頭、7枠3頭、8枠3頭
のフルゲート18頭立てのレースに枠番3連複、枠番3連単という馬券があるとしよう。
1−1−2、4−5−8、2−2−6、などといった組み合わせは一体何通りになる???3連複は数字の組み合わせで順序は考慮しない。(2−1−1も1−2−1も同じ)
3連単は順序を考慮する。

こちらの方が本スレっぽいので、質問を移させてもらいました。
>726
それは趣味の範囲だよ。
731132人目の素数さん:02/06/21 22:55
>>718
AP↑=xAB↑+yAC↑の形にして、

Pが三角形ABCの内部

0<x+y<1, x>0, y>0

を用いるのがよいよい。
732一般性を失わない座標設定について:02/06/21 22:56
大学受験生の質問です。

例えば複素数の問題で複素数平面状の3点が三角形を形成するための必要十分条件は「(3つの複素数z1 z2 z3についての式)」であることを証明せよと言う問題で、
二つの複素数を勝手にz1=0,z2=1とおいてしまっても一般性を失わないような気がするんですけどこれはダメですか?

よろしくお願いします。
733132人目の素数さん:02/06/21 22:59
「一般性を失わない」という意味がわかってない
全然駄目ぼ
734132人目の素数さん:02/06/21 23:01
またもやすみませんなんですが。
715の(2)、余弦定理でcosCを出すと
x^2+27/12x になるんですが、
これからどうすれば良いのでしょうか。
735132人目の素数さん:02/06/21 23:02
創価層状だ > 734
>734
微分するなり、相加相乗を使うなり好きにしておくれ。
相加相乗を使うなら、
(1/12){x+(27/x)}に変形するンよ。最後に変域の確認を忘れずに。
737N:02/06/21 23:07
>>727
たしかにかまわないですけど、結局問題を解くにあたってPCの係数が4なんで
ムダでしょう。
738132人目の素数さん:02/06/21 23:07
>>1の名前は“132人目のともよちゃん”
・タイトルの前後は◆
>>1がローカルルールの変更を申請してくる
なになに、まだ準備するのは早いでしょ。
740一般性を失わない座標設定について:02/06/21 23:17
732です。
全くダメですか?
任意の複素数平面上の三角形に対して一つの点を原点、もう一つを1にくるような座標を新たに考えればいけるんじゃないんでしょうか?
>740
では、一般性を失わない事を証明してください。
742132人目の素数さん:02/06/21 23:22
>741
だから、>740が説明してるじゃん。
743132人目の素数さん:02/06/21 23:24
すべての三角形は0と1を2頂点にもつと考えて表せる。と言うのも大きさの問題は座標そのものを縮めたり伸ばしたりすると考えればいいのでは?ということが言いたいのですが。
>740
それは三角形に対して、その2頂点を複素平面上の点0と1に対応させることができる
と言っているだけで、元から複素平面上にある三角形に対して、複素平面上のどこにあっても
その式(何だか知らないけど)が成り立つという事を言わなければならないと思うのですが。
0と1を2頂点に持つ「特別な」三角形についての話しではないですよね?
745132人目の素数さん:02/06/21 23:34
715の者です。無事に解けました。
皆様丁寧にありがとうございました。
>743
もう少し言えば、座標の伸縮をしたとして、どんな式でも普遍になる…
ということはないよね?
747132人目の素数さん:02/06/21 23:39

z1,z2,z3が三角形⇔0,1,(z3-z1)/(z2-z1)が三角形

じゃねえの?
748740:02/06/21 23:40
「複素数平面上の任意の三角形とその外側に元の三角形の一辺を一辺とする正三角形を三つ書く。
外側の三角形の重心の重心と元の三角形の重心は一致することを示せ」(うろ覚えなのでちょっと違うかも)
と言う問題では元の三角形の2点を0と1にしても一般性を失わないと書いてあったのでそれにヒントを得たんですが・・
この場合はどうして良いのでしょうか?
749132人目の素数さん:02/06/21 23:45
(n+1)(n+2)(n+3)……(2n)=2^n・1・3・5・……(2n-1)を数学的帰納法で証明せよという問題ですが、
どうしても息詰まってしまうのです。
よろしくお願いします。
750740:02/06/21 23:47
(z1-z2)/(z2-z3)=(z3-z1)/(z2-z1)みたいな感じの式です。因みに。
>>749
息詰まった所まで書くべき。
752いなか者:02/06/21 23:49
>>729
とりあえず、三連複は114、三連単は506通りになった。
多分間違っていない。
753132人目の素数さん:02/06/21 23:50
>>732, >>740
「(3つの複素数z1 z2 z3についての式)」がたとえば「f(z1,z2,z3)=0」
というものである場合・・・。

「f(Z1,Z2,Z3)=0」⇔「f(a*Z1 + b, a*Z2 + b, a*Z3 + b)=0」
が如何なる複素数a,b(但しa≠0)に対しても成り立つ。

ということが確認できれば
「二つの複素数を勝手にz1=0,z2=1とおいてしまって」
考えてもOK。

つまりモンダイの3点に関する条件が1次変換で不変である
ならOK。1次変換で変わってしまうなら駄目。
754740:02/06/21 23:56
一次変換というのが良く分からないんですが、>>750の通りなのでf(z1,z2,z3)=0は満たしているような気がするんですが
どうでしょう?因みに一次変換が現行過程では削除されてしまっている以上はこういった勝手な座標導入は証明しようがないのでバツですか?
感覚的に明らかなような気がするんですが。

>740
748の問題なら三角形の位置や大きさに関係のない性質。
732の問題だとそうはいかない。
747のような変換を考えればいいのだろうが、それを説明するのなら
元の問題をストレートに考えたのとどちらがやりやすいか。
756いなか者:02/06/22 00:01
>>749
k→k+1とするときに
左辺の因子はk+1が無くなり、2k+1と2k+2が加わる。
相殺して2と2k+1が加わる。後は自分で・・・というよりもう答えか。
行き詰まるというよりは、取っ掛かりが無いのでは?
757132人目の素数さん:02/06/22 00:07
∫sinx/xdx
はどうなりますか?
758競馬:02/06/22 00:07
>>752
ありがとう。それで正解です。
759132人目の素数さん:02/06/22 00:09
>757

cos(x^2)/x^2
760132人目の素数さん:02/06/22 00:11
>>757
logx
761753:02/06/22 00:20
>>754
753で言う「1次変換」とはz→az+bという変換のこと。
ただしa,bは複素数の定数でa≠0。
f(z1,z2,z3)={(z1-z2)/(z2-z3)}-{(z3-z1)/(z2-z1)}
だったらそのような変換で不変ですね。

>因みに一次変換が現行過程では削除されてしまっている以上は
>こういった勝手な座標導入は証明しようがないのでバツですか?
>感覚的に明らかなような気がするんですが。

わるいけどこのへんはわかりません。
762759:02/06/22 00:21
>760

びぶんすると1/xとなってちがうじゃねーか
ばーか
763いなか者:02/06/22 00:32
>>759
しかし759の式もうまく微分できない。
面倒で悪いが導出を書いてくれないだろうか。
馬鹿ばっか。
>>763
あんたからかわれているんだよう。
>757
似たような式が最近あったような、(でも覚えてない)。
それであなたはからかわれてしまうんだろうと思います。
ウン、からかわれてるのはこちらのほうか?
767132人目の素数さん:02/06/22 01:08
∫e^(-x^2)dx

を積分してみてください。よろしくお願いします。
768いなか者:02/06/22 01:11
>>766
まあそう言わんと、
誰も答えてくれていない簡単な問題に答えていこうじゃないか。
>>767
さすがにそのネタはわかるぞ。一応統計やっているから。
769132人目の素数さん:02/06/22 01:17
>>768

「数学@2ch掲示板」のどっかでこの問題を見たんですが、
どこか分からなくなってしまったんです。
なにが「ネタ」なのかも分からないんで、なにかコメントよろしく・・・
「統計」やってると???
770132人目の素数さん:02/06/22 01:24
>>767
e^(-x^2) = 1 - x^2 + x^4/2 - x^6/6 + ...
だから
∫e^(-x^2)dx = C + x - x^3/3 + x^5/10 - x^7/42 + ...
771132人目の素数さん:02/06/22 01:26
>>770

あら、とても早くにありがとうございます。
高校の範囲ではなかったようですね。
772いなか者:02/06/22 01:28
>>769
ああ、「今日はあほが入れ食いだ」とか言われてるのかな。
それでも答えてしまうお人よしなわたし・・・
問題の式は、不貞積分は初等関数では表せないが、-∞〜+∞の定積分ではπか
何かになる。正規分布がらみの計算によく使う。
違っている場合はちゃんとした方が答えてあげてください。
773いなか者:02/06/22 01:30
>>770
なるほど、そういう表現があるのか。
やっぱりあほの入れ食いだ。
774 :02/06/22 01:32
次の数列の一般項Anと、初項から第N項までの和Snをそれぞれ求めよ。

1,1+3,1+3+5,1+3+5+7,・・・・

夜遅くにごめんなさい。。お願いします!!

>>732=740=750
ていうか、最初から>>750の式を書いておけっつーの。
その式を書かないで、z1=0,z2=1で一般性を〜とかいったら、
どんな式でもz1=0,z2=1としていいと言ってるように聞こえるんだよ。

説明する時は面倒がらずにちゃんと書くようにしる。
776132人目の素数さん:02/06/22 01:37
>>774
足し算しる。
1, 4, 9, 16,・・・
777132人目の素数さん:02/06/22 01:46
>>774
a(1)=1,a(2)=3...a(n)=2n-1として
AnはΣ_[k=1,n]a(k)。あとは和の公式でも使って自力で計算しる。
778132人目の素数さん:02/06/22 01:51
すいません、この積分できる方いらっしゃいませんでしょうか?
∫[0,x](tanh(t)/t)dt
昨日一日かけて考えたんですがだめでした。なんでも特殊関数を使うようなのですが。
779132人目の素数さん:02/06/22 02:00
>>778
cosh(x)/x^2
780132人目の素数さん:02/06/22 02:01
>>778
logx
781  :02/06/22 02:05
>777
ありがとうございます!
答えは一般項An=2n−1、
和Sn=1/6n(n+1)(2n+1)と出たんですが、あってるでしょうか?!
782778:02/06/22 02:07
>>779
それ微分してみたら(-2cosh(x)+xsinh(x))/x^3になっちゃいましたが。
>>780
それ微分してみたら1/xになっちゃいましたが。
783132人目の素数さん:02/06/22 02:10
>>778
>なんでも特殊関数を使うようなのですが。

・誰が言ったの?
・どんな特殊関数を使うか言ってた?
784132人目の素数さん:02/06/22 02:10
>>781
一般項 An=Σ(2n-1) = n^2
わかってるとおもうけど。
785778:02/06/22 02:15
>>783
・誰が言ったの?
問題をだした相手です。ただやり方は1通りであるかどうかはわからないので、
特殊関数を使わなくてもできるかもしれません。
・どんな特殊関数を使うか言ってた?
ガンマ関数やベータ関数のようです。ただ、この関数をそのまま利用するのではなく、
考え方を利用するのかも知れません。そのあたりはわかりません。
786  :02/06/22 02:18
>784
分かってませんでした。ありがとう〜★
787132人目の素数さん:02/06/22 04:23
>>778
確か昨日も>>635で質問してたよね。
昨日いろいろ調べたけどわからなくて、気になってた。
∫[0,x]tanh(t)dt だったら簡単に解けるんだけど…。

http://www.sosmath.com/tables/integral/integ32/integ32.html
ここの9番に載っている。特殊関数はベルヌーイ数のことかな?

俺は全然わからないからここまでだ。あとは誰か頼む。
788132人目の素数さん:02/06/22 04:51
あみだくじのゴールがどうして重ならないか教えて下さい。
789132人目の素数さん:02/06/22 05:27
>>788
横棒が0本の場合、
横棒が1本の場合、
横棒が2本の場合、
横棒が3本の場合、
・・・
と考えていけば?
790132人目の素数さん :02/06/22 05:33
>>788
ゴールから逆にたどっていく道順を考えたら?
結局背理法を使うことになるかな。
791132人目の素数さん:02/06/22 05:36
線形代数です。よろしくお願いします。

A,Bをn次実正方行列、Cを複素数体、Rを実数体とする。
P∈GL_n(C)があって  B=PAP^(-1)
となったとすると
Q∈GL_n(R)が存在して B=QAQ^(-1)
となることを示せ。
792直感:02/06/22 05:41
紐を横に並べてねじっていけばアミダくじと同じこと
一ヶ所ねじると行く先が交換される

│  │         │  │
├─┤    →.    \/
│  │    →.    /\
│  │         │  │

これを     →    こう書き直す
>754
>感覚的に明らかなような気がするんですが。

感覚的に明らかそうなことでも示せない事実なら
それは、あなたの能力以上の問題だということです。
一次変換を授業で扱っていなくても、何故一般性を
失わないのか?という理由を書くことが出来る人なら
満点をもらうことができるでしょう。
しかし、あなたは、「気がする」だの「感覚的に明らか」だの
いうだけで、「一般性を失わない」ということに疑いを持っている人を
納得させるのに十分な理由が、全く書けていないので
その方針での解答は、あなたには無理だと思います。
794754:02/06/22 09:36
>>755
なるほど位置や大きさではなくて三角形特有の性質の照明の場合はこういう置き換えも可能なんですね。

>>775
すいません。ちゃんと書くべきでした。

>>793
そうですか。では>>761さんが書いてくださったようにz=αω+βとおいてここでω=1,0とすれば3つの複素数はβとα+βとαz+βとなり、任意の点ということができそうですが(結局z=0,1にしたときと同じになるようになりそう)これならばOkでしょうか?
これならそう照明に時間とられないようなきもするんですが・・
あとこの考え方が一次変換というやつなのですか?
795まおまお:02/06/22 10:19
>>791

まずは P = R + iS ( R, S ∈ M(n; R) ) と置きましょう。

B (R + iS) = (R + iS) A ということですから、もちろん
BR = RA
BS = SA
という2つの式が成り立ちます。

ここで、例えば仮に R (or S) ∈ GL_n(R)であれば、求めるQはすなわち
R (or S)です。しかしもちろんそんなに都合良くはいかない(笑)ので
あって、R, S共に非正則かもしれません。

・・・と、ここまで考えると、RとSを用いて実正則行列を構成するや
り方が、浮かんでくることでしょう。
796778:02/06/22 10:19
>>787
はい、すいません。昨日も質問したんですが、
昨日は他にもたくさん積分の質問があったので流れてしまったかと思いまして。
教えていただいたHP拝見しました。
この形から見るとまだ確認はしていないのですが、どうやら
被積分関数をテーラー展開してから積分してるようですね。
ありがとうございました。
797132人目の素数さん:02/06/22 11:01
>>794
>これならばOkでしょうか?

全然ダメ。

みんなの言ってることが理解できてないようなので
普通にやった方がいいかも、、、

一次変換については、googleで「一次分数変換」という単語を検索してみてください。
たくさんの解説ページがあります。
798田舎者:02/06/22 11:06
多項式t^6-8に対するQ上の分解体であるCの部分体ってQ(√2,√3i)で、
Qの拡大体の次数は基底が{1,√2,√3i,√6i}で4でないの?
答えには次数12になってたけど、なんで?
根本的に間違ってる?
誰かおせーて。
(1、√2、√3i)
で3じゃないのか(√6iって、√2 x √3i だろう?)
A=Q(√2,√3i)
√2,√3iは明らか
かけて√6i∈A
√3iをかけて3√2i∈A
3で割って√2i∈A
√2で割ってi∈A
-√6iをかけて√6∈A
√2で割って√3∈A
まだ他にあるかな?
i,1,√2,√2i,3,√3i,√6,√6iはQ上一次独立だから
Aは8次以上の拡大体
まちごーた。
1,i,√2,√2i,√3,√3i,√6,√6iはQ上一時独立ね。3なんか入れても独立にはならないな。念の為。
802132人目の素数さん:02/06/22 12:08
>800
>かけて√6i∈A
>√3iをかけて3√2i∈A

-3√2では?
そもそも√3とiって分離できる?
やっぱ4で合ってるような気が
804田舎者:02/06/22 12:58
何人かの132人目の素数さん有難う。
で、結局4でええのか〜。
独学なので辛いけど、修行を進めます。
行列Mを次のように定義します。
M=[M[1,1]=p,M[1,2]=1-p][M[2,1]=1-q,M[2,2]=q](0<p,q<1)
ここでlim(M^n)を求めたいんですが、方針が全然分かりません。
n=2、3の場合を計算しても僕には規則性を見出せませんし。。。
だれか教えてください、お願いします。
>>805
λ^2-(p+q)λ+(p+q-1)=0
(λ-1)(λ-(p+q-1))=0
>805
行列の固有値、固有ベクトルを勉強してください。
P^(-1)APを利用して計算します。
(p+q-1)=a
|a|<1
(a-1)M^n→[[(a-p),(a-q)],[(a-p),(a-q)]] (n→∞)
809132人目の素数さん:02/06/22 14:11
>>795
その取り方がわからないのです。
kohonenの自己組織化のアルゴリズムってあるでしょ。
あれって入力Xi(i = 1,…,n)が与えられたときに出力Yj(j = 1,…,m)に
対する新しい重みWij(t+1)を求めるには、今の重みWij(t)が必要なんですよね?
つまり初期値Wij(0)が必要ってことですよね?この初期値が与えられてない
んですが、だいたいどのぐらいの値をおけばいいんでしょうか?
アルゴリズムの説明を読むと小さいランダム値としか書いてないんですが…。
ちなみに入力は(0.2~0.8, 0.2~0.8)ぐらいの値です。
811132人目の素数さん:02/06/22 14:26
「A⊃B⇔Aの補集合⊂Bの補集合」を示せ。

ってのがよく分かりません。直感的には分かるのですが、
証明はどうするのか・・・・
812悩める少年。:02/06/22 14:45
Σ(k=1〜∞)6/(k^4) が (π^4)/15 になるそうです。
解き方がわかりません。教えてください。よろしくお願いします。
813132人目の素数さん:02/06/22 15:11
>809

例えば
tR+(1-t)S
814132人目の素数さん:02/06/22 15:14
>811
記号の意味が理解できていないのだろうと思うけど

A⊃B

x∈B ⇒ x∈A
という意味だよ。
815132人目の素数さん:02/06/22 15:18
>812
ζ関数について書いてあるHPなんて腐る程あるから
googleなんかで検索してクレ
>>806-808
分かりました、ありがとうございます。
817811:02/06/22 15:27
>>814
と、いうことは
「x∈B ⇒ x∈A」の対偶をとるだけで示せる・・・
なるほど分かりました。
>>813
なんでそれが正則になるんですか?
819まおまお:02/06/22 15:57
意味的には、私の言わんとしたことと、>>813氏の意図は同じでしょう。
ただ、「R + αS」(もちろんαは実数)と置いた方が、少し見易いかも
しれません。
820まおまお:02/06/22 15:59
もちろん言いたいことは、
「全ての実数αに対してdet(R + αS)が0となるようなことはない」
つまり
「あるαで正則になる」
ということです。(背理法を使おう・・・)
821132人目の素数さん:02/06/22 16:12
>>748
その問題は「元の三角形の2点を0と1に」しなくても簡単。

三点をA,B,C,その座標をそれぞれa,b,cとおく。
△ABCの辺ABを一辺とする正三角形のもう一つの頂点をPとおく。
ただしPは直線ABに関してCと反対側にある点。
同様に辺BCに対してQ、辺CAに対してRをとる。
点A,B,C,P,Q,Rの座標をそれぞれa,b,c,p,q,rとおく。すると
p=b+w(a-b)、q=c+w(b-c)、r=a+w(c-a)
がなりたつ。ただしw=cos(π/3)+i*sin(π/3)。(以下略)
822821:02/06/22 16:15
>三点をA,B,C,その座標をそれぞれa,b,cとおく。

この一行は余計。消し忘れた・・・
823132人目の素数さん:02/06/22 16:15
>>821
話題ズレまくりだよ…
824821:02/06/22 16:17
そうだね(w
tR+(1-t)S

は生息

826132人目の素数さん:02/06/22 16:28
atan(0.15915)のだいだいの答えを教えて下さい
>826
windowsなら付属の電卓で計算できるでしょ
828小平邦彦:02/06/22 16:56
私に分からない問題などありません
>>826
だいたいの答でいいなら、0.15915。(正しくは 0.157826)
830132人目の素数さん:02/06/22 17:24
atan て誰だ?
数学でよく使われる表記を使えよ
831132人目の素数さん:02/06/22 17:45
あーたん
832132人目の素数さん:02/06/22 18:00
lim_[x→3]ax^2+bx/x-3=1を満たす定数a,bを求めよ。
やり方がいまいちわかりません。出来るだけ詳しく教えて欲しいです。
お願いします。
833132人目の素数さん:02/06/22 18:19
>>832
わからないならば、わからないままにして次に進みましょう。
時間掛けるだけ無駄です。
834832:02/06/22 18:20
ここ質問スレじゃん。
835132人目の素数さん:02/06/22 18:23
表記くらい勉強してからきましょう > 832
836132人目の素数さん:02/06/22 18:25
>>832
分母→0なら
分子→0
837832:02/06/22 18:26
>>836
その辺がよくわからないんですよ。
>>832 極限値をもつためには、分子 a x^2 + bx = x(ax + b) が x-3
で割りきれなければならない。たとえば、a=1, b= -3 とすればよい。
で、lim[x->3] x = 3。ありゃ、1にならないなあ。
839132人目の素数さん:02/06/22 18:30
>>832
正確じゃないけど
lim(ax^2+bx)=lim(x-3)
と考えれば分かりやすいのでは?
840名無人 ◆TCcC3EVE :02/06/22 18:35
>>832
lim_[x→3]ax^2+bx/x-3=1⇒lim_[x→3](ax^2+bx)/(x-3)=1でしょ?

まず、lim_[x→3]のとき分母が0に収束するのはいいよね?
それでもってlim_[x→3]のとき分子が0以外の定数に収束すると仮定したら
lim_[x→3](ax^2+bx)/(x-3)=m(定数)/0=∞となってしまい、1には収束しないでしょ。
だから1に収束するには分子、すなわちax^2+bxはlim_[x→3]のとき0に収束する必要がある。
つまり
lim_[x→3]ax^2+bx=a3^2+b3=9a+3b=0
3a+b=0
が成り立つ。
これをもう一度lim_[x→3](ax^2+bx)/(x-3)=1に代入してみる。
b=-3aより
lim_[x→3](ax^2+bx)/(x-3)=lim_[x→3](ax^2-3ax)/(x-3)=lim_[x→3]ax(x-3)/(x-3)

=lim_[x→3]ax=1

よってa*3=3a=1  a=1/3 このときb=-3b=-1
R+tSに訂正します

失礼しました
>832
教科書なんかでは
limf(x)、limg(x)に極限値があればlimf(x)g(x)=limf(x)limg(x)より
lim[x→3](ax^2+bx)
=lim[x→3]{(ax^2+bx)/(x-3)}(x-3)=0
ってやってるな。
だから9a+3b=0 ∴b=−3a
843ベッセル関数名無しさん:02/06/22 19:22
ゴメソ、再び書き込みます。この問題をどう解いたらいいか、という事も重要
ですが、どの論文雑誌を見たら出ていそうというかという事も含めてレス下さ
い。>>496

ベッセル関数の積分の漸近形を探しています。ご存知の方は教えてください。
【詳細】
整数次の第1種ベッセル関数 J_n(x) の区間 0からx の積分をA_n(x)とする。
xが大きなところの関数形を求めよ。ただし、0から無限の定積分はWeber積分
と呼ばれていてガンマー関数で書けることは知られている。またJ_n(x)自身の
漸近形はよく本に載っている。知りたいのは積分した A_n(x) の漸近形。
844132人目の素数さん:02/06/22 19:40
>>843
どういうものが欲しいのかががイマイチわからないんだけど
Watsonの本は見ました? そこに無ければ厳しいんじゃないかと。
あくまでもネットにこだわるならsci.mathかfj.sci.mathか
数理科学MLで聞いたらどうですかねえ・・・
>843
岩波の数学公式Vにも載ってなかったのかい?
846132人目の素数さん:02/06/22 20:21
40 名前:132人目の素数さん :02/06/22 18:49
eとiを用いてπを表せても、iとπを用いてeを表せん。

これいかに?
>>492 (もう見てないだろな)
R^2の元(x,y),(x',y')に同値関係〜を
(x,y)〜(x',y')⇔x=x' かつ y-y'∈Qとして入れる。
C(x,y)で(x,y)を含む同値類とする。
Y=∪[α∈H]C(α,0)
H=[0,1/2) ∪ {(1/2,1)に属する有理数}とする。
Hは明らかに非加算集合
Yに属する列で、その極限が、どのC(α,0)の閉包にも
属さないものが存在する。
実際、zn∈Yで zn=(xn,qn) xn->(1/2,1)内のある無理数
というものがその典型
848あい:02/06/22 20:39
iって2乗すると-1になる、ということは知ってますが意味がよくわかりません。詳しくいうとなんなの?
849132人目の素数さん:02/06/22 20:42
三角不等式|a+b|≦|a|+|b|を
a,bが複素数の時に示せ。

という問題は普通にするとマズイのでしょうか?

普通というのは、2乗して絶対値の性質|ab|≧abを使うやり方です。
詳しく言うと2乗すると-1になるんだ。
分かりにくくいうと判別式が負の時解が存在しないから
2乗して-1になる数があると仮定したんだ
>849
まずいだろう。複素数には大小関係は無い。
852あい:02/06/22 20:52
じゃあ複素数でi-sinθがY座標を表してるのはなんで?
http://www.yozemi.ac.jp/nyushi/sokuho/kobe/zenki/index.html
↑2002の神戸大文系の問題です。

これの解答の、(1)の終わりの方に、
与えられた直線は、(1-t^2)/(1+t^2)x-(2t)/(1+t^2)y=1なので、
円x^2+y^2=1との接点は、((1-t^2)/(1+t^2).-(2t)/(1+t^2))
とありますが、これはどうやってそういえるのでしょうか?
854132人目の素数さん:02/06/22 21:02
>>853
問題の番号くらい書いてくれてもいいと思うけど。
855854:02/06/22 21:11
>>853
問2だな。
公式そのまま。
円x^2+y^2=r^2の周上の点(x1,y1)における接線はx1x+y1y=r^2
856132人目の素数さん:02/06/22 21:15
i-sinθ


訳わかんねぇ。。。
>>856
isinθのことかと思われ

問 任意の整数nにたいして、次の数は7の倍数であることを示せ

       2n^7+1995n^4+5n

やっぱり帰納法ですか?nが自然数のときを照明してそのあとに負の数のときを証明?
問題のテーマに「剰余系の代数」なんて書いてあるのが気になるんですけど
>852
x+yi に (x,y) を対応させる。
しかし書き方に注意してもらいたい。
860 :02/06/22 21:24
nが自然数のとき、不等式1+1/2^3+1/3^3…+1/n^3≦1/2(3^1/n^2)
が成り立つことを数学的帰納法を用いて証明せよ。

なんですけど、Σ1/k^3が分からなくて解けません?
それともうまく右辺を使って、もうA<B B<C として解く?
…おねがいします
861132人目の素数さん:02/06/22 21:24
4次元って
縦横高さ+時間
だけじゃなくて
縦横+時間+なんかの要素
でもいいんでしょ?
2n^7+1995n^4+5n = 2n + 0 + 5n = 0 (in mod 7)
863860:02/06/22 21:29
不等式1+1/2^3+1/3^3…+1/n^3≦1/2(3−1/n^2)
の間違いでした(汗
864132人目の素数さん:02/06/22 21:31
n~(7-1)=1 mod7 やねぇ。お見事>>862
>860
狽ェ使えるなら数学的帰納法に頼らなくても求まってしまう。
866132人目の素数さん:02/06/22 21:43
頼む・・・
>>861
キボンヌ
すげー疑問に思ったことなんでお願いします
>861
大体なんで縦横にこだわるの?
868132人目の素数さん:02/06/22 21:48
>>867
いや別に何の要素でもいいんですか?
とにかく4つあればいいんですよね?
>>854-855
ありがとうございます。
問題に直接リンクできるかと想ったのですが、ちがったようです。
今度からは気をつけますね。
>865
ちょっと誤解を与える言い方だったかな。
Σ1/k^3の公式があるとかいうわけではない。
871132人目の素数さん:02/06/22 21:51
初歩的な質問で申し訳無いんですが、
√38を整数にするといくつでしょうか?
>868
確かに3次元は縦、横、高さで表現できるし
4次元といえば時間とSFでは決まっているけど
何でもいいよ。
2*19=38
874132人目の素数さん:02/06/22 21:55
>>872
どんな空間か創造できないですけど時間の要素だけがある1次元とかもありですよね?
>>874
数学の話をしてるとは思えないので、物理板もしくは哲学板に逝ってください。
876132人目の素数さん:02/06/22 21:59
>>875
はあ・・
とにかくありがとうございます
877132人目の素数さん:02/06/22 22:04
>873
√19って大体いくつですか?
程度が低くて本当にもうしわけないです。
6<√38<7
879田舎者:02/06/22 22:08
>>812
フーリエ級数展開を利用するんじゃなかったけかな〜。
4<√19<5
881132人目の素数さん:02/06/22 22:11
「分散」ってどうやって求めるんでしたっけ?
確か 1/n(Σ1≦i≦n(xi-平均)^2) 以外にもあったような・・・。
882132人目の素数さん:02/06/22 22:12
>878
>880
ありがとうございました。
>>881
二項分布
>881
(2乗の平均)−(平均の2乗)
{(1/n)(xi)^2}−m^2
885132人目の素数さん:02/06/22 22:32
f(x)=x^2+2x+5
このf(x)って何なんですか?
さっぱり意味が分かりません
マジです
お願いします
ネタ
887132人目の素数さん:02/06/22 22:33
ネタじゃなりません
マジです
お願いします・・・・
恒等式ってことですか?
本当にわかりません・・・
888132人目の素数さん:02/06/22 22:34
>>20世紀に活躍した仲間たちが船上パーティを開いた。
時代を代表するヒーロー達はお互いをたたえあいデッキでバーベキューをした…がしかしサザエの壺焼きを食べたドラえもんが、もがき苦しんだ。
その時、紙に謎の文字を残した『わし/はわた/かん』そして息絶えた。
その横にはなぜかチーズのかけらが…このパーティに参加していたのはドラえもん・リカちゃん・サザエさん・ガチャピン・ミッキー。
犯人はこの中に必ずいる
謎の文字を解読し、犯人を探せ











889743さん:02/06/22 22:35
sinh(x)の微分は、ただ普通に((e^x)-(e^-x))/2を微分すればよいのでしょうか?
それとも、(sin(x))'=cos(x)のようなものがあるのか教えて下さい。
f(x,y)=sin(x)・sinh(y)の
d^3/d^2xdyを求めたいのですが、、、。
お願いします。
890132人目の素数さん:02/06/22 22:39
「Pascalな人」ってどういう意味ですか?
sinh(x)とcosh(x)の定義式を見る。
その微分はそのまま普通にやればいいけど、三角関数と非常に似た関係がある。
(まったく同じではない)
練習として作ってみたら。
892132人目の素数さん:02/06/22 22:47
>>889
普通に微分すれば良し。
(sinhx)'=coshx、(coshx)'=sinhxになる。
>>885
ネタだとは思うがもしもの時の為に。
xについての関数
>>892
sinhxは合成関数じゃないのか?
だから(sinhx)'=coshx*(hx)'でわ?
俺の勘違いかもしれんが
>894
またネタかよ
896743さん:02/06/22 22:57
>>891-892 && >>894
ありがとうございました。
894どおりの回答を始めに出したのですが、
戻ってきた答案を見て初めてsinh(x)の存在を知りました。
どうもthanksです。
>894
勘違い。sinh はいぱ簿リックsin。
ネタ、ネタじゃないにしてもちょっとまずいかも。
898894:02/06/22 23:02
工房の癖に分かったふりして書きこんですみません
899892:02/06/22 23:06
ハイパボリック サイン エックス
sinh(x) = {e^x-e^(-x)}/2

ハイパボリック コサイン エックス
cosh(x) = {e^x+e^(-x)}/2

ハイパボリック タンジェント エックス
tanh(x) = sinh(x) / cosh(x) = {e^x-e^(-x)} / {e^x+e^(-x)}

次の方、どうぞ
900132人目の素数さん:02/06/22 23:06
>>884
思い出しました。ありがと==
攻防ならそのうちやるさ。
ん〜工房ではやらないんだった
902832:02/06/22 23:14
>>842
もうこれ以上は無理かと思うが出来るだけわかりやすく書いてください。
903132人目の素数さん:02/06/22 23:16
>>902
受験板のほうがわかりやすく教えてくれるよ。
904832:02/06/22 23:21
受験板のどのスレに書くの?
905132人目の素数さん:02/06/22 23:24
an^(a+b)+bnがa+bで割り切れるのは、
a=bかa+bが素数のときのような気がしますが、
それは正しいですか?
906132人目の素数さん:02/06/22 23:28
(x-a)(x-b)(x-c),....,(x-y)(x-z)=0
が証明できません。僕はヴァカですか?
907132人目の素数さん:02/06/22 23:31
>>906
証明できないのとは違う意味でバカです
908132人目の素数さん:02/06/22 23:32
ワラ田
909132人目の素数さん:02/06/22 23:46
ひとつ質問します。

「n次の多項式行列A(x)」と書かれた場合、
これは変数xに関するn次の正方行列とみなしていいのでしょうか?

正方行列であるかどうかが大切な問題だと思うので・・・
910132人目の素数さん:02/06/22 23:49
>>909
その文なら正方行列とはかぎらないに一票。
911132人目の素数さん:02/06/22 23:54
正方行列です。

そんなことは自分で判断できるようにしないとまともに生きていけないよ。
>909
正方行列ではないとしたら、n次ってなんのことだと思うの?
913132人目の素数さん:02/06/22 23:57
>>911-912
しまった。ほんとだ正方行列だ。
914132人目の素数さん:02/06/22 23:58
>>913=>>910>>909。念のため
>>1の名前は“132人目のともよちゃん”
・タイトルの前後は◆
>>1がローカルルールの変更を申請してくる

差し替え
くだらねぇ問題スレ ver.3.14159265358979
http://science.2ch.net/test/read.cgi/math/1024699741/

俺スレ立てられなかったので誰かよろしく
916132人目の素数さん:02/06/23 00:08
>>905
a=10,b=15とかでだめじゃないか?
917132人目の素数さん:02/06/23 01:35
3・4日来ないとあっという間に遅レスになるなぁ。
何か積み残された感のある>>454
n = 3 の時 まず4が先頭というのはしょうがないとして
LLLRRRを並べ替えたもの(6C3=20通り)のL,Rにそれぞれ
n = 2 での平衡二分検索木の並びを(Lは1から3、Rは5から7)
入れていくと題意にあうのでa(3)=6!/3!*a(2)^2=80通り。
以降は a(n+1) = ( 2* (2^n-1) ) ! / (2^n-1) ! * a(n)^2
とりあえず簡単な式にはならなさそう。
918132人目の素数さん:02/06/23 01:37
|ab|≦|a||b|  (a,b∈C)
を示したいのですが、これって本当に成り立ちますか?

こういういかにも自明っぽい事って証明するのが難しくて・・・お助け〜
>>918 複素数じゃ、成り立つとしても必ずイコールだぞ。
どこからこんな式持ってきた?
**複素数zに対して、次の2条件を考えます。
(1)1.z^2.z^3はすべて異なる。
(2)1.z^2.z^3は複素数平面上において一直線上
問1.(1)を満たさない複素数zを全て求めよ。
問2.(1)(2)をともに満たすzの範囲を求めよ。

よろしくおねがいします。

921132人目の素数さん:02/06/23 02:38
>>919
いや、教授が言ってたんですよ・・・たぶん間違いですね・・・

脳内教授か?
923132人目の素数さん:02/06/23 02:40
行列
(x^2 0 )
(0 (x+1)^2)
の単因子を求めよ。

教えてください。お願いします。
924132人目の素数さん:02/06/23 02:41
脳内?

リアルですよ・・・一応、私は難関であろう大学の1年です。
925923:02/06/23 02:44
すいませんずれました・・・
2次正方行列です。
(x^2....0..)
(0..(x+1)^2)
926132人目の素数さん:02/06/23 02:46
z=r*exp(iQ)とおく
(2).z^2.z^3は実数なのでQ=Nπ
(1)取り敢えずr<>1ならQに関わらずOK
  r=1でもQ<>NπならOK
927132人目の素数さん:02/06/23 02:46
2x+1=a(x二乗+x+1)+(bx+c)(x-1)がxの恒等式となるように、定数a,b,cの値を定めよ。

教えてくださいまし。
928訂正:02/06/23 02:47
すまん(2)は大間違い
929132人目の素数さん:02/06/23 02:48
>927
展開して両辺の係数が同じになればいいんだろう
930927:02/06/23 02:51
実際に展開してみると出来ないんですよね
931132人目の素数さん:02/06/23 02:59
927は放置でよろしく。

次の方、どうぞ
932132人目の素数さん:02/06/23 03:04
>>927
a=1 b=-1 c=0
933923:02/06/23 03:10
>>931
お願いします・・・
基本変形の仕方がわかりません・・・
934132人目の素数さん:02/06/23 03:12
>920
(1) (z^2-1)(z^3-1)(z^3-z^2)=0 ⇔ z=0,±1,(-1±i√3)/2
(2) 一直線の条件から
  Im(z^3-z^2)(z^2-1)~=0
⇔ Im{z^2(z-1)(z~-1)(z~+1)}=0
⇔ Im{z^2(z~+1)}=0

あとは適当にやれば、条件を満たす集合は、実軸と円であることがわかる。
当然、(1)で求めた部分を除く
935931:02/06/23 03:15
(右辺)=a(x^2+x+1)+bx^2-bx+cx-c
     =(a+b)x^2+(a-b+c)x+a-c
左辺と比較して a+b=0,a-b+c=2,a-c=1 これを解いて>>932
936927:02/06/23 03:31
>>932 >>935
ありがとうございました。
>936
どこが分からなかったのかを書いてくれると助かる
つーか分からないまま書き写して宿題提出,ってなことだけはやめろよ
>>791
一応解答例
AC=CB
C=R+iJ R,Jは実正方行列とする。

AR+iAJ=RB+iJBより
AR=RB
AJ=JB
sを複素数として
A(R+sJ)=(R+sJ)Bは恒等的に成立
しかしdet(R+sJ)=0を満たすようなsは高々n個
だから実数sでdet(R+sJ)≠0となるものが無限個取れる。
そこで一般に実数sに対しR+sJは正則
A=(R+sJ)B(R+sJ)^(-1)
>>923
互いに素だから・・・
940923:02/06/23 04:10
>>939
互いに素だから答えは
1とx^2(x+1)^2ですよね?

その導出過程はどうしたらいいんでしょうか?
「互いに素だから」で終わらせていいんでしょうか?
941脳内公理:02/06/23 04:11
>>921
式自体は間違ってはいないと思うが…。
a,bってのを複素数じゃなくて、例えばベクトルなどに拡張して考えれば、この式に意味が出てくる。
教授が言っていたのはそういうことでは?
>>940
> その導出過程はどうしたらいいんでしょうか?
基本変形でできるでしょ。

> 「互いに素だから」で終わらせていいんでしょうか?
漏れは単因子のこと良く知らないから、
誰か別の人教えてあげてください。
943132人目の素数さん:02/06/23 04:46
>>1の名前は“132人目のともよちゃん”
・タイトルの前後は◆
>>1がローカルルールの変更を申請してくる

差し替え
くだらねぇ問題スレ ver.3.14159265358979
http://science.2ch.net/test/read.cgi/math/1024699741/

俺スレ立てられなかったので誰かよろしく

944923:02/06/23 04:51
>>942
ありがとうございます。

ところで基本変形の
「ある行にある行のf(x)倍を加える」というのは、
x^-1をかけてもいいんでしょうか?
つまり「f(x)=x^-1」というのは許されるのでしょうか?
>>938
一応そこまでは自分でも導けてたんだけど
なぜそれがsの0でない多項式なのかが疑問で・・・
>938
>そこで一般に実数sに対しR+sJは正則

この「一般に」は不味いので

「そのようなsを1つ選べば」程度の表現に直しませ

>945
det(R+sJ)≡0だとすると

sにiを入れても0になってしまい仮定に反するから。
948答はあってる:02/06/23 09:03
小行列式を使う > 単位ん死

   / ̄   ̄ ヽ
  / ,,w━━━.、)   / ̄ ̄ ̄ ̄ ̄ ̄ ̄ ̄ ̄ ̄ ̄ ̄ ̄ ̄ ̄ ̄ ̄
  ! .fw/f_」」_|_|_i_)  | 新しいスレ立てましたわ
  ヽ|:::(6||f;j' ,fj'||)  | ◆ わからない問題はここに書いてね 37 ◆
 ∠|::i:!::|:|、_ワノ:i、 <  http://science.2ch.net/test/read.cgi/math/1024790384/l50
  .|::|< |::|ヽーノ`l:i;ヽ \_________________
  .ノ:ノ' i:::l `只´|:|i)::)
 (::(:i  |:::|ノ ) j:j|:(
>>947
わかりました。ありがとう
951132人目の素数さん:02/06/23 14:46
∫(1/x^3+8)dx[0→1]
なんですけど途中の部分分数展開の仕方がわかりません
どうか教えてください
952923:02/06/23 15:02
>>948
小行列ですか?
すいませんどのようにつかえばいいのかもう少し教えていただけませんか?
>>944
多項式の世界で考えてるんでしょ。
だから、
> 「ある行にある行のf(x)倍を加える」というのは、
> x^-1をかけてもいいんでしょうか?
はダメです。
954923:02/06/23 15:22
>>953
やはりそうですよね・・・
x^-1はx=0の時が問題になるからですか?
>>954
> x^-1はx=0の時が問題になるからですか?
そんなことは関係ない。
成分が多項式の行列で話をしてるんでしょ。
1/xは多項式ですか?
956923:02/06/23 15:32
>>955
うーん・・・単項式ですよね。
でももともとの成分のx^2も多項式ではないですよね?
その辺のところがイマイチよくわかりません・・・
>>956
もういいや。俺はパス。
他の人に聞いて下さい。
958132人目の素数さん:02/06/23 15:40
>>956
君以前A(x)とその転置行列tA(x)について正則行列P(x),Q(x)を
PAQ=tAととれることをしめせ。って問題きいてた子じゃないの?
既出の対角化して解く方法はわかんないの?単因子つかう方法も
理解しとかんとだめだけど。
959132人目の素数さん:02/06/23 15:51
じゃあ不親切な>>957を引き継いで。

>>956
1/xは単項式でもないです。
単項式とは累乗が0以上のものしか言いません。

したがって、多項式でもない。
だから、1/x倍することはできません。

もっと基本から見直したほうがいいかも。
960132人目の素数さん:02/06/23 15:55
>>956>>958
ああ、また違う問題か。ゴメソ。
961132人目の素数さん:02/06/23 15:58
>951
収束しないんですけど・・.
>956
激しく笑ったけど
最近は、多項式とか単項式っていう言葉は
中学とか高校では習わないのかな?
>951
分数の表記の仕方から学ぼう。
964923:02/06/23 16:01
>>959
恥ずかしい限りです・・・
わかりました。
基本を見直します。ありがとうございました。

はじめにバカな自分に付き合っていただいた>>957さんもありがとうございました。
>>962
最近そういった難しい言葉は
墨で黒く塗りつぶされた教科書を使っているそうです(w
966923:02/06/23 16:05
>>962
習ったはずなんですけど、累乗が0以上ということは全く頭にありませんでした・・・
恥ずかしいです。

基本変形のやり方がわからず1/x倍が使えたら、みたいに考えてました。
前に書いていただいた小行列式を使うというのもわからないし・・・

あーーーヤバイ・・・
967名無数:02/06/23 17:02
(-.-;か〜〜なりピントがずれた質問かもしれないんですけど、
関数が常に増加ってのを示すのは、『導関数≧0』を示せばいいんですよね?
『導関数>0』じゃ無いって事は
『導関数=0』は増加と見るんですよね。
って事はy=f(x)について全てのxについてyが常に一定値‥

例えばy=3とかも常に増加っていうんですか?

なんかおかしいっすよね??
なんなんだろ!誰か助けて!
968132人目の素数さん:02/06/23 17:15
>>967
「常に増加」は『導関数>0』で
「単調増加」が『導関数≧0』では?
969132人目の素数さん:02/06/23 17:16
>968
y=x^3は常に増加してませんか?
970132人目の素数さん:02/06/23 17:27
>>969
x=0に近傍で増加していないよ
971132人目の素数さん:02/06/23 17:31
x=0の近傍
の間違え
972国語の辞書と:02/06/23 17:31
数学書の使い分けをしてくれYO
973132人目の素数さん:02/06/23 17:32
>970
a<0<b なら a^3<0<b^3 だけど、増加してないですか?
974132人目の素数さん:02/06/23 17:33
「単調増加」とは「減少しない」の意味
975132人目の素数さん:02/06/23 17:35
>>973
??
その式の意味がわからん
976名無数:02/06/23 17:35
常に増加するための条件を求めよってので、本では答えが導関数≧0を使ってるんです
俺は>0ってして間違いました。

上の人が答えてくれてましたが、常に増加と単調増加って違んですね?
977132人目の素数さん:02/06/23 17:37
>>976
その関数は定数関数ではないんでしょ?
978132人目の素数さん:02/06/23 17:39
>975
なんだ。アホか。
話しただけ損だったYO
979132人目の素数さん:02/06/23 17:42
>>978
してないやろ
980132人目の素数さん:02/06/23 17:42
単調増加には狭義と広義がある
単調非減少という言葉もある
981132人目の素数さん:02/06/23 17:55
y=x^3は狭義単調増加ってことでいいですね
>981
いいですよ
f’(x)=0だけでは増加、減少は判定できない。
一点だけでf’(x)=0ならその前後も合わせて判定する。
ある区間でずっとf’(x)=0なら、その区間では定数。

広義の増加は定数になる範囲があっても良い。
狭義の増加は定数になる部分があったらダメ。
>976
常に増加と単調増加なんて区別は無い。
985132人目の素数さん:02/06/23 19:40
986132人目の素数さん:02/06/23 19:40
987132人目の素数さん:02/06/23 19:41
988132人目の素数さん:02/06/23 19:41
989132人目の素数さん:02/06/23 19:41
990132人目の素数さん:02/06/23 19:41
991名無数:02/06/23 19:46
1000!
992命名:埋め立てやさん:02/06/23 19:54
993埋め立てやさん:02/06/23 19:55
994埋め立てやさん:02/06/23 19:56
995埋め立てやさん:02/06/23 19:56
996埋め立てやさん:02/06/23 19:56
997埋め立てやさん:02/06/23 19:56
998埋め立てやさん:02/06/23 19:56
1000!
1000ゲッター:02/06/23 19:57
1,000
10011001
このスレッドは1000を超えました。
もう書けないので、新しいスレッドを立ててくださいです。。。